Elders Final Review - Full Chapter Questions

Ace your homework & exams now with Quizwiz!

4. The American College of Sports Medicine (ACSM) has designed programs to incorporate exercise into every patient encounter. Which of the following is the correct phrase for this program? 1. Back to Exercise 2. Health and Wellness 3. Exercise is Medicine 4. Exercise is Wellness

4. Answer: 3 Page: 20 Feedback 1. Back to Exercise is not the correct phrase for the program the ACSM designed. 2. Health and Wellness is not the correct phrase for the program the ACSM designed. 3. Exercise is Medicine is the correct phrase for the program the ACSM has designed to incorporate discussion of exercise into patient encounters. 4. Exercise is wellness is not the correct phrase for the program the ACSM designed.

6. The nurse practitioner sees a patient in the emergency department with reports of chest pain and nausea. To analyze the patient's condition, what further information is needed? 1. Whether there has been constipation. 2. A description of the pain. 3. Information regarding last food intake. 4. Information regarding urinary difficulties.

6. Answer: 2 Page: 153 Feedback 1. Constipation is not included in the major signs or symptoms of CAD. 2. Ask the following about chest pain: Precipitating/Palliating factors, Quality, Radiation, Severity, and Timing (PQRST). 3. Chest pain may be presented as heartburn or indigestion, particularly if the patient also has gastroesophageal reflux disease (GERD). GERD needs to be ruled out. 4. There are no associated urinary symptoms with chest pain.

6. Anne is a 50-year-old post-mastectomy patient diagnosed with secondary lymphedema. Based on her surgical history, which of the following may be the contributing factor to the lymphedema? 1. Gynecological cancer 2. Breast cancer 3. Urological cancer 4. Infection

6. Answer: 2 Page: 218 Feedback 1. Gynecological cancer is a contributing factor for secondary lymphedema, but it is not the best option for this scenario because of Anne's history. 2. Breast cancer is the best option and is a contributing factor for secondary lymphedema in Anne's case because of the previous mastectomy. 3. Urological cancer is a contributing factor for secondary lymphedema, but it is not the best option for this scenario because of Anne's history. 4. Infection is a contributing factor for secondary lymphedema, but it is not the best option for this scenario because of Anne's history.

8. The nurse practitioner performs a thorough systemic examination of a patient who reports diarrhea and pain in the upper left quadrant of the abdomen. One possible diagnosis would be: 1. Hirschsprung's disease 2. Pancreatitis 3. Appendicitis 4. Gastrointestinal inflammatory bowel disease (GIBD)

8. Answer: 4 Page: 5 Feedback 1. Hirschsprung's disease is an obstruction of the colon in infancy. 2. Diarrhea is not associated with pancreatitis. 3. Appendicitis is more common in younger patients than in older patients. 4. GIBD is manifested differently in a bimodal pattern.

9. As the nurse practitioner is teaching about the need for more exercise, the patient says that it takes away too much energy needed for other activities. The nurse practitioner explains that the benefits of exercise include which of the following? Select all that apply. 1. Increased flexibility. 2. Increased muscle mass. 3. Maintenance of optimal weight. 4. Lower blood pressure. 5. Decreased urinary output.

9. Answer: 1, 2, 3, 4 Page: 7 Feedback 1. The health benefits of regular physical activity are well documented and include flexibility. 2. Exercise increases muscle tone and mass. 3. Exercise uses calories from stored fat and allows for weight loss. 4. Exercise strengthens the cardiac muscle and thereby lowers blood pressure. 5. Exercise does not affect urinary output.

1. A patient asks the nurse practitioner what is meant by health promotion. Which of the following is the nurse practitioner's best response? Health promotion: 1. Includes activities that an individual performs proactively to increase health and well-being. 2. Is a process of keeping track of immunizations. 3. Includes a set of programs that help people cope with the disease. 4. Includes strategies that prolong life.

1. Answer: 1 Page: 6 Feedback 1. Health promotion includes not only preventive and health-protective measures, but also actualization of one's health potential 2. Immunizations are only one part of health promotion. 3. Health promotion occurs before the onset of disease. 4. Quality of life is the important focus of health promotion, not a prolongation of life.

11. In conducting a physical assessment of an older person, the nurse practioner assesses which of the following? Select all that apply. 1. Pulse oximetry 2. Weight 3. Color of hair 4. Functional assessment 5. Presence of pain

11. Answer: 1, 2, 4, 5 Page: 27, 28 Feedback 1. Pulse oximetry provides a reading of oxygen saturation for the end tissues. 2. Measuring weight provides information about nutrition and fluid status. 3. Hair color is not directly associated with health status. 4. The ability to function for ADLs is crucial to the health of an older person. 5. Pain is the fifth vital sign and provides information about underlying conditions.

11. When a patient is suffering from constipation, the clinician evaluates the etiology and contributing factors. Which of the following diagnoses are being ruled out in these evaluations? Select all that apply. 1. DVT 2. Fecal impaction 3. IBS 4. Botulism toxin 5. Bowel incontinence

11. Answer: 2, 3 Page: 35, 37 Feedback 1. DVT is a condition that produces unilateral lower extremity swelling and tenderness. 2. Fecal impaction is a differential diagnosis, so it needs to be ruled out. 3. IBS is a differential diagnosis that is being ruled out. 4. Botulism toxin is sometimes used for treatment of conditions arising from symptoms of chest pain, such as esophageal spasms. 5. Fecal or bowel incontinence is the inability to control bowel movements, causing stool (feces) to leak unexpectedly from the rectum.

12. Mrs. Parsons is 75 years old and is anticipating renewing her driver's license. The nurse practitioner conducts an eye examination and is concerned to find which of the following? Select all that apply. 1. Visual acuity better than 20/20. 2. Visual acuity worse than 20/40. 3. Abnormality in the six cardinal fields of gaze. 4. Ability to gaze downward. 5. Absence of nystagmus.

12. Answer: 2, 3, 5 Page: 28 Feedback 1. An acuity of 20/20 is normal. Better than that is good. 2. Visual acuity of worse than 20/40 suggests weakness in the ability to see. 3. Abnormality in cardinal fields of gaze may indicate a neurological condition. 4. Ability to gaze downward is normal. 5. Abnormality in the six cardinal fields of gaze with nystagmus or lack of a downward gaze may reveal neurological disease.

6. Older adults who have chronic conditions and are considering an increase in their physical activity should: 1. Start slowly, using lower weights with high repetitions. 2. Consult with a personal trainer. 3. Consult a health-care provider to help set physical activity goals that are realistic and safe. 4. Start with low repetitions but heavier weights.

6. Answer: 3 Page: 21 Feedback 1. Starting slowly with low weights and high repetitions may be a good start but must be monitored or initiated by an appropriate medical professional for instruction in proper use. 2. A consultation with a personal trainer is not the best option because some trainers may not be familiar with chronic conditions. 3. Consulting a health-care provider for physical activity goals that are realistic and safe is the best option. 4. Starting with low repetitions but heavier weights is not a good way to start an exercise program because it may result in injury.

6. A 60-year-old male enters the burn center for triage and treatment due to a burn he received at a campfire. His left arm has an area that is erythematous and painful, and another area has a blister. What does the nurse practitioner record as the degree of burn? 1. First degree 2. Second degree 3. First and second degree 4. Second and third degree

6. Answer: 3 Page: 98 Feedback 1. First-degree burns involving the epidermis are erythematous and painful but do not blister. 2. Second-degree burns involve the dermis and are characterized by blisters. 3. The patient presents with erythematous skin, painful with blisters, which indicates both first- and second-degree burn areas. 4. In third-degree burns there is no sensation when the wound is pinpricked.

7. Reduction of pain is the main goal for patients suffering from acute joint pain. The mnemonic PRICE is a helpful reminder of the treatment when this occurs. What does the "E" in PRICE stand for? 1. Elevation 2. Emergency 3. Elastic 4. Electrical

7. Answer: 1 Page: 73 Feedback 1. Elevating the joint above heart level may help reduce pain and swelling. 2. This is not part of the mnemonic. 3. This is not part of the mnemonic, although the "P" does stand for protection with a brace or wrap. 4. This is not part of the mnemonic.

1. The genitourinary and reproductive systems undergo many age-related changes. Which of the following statements is true about these changes? 1. The older male patient undergoes more gradual changes than does the older female patient. 2. The older female patient undergoes more gradual changes than does the older male patient. 3. Older male and female patients undergo the same changes. 4. The older male patient undergoes less gradual changes than those of the older female patient.

1. Answer: 1 Page: 281 Feedback 1. This statement is correct. Older male patients undergo more gradual changes than those of older female patients. 2. Older male patients undergo more gradual changes than those of older female patients. 3. Older male patients undergo more gradual changes than those of older female patients. 4. Older male patients undergo more gradual changes than those of older female patients.

1. Mrs. James is 78 years old and comes to the clinic for an annual checkup. The nurse practitioner addresses the psychosocial needs of older adults and specifically the need for which of the following? 1. Dependence. 2. Dignity, credibility, and respect. 3. Being alone. 4. Not being touched.

1. Answer: 2 Page: 428 Feedback 1. The psychosocial assessment of the older adult entails an evaluation of the following basic needs: autonomy and independence. 2. The psychosocial assessment of the older adult entails an evaluation of the following basic needs: dignity, credibility, and respect. 3. The psychosocial assessment of the older adult entails an evaluation of the following basic needs: communication and belonging. 4. The psychosocial assessment of the older adult entails an evaluation of the following basic needs: touch.

1. Neurological assessment of the older adult requires specific tools and considerations to assess patient mental status. Which of the following should be used to assess patient cognition? 1. Administer SLUMS, a short dementia assessment. 2. Administer MoCA v 7.1, a 12-item assessment to detect mild neurocognitive disorders. 3. Administer testing of cranial nerve III. 4. Administer Mini-Cog, a three-word recall with clock drawing capability.

1. Answer: 4 Page: 328 Feedback 1. SLUMS is a 12-item assessment used to detect mild neurocognitive disorders. 2. MoCA version 7.1 is a tool designed to measure the status of a patient's attention, concentration, executive function, memory, and orientation. 3. A complete neurological examination begins with the testing of cranial nerve function. 4. Mini-Cog is a short dementia assessment that combines three-word recall with clock drawing capability to assess patient cognition.

10. Mr. Ed, 46 years old, experienced a mild stroke and was hospitalized through the emergency room. The general work-up showed that his blood glucose was 300 mg/dl. He denied knowing that his sugar was so high. He did report frequency of urination and polyphagia. On further examination, the nurse practitioner asked about family history, and he identified that his mother and grandfather both had adult-onset diabetes. Although he is under 65 years old, the nurse practitioner teaches him which of the following? Select all that apply. 1. A urine sample that shows ketones in the urine, his age, and his body mass index (BMI) will tell if he is a type 1 or type 2 diabetic. 2. Having the mild stroke is connected to having diabetes. 3. Medications could be associated with diabetes. 4. Because he had a family history of type 2 diabetes, he should have been screened every year. 5. Screening would not have identified him as a risk.

10. Answer: 1, 2, 3 Page: 370 Feedback 1. The determination of whether or not the diabetes is type 1 or type 2 is based on ketonuria, the age of onset, and BMI. 2. The primary presentation may be due to complications of underlying and undiagnosed hyperglycemia in conditions such as stroke, myocardial infarction, and ischemia, intermittent claudication, impotence, peripheral neuropathy, proteinuria, retinopathy, slow wound healing, or fatigue. 3. Drugs associated with hyperglycemia include alcohol, beta-adrenergic agents, calcium channel blockers, corticosteroids, lithium salts, rifampin, asparaginase, diazoxide, diuretics, glycerol, niacin, phenytoin, and sympathomimetics. 4. Screening should begin for the general population at 45 years old and be repeated at 3-year intervals. More frequent screening is recommended for individuals with several risk factors. 5. Among adults 40 years and older, diabetes often is discovered as an incidental finding during the work-up for cardiovascular, renal, neurological, or infectious diseases. More frequent screening is recommended for individuals with several risk factors.

10. Which of the following factors affect drug absorption? Select all that apply. 1. Metabolic diseases 2. Esophageal erosion 3. Kidney diseases 4. Drug-food interactions 5. Huntington's disease

10. Answer: 1, 2, 4 Page: 471 Feedback 1. Metabolic diseases such as thyroid disease or diabetes can cause an increase or decrease in transit time and therefore can cause either increased or decreased drug absorption. 2. Esophageal erosion has been noted with caustic drugs such as alendronate potassium and tetracycline. These types of drugs increase the potential for esophageal damage from the dissolution of drugs. 3. Kidney diseases can affect drug elimination. 4. Drug-food interaction is likely to influence drug absorption 5. Huntington's disease is a fatal genetic disorder that causes the progressive breakdown of nerve cells in the brain. It deteriorates a person's physical and mental abilities during their prime working years and has no cure.

10. Mr. Fekjar is diagnosed with unstable angina. Which of the following is a contraindication for exercise therapy and prevents patients from joining an exercise program? Select all that apply. 1. Uncontrolled hypertension 2. Fibromyalgia 3. Unstable aortic aneurysm 4. Chondromalacia 5. Hypoxemia

10. Answer: 1, 3 Page: 20 Feedback 1. Uncontrolled hypertension is a medical contraindication for exercise therapy. 2. Fibromyalgia is a condition in which exercise therapy is useful for health promotion. 3. Unstable aortic aneurysm is a medical contraindication for exercise therapy. 4. Chondromalacia is an orthopedic condition in which exercise therapy is useful for health promotion. 5. Low oxygen saturation or hypoxemia is a below-normal level of oxygen in the blood, specifically in the arteries. Hypoxemia is a sign of a problem related to breathing or circulation, and may result in various symptoms, such as shortness of breath and dizziness during exercise. Though it is not a contraindication for exercise therapy, its symptoms could be a barrier.

10. AAA is mostly atherosclerotic in nature, but can also be caused by trauma, infection, and inflammation. Which of the following is a true statement regarding the tendency to develop AAA? Select all that apply. 1. There is no dominant ethnic group that develops AAA. 2. Caucasians have a higher risk of developing AAA. 3. Women have a higher risk of developing AAA. 4. There is a familial history associated with AAA development. 5. AAA is the 13th leading cause of death in the United States.

10. Answer: 1, 4, 5 Page: 216 Feedback 1. There is no dominant ethnic group that develops AAA, but there is a familial history associated with AAA development. 2. There is no dominant ethnic group that develops AAA. 3. Onset occurs around age 50 years for men and 60 years for women. Incidence steadily increases with age and peaks at age 80 years. AAA is five times more likely in men than in women. 4. There is no dominant ethnic group that develops AAA, but there is a familial history associated with AAA development. 5. According to studies, AAAs are the 13th leading cause of death in the United States. Mortality rates for ruptured aneurysms are 70% to 90% compared with 5% operative mortality for elective open surgical repair, and 2% to 3% for endovascular stent AAA exclusion.

10. To rule out age-related benign symptoms, motor functions should be inspected for body position and involuntary movements. Which of the following should be included in the assessment of motor function? Select all that apply. 1. Reflexes 2. Hypersensitivity 3. Muscle tone 4. Muscle strength 5. Sensory loss

10. Answer: 3, 4 Page: 330 Feedback 1. Reflex is action that is performed as a response to a stimulus and without conscious thought, and is not a motor function. 2. Hypersensitivity is an undesirable reaction produced by the normal immune system, including allergies and autoimmunity, and is not included in the motor function assessment. 3. Muscle tone is the internal state of muscle fiber tension within individual muscles and muscle groups and is a part of motor function. 4. Muscle strength is the ability of a muscle group to develop maximal contractile force against a resistance in a single contraction and is a part of motor function. 5. Sensory loss is included in the assessment of sensory function.

11. Which of the following may be included in the healthy lifestyle counseling during the initial Welcome to Medicare visit? Select all that apply. 1. Level of physical activity assessment. 2. Physical therapy consultation. 3. Referral to an herbalist. 4. Referral to a health coach. 5. Referral to an acupuncturist.

11. Answer: 1, 2, 4 Page: 20 Feedback 1. Current level of physical activity is assessed during the initial Welcome to Medicare visit. 2. Physical therapy consultation may be recommended for patients with disabilities or functional limitations. 3. Referral to an herbalist is not part of the initial Welcome to Medicare visit. 4. Referral to a health coach may be recommended to patients during the initial Welcome to Medicare visit to keep them engaged. 5. Referral to an acupuncturist is not part of the initial Welcome to Medicare visit.

11. The nurse practitioner is preparing home-going instructions for a 66-year-old man with newly diagnosed type 2 diabetes. The goals include which of the following? Select all that apply. 1. Maintenance of weight at present status 2. Glycemic control 3. Good nutritional status 4. More rest 5. Exercise

11. Answer: 2, 3, 5 Page: 370 Feedback 1. The goal of treatment for patients with type 2 diabetes is glycemic control through good nutritional status with weight management. 2. The goal of treatment for patients with type 2 diabetes is glycemic control. 3. The goal of treatment for patients with type 2 diabetes is glycemic control through good nutritional status. 4. The goal of treatment for patients with type 2 diabetes is glycemic control. They recommend individualized treatment plans based on the health and functional ability of separate patients into three categories. 5. The goal of treatment for patients with type 2 diabetes is exercise.

11. Larry is 69 years old and beginning to experience a cognitive decline. His son brings him to the practitioner asking for an evaluation. The nurse practitioner assesses for which of the following signs or symptoms? Select all that apply. 1. Cognitive decline in complex attention, executive function, learning, and memory. 2. Gradual onset and the course of illness and progression are typically slow. 3. Impaired ability to care for oneself. 4. Total or partial loss of the ability to recognize familiar people. 5. Improvement in symptoms in time.

11. Answers: 1, 2, 3, 4 Page: 443 Feedback 1. Cognitive decline must be in at least one of the following cognitive domains: complex attention, executive function, learning, and memory, language, perceptual motor or social cognition. 2. Dementia has a cognitive decline and must be in at least one of the following cognitive domains: complex attention, executive function, learning, and memory, language, perceptual motor or social cognition. 3. Symptoms vary from person to person, and cognitive deficits cause significant impairment in social and occupational functioning, impaired ability to care for oneself, and altered behavioral patterns. 4. Signs and symptoms progress from memory loss to impaired executive functioning, language deficits, coordination, and perception with total or partial loss of the ability to recognize familiar people or objects. 5. The etiology of dementia includes numerous systemic disorders, however, most cases of dementia are irreversible, because dementia is a progressive disease process unto itself.

12. Which of the following may be present in those suffering from delirium? Select all that apply. 1. Cognitive changes involving memory impairment, disorientation, and language disturbance. 2. Perceptual disturbances, including delusions and hallucinations. 3. Marked lethargy. 4. Disturbance of consciousness with impaired ability to focus. 5. Depression.

12. Answer: 1, 2, 4 Page: 486 Feedback 1. Patients suffering from delirium may develop cognitive changes involving memory impairment, disorientation, and language disturbance. 2. Patients suffering from delirium may develop perceptual disturbances including delusions and hallucinations. 3. Lethargy is a lack of energy and enthusiasm, and can develop with any conditions, real or perceived. 4. Patients suffering from delirium may develop disturbance of consciousness with impaired ability to focus, sustain, or shift attention. 5. Depression is a broad diagnosis in which low mood and/or loss of interest or pleasure in most activities are key features. Other symptoms include irritability, anxiety, social withdrawal, pain, fatigue, reduced or altered sleep and appetite, feelings of guilt and worthlessness, and reduced self-esteem and confidence. Delirium is a syndrome involving the sudden deterioration of mental functioning, which is triggered by acute illness of the body or brain, acute injury, or drug intoxication.

12. The nurse practitioner implements an approved plan of care proposed in the National Asthma Education and Prevention Program Expert Panel Report-3 (NAEPP-EPR-3) for an older adult patient with asthma. This includes which of the following? Select all that apply. 1. Completion of the assessment and monitoring, as this is not an ongoing process. 2. Patient, family, and professional education so as to create a partnership in asthma management. 3. Control of comorbidities. 4. Monitoring of appropriate use of medications. 5. Use of a standardized asthma care plan.

12. Answer: 2, 3, 4 Page: 156 Feedback 1. Asthma is a chronic health problem, just like hypertension and diabetes. Regular chronic maintenance visits are required for optimal management. 2. The NAEPP-EPR-3 (2007) has identified four components of asthma management, which includes educating the patient, family, and professionals for partnership in asthma management. 3. The NAEPP-EPR-3 (2007) has identified four components of asthma management, one of which is the control of comorbidities and environmental factors affecting asthma. 4. The NAEPP-EPR-3 (2007) has identified four components of asthma management, which includes pharmacotherapy. 5. Treatment in older adults may need to be individualized based on comorbidities.

12. Lymphedema is characterized by swelling of the extremity. Which of the following symptoms are also presented with lymphedema? Select all that apply. 1. Overall lethargy 2. Aching in the affected area 3. Restricted range of motion 4. Purulence 5. Cellulitis

12. Answer: 2, 3, 5 Page: 218 Feedback 1. Lethargy, or tiredness, is a state with a lack of energy and enthusiasm and not generally presented with lymphedema. Lethargy has been reported by people with multiple sclerosis, rheumatoid arthritis, hepatitis C, high blood pressure, and depression. 2. Lymphedema usually develops gradually over time and the swelling can be mild, moderate, or severe. Swelling, aching, discomfort, and fatigue in the affected limb may become present. 3. Restricted range of motion is a symptom presented with lymphedema due to the swelling in the affected limb. 4. Purulence is not presented with lymphedema. 5. Cellulitis is a sudden, noncontagious infection of the skin, characterized by redness, swelling, and heat, and is accompanied with pain and tenderness. Patients with lymphedema are particularly susceptible to cellulitis because the lymphatic system is damaged or overloaded and does not function adequately to fight infection.

12. Mrs. Kesler, 91 years old, is considered frail and yet she is active and lives alone with supervision from her daughter. Her frailty is based on which of the following? Select all that apply. 1. A 10-pound weight gain in the last month. 2. Self-reported exhaustion. 3. Strength based on grip. 4. Low walking speed. 5. Decreased physical activity.

12. Answer: 2, 4, 5 Page: 478 Feedback 1. Unintentional weight loss is one of the clinical criteria for diagnosing frailty as a clinical syndrome. 2. Self-reported exhaustion is one of the clinical criteria for diagnosing frailty as a clinical syndrome. 3. Weakness based on grip strength is one of the clinical criteria for diagnosing frailty as a clinical syndrome. 4. Slow walking speed is one of the clinical criteria for diagnosing frailty as a clinical syndrome. 5. Low physical activity is one of the clinical criteria for diagnosing frailty as a clinical syndrome.

13. Dyspnea derives from interactions of physiological, psychological, social, and environmental factors that may induce secondary physiological and behavioral responses. Which of the following are layman's terms for dyspnea? Select all that apply. 1. Shortness of breath 2. Breathlessness 3. Sobbing 4. Asthma 5. Hyperventilation

13. Answer: 1, 2 Page: 488 Feedback 1. Shortness of breath (SOB) is a layman's term associated with dyspnea. SOB can come on suddenly and affects ability to function. It is also a distressing symptom at end of life and affects quality of life. 2. Breathlessness is a layman's term associated with dyspnea. Breathlessness is described as the feeling of shortness of breath. It is a very common complaint associated with various medical conditions affecting the heart, lungs, and other systems. Breathlessness can be distressing and frightening for the patient. 3. Sobbing is crying noisily and making loud, convulsive gasps, but it rarely produces breathlessness. 4. Asthma is a respiratory condition marked by spasms in the bronchi of the lungs, causing difficulty in breathing or dyspnea. 5. Hyperventilation is rapid deep breathing, but in contrast to hyperpnea, it does not have a physiological function in the body. Dyspnea refers to the sensation of shortness of breath. With dyspnea, the breathing rate may be rapid, slow, or normal, and the depth of breathing may be shallow, deep, or normal.

13. A 65-year-old woman seeks relief from pain and swelling in her hands. She consults with a nurse practitioner for diagnosis and treatment. The nurse practitioner conducts an examination and interview about the patient's ADLs and how this pain and swelling inhibits her activity. Which of the following diagnostic tests will be ordered? Select all that apply. 1. Radiology of both hands. 2. Blood tests for conditions such as rheumatic factor. 3. Anti-cyclic citrullinated peptide (anti-CCP) antibodies. 4. Urinalysis. 5. Abdominal ultrasound.

13. Answer: 1, 2, 3 Page: 323, 324 Feedback 1. Radiographs of the hands and feet are needed to look for early signs of erosions, which are an important factor indicating the need to start an aggressive treatment approach aimed at halting further joint damage progression. 2. Blood tests such as rheumatoid factor (RF), erythrocyte sedimentation rate (ESR), and C-reactive protein are useful in the presence of diagnostic indicators from physical examination. 3. Testing for antibodies to cyclic citrullinated peptide (anti-CCP antibodies) is newer than RF testing and is associated with higher sensitivity and specificity for RA. 4. Urinalysis for uric acid crystals would indicate gout, but not RA. 5. Additional radiographic findings in RA include soft tissue swelling, symmetrical joint space narrowing, and joint subluxations.

13. Ms. Smith, 70 years old, is being treated for geriatric depression. What choices does the nurse practitioner have to order? Select all that apply. 1. Evaluate the present medication regimen. 2. Treat any comorbidity. 3. Maintain basic daily needs. 4. Order a selective serotonin reuptake inhibitor (SSRI). 5. Order Abilify.

13. Answers: 1, 2, 3, 4, 5 Page: 454 Feedback 1. The initial step in treating depression in older adults is to evaluate the present medication regimen and remove or change any medications that may contribute to symptoms. 2. Treat any systemic disorder that may have predisposed the patient to depression. 3. Ensure adequate nutrition, elimination, sleep, and physical activity. Optimum depression treatment for older adults should not be managed with medications alone and should include social interventions and possibly psychological modalities. 4. The most widely prescribed antidepressants with greatest evidence for achieving remission are the SSRIs. 5. Several second-generation antipsychotic agents are FDA-approved for augmentation to antidepressant therapy in treatment resistant depression, including aripiprizole (Abilify).

14. Mrs. Pope presents with twitching and burning sensations in her right leg that cause difficulty in walking. Initial suspicion is peripheral neuropathy. The proper diagnosis of peripheral neuropathy begins with a detailed history and physical examination. Which of the following information in her social history may be a cause of her symptoms? Select all that apply. 1. Substance abuse. 2. Use of supplements. 3. Use of shoes with good support. 4. Current occupation. 5. Going out to dinner.

14. Answer: 1, 2, 4 Page: 336, 337 Feedback 1. Substance abuse can be a potential cause of peripheral neuropathy. 2. Use of over-the-counter supplements can be a potential cause of peripheral neuropathy. 3. Using shoes with good support is not related to peripheral neuropathy. 4. Occupations with exposure to toxins can contribute to neuropathy. 5. Going out for dinner is not related to peripheral neuropathy.

15. The most common diagnostic test for AR is skin testing that involves scratching the surface of the skin with a single stylus for each allergen. Which of the following are possible if further testing is needed? Select all that apply. 1. Skin prick testing 2. CT scan 3. Blood test 4. MRI 5. Myelogram

15. Answer: 1, 2 Page: 148 Feedback 1. Skin prick testing is a next step, but intradermal testing may be needed if results are negative. 2. CT scan should not be used in the absence of recurrent infection. 3. Blood test is a not common diagnostic testing for AR. 4. MRI is a not common diagnostic testing for AR. 5. A myelogram is a diagnostic imaging test generally done by a radiologist. It uses a contrast dye and x-rays or CT scan to look for problems in the spinal canal.

15. John is 80 years old and is experiencing the following: weight gain, tachycardia, neck vein distention, and cognitive impairment. The nurse practitioner assesses for which of the following to support a diagnosis of heart failure? Select all that apply. 1. Laterally displaced point of maximum impulse of apical pulse. 2. Rales in bilateral lower lobes not due to atelectasis or ascites. 3. Functional impairment. 4. Increase in appetite and alertness. 5. Asymptomatic.

15. Answer: 1, 2, 3 Page: 171 Feedback 1. Signs that suggest heart failure include weight gain, tachycardia, S3 or S4 heart sound, and laterally displaced point of maximum impulse. 2. Signs that suggest heart failure include rales in bilateral lower lobes that are not due to atelectasis, positive hepatojugular reflux, and ascites. 3. Heart failure classification is based on risk, cardiac structural changes, symptom presentation, and functional impairment. 4. Progressive lower extremity and abdominal symptoms (pain, distention, nausea) related to hepatic enlargement are symptoms suggestive of right ventricular failure or systemic venous congestion. 5. Asymptomatic means without symptoms of disease. Geriatric patients with severe heart failure may be asymptomatic, but in this case, the patient does have symptoms.

15. Marie, 63 years old, is presenting with symptoms of postmenopausal bleeding and is diagnosed with endometrial cancer. Other than gender and age, which of the following can be contributing factors in the development of endometrial cancer? Select all that apply. 1. Diabetes 2. Use of tamoxifen 3. Use of hormonal contraception 4. Obesity 5. Lynch syndrome

15. Answer: 1, 2, 4, 5 Page: 292, 293 Feedback 1. Diabetes has been correlated with endometrial carcinoma. 2. Use of tamoxifen in postmenopausal women is a risk factor for endometrial carcinoma. 3. Use of hormonal contraception is a negative risk factor for endometrial carcinoma. 4. Obesity is a contributing factor in the development of endometrial cancer. 5. Women who have Lynch syndrome, which is an autosomal dominant genetic condition causing hereditary nonpolyposis colorectal cancer, are at high risk for endometrial malignancies.

15. According to studies, RLS is a condition for which there is no cure. Which of the following are recommended for the management of RLS symptoms? Select all that apply. 1. Glass of wine every night to fall asleep 2. Massage 3. Exercise 4. Group therapy 5. New bed

15. Answer: 2, 3, 4 Page: 345 Feedback 1. Drinking a glass of wine every night to fall asleep is not recommended for the management of RLS symptoms. 2. Massage is recommended for RLS symptoms to help relax the muscles. 3. Daily exercise and stretching has been found to relieve RLS symptoms. 4. Group therapy may offer support and an opportunity to learn new ways of managing RLS. 5. Purchasing a new bed is not known to reduce RLS symptoms.

15. Bereavement is the process of experiencing the death of a loved one and adjusting to a world without the deceased. Bereavement can be expressed in many ways. Which of the following relates to bereavement and grief? Select all that apply. 1. Normal grieving is only presented through emotional responses. 2. Medical providers should instruct those involved regarding normal grief reactions and bereavement resources in the community. 3. Memory loss and impaired concentration are common grief reactions. 4. Risk for self-harm is only present when loved ones left behind have a prior diagnosis of major depressive disorder. 5. Bereavement care should include questioning of spiritual and religious beliefs.

15. Answer: 2, 3, 5 Page: 496 Feedback 1. Normal grieving includes physical, emotional, cognitive, and behavioral responses. 2. Family education should include instruction on normal grief reactions, bereavement resources in the community, and awareness of signs of complicated grief and depression. 3. Common grief reactions include: somatic symptoms, sleep and appetite disturbances, memory loss and impaired concentration, social withdrawal and disinterest in prior activities, a sense of the presence of the deceased person, auditory or visual hallucinations, questioning of spiritual and religious beliefs, and emotional reactions that may include relief, numbness, helplessness, self-reproach, sadness, guilt, or despair. 4. Complicated grief typically includes intense shock, anger and denial, feelings of numbness or hopelessness, a blunted emotional expression and anxiety, panic, chronic depression, and a risk for self-harm, such as suicide or substance abuse. The risk for self-harm is possible with or without a prior mental health disorder. 5. Grief is our natural reaction to a loss. The ones left behind feel a deep and aching pain when a loved one is no longer around. Often times, loved ones start doubting their religious beliefs, especially after watching someone suffer from a prolonged or painful illness or sudden death.

2. Lucy is a 56-year-old woman experiencing itching, discomfort, and bleeding after intercourse. Which of the following conditions may be developing in the patient? 1. Atrophic vaginitis 2. Vaginal cancer 3. Fibroadenoma 4. Cystitis

2. Answer: 1 Page: 282 Feedback 1. Atrophic vaginitis presents symptoms of itching, discomfort, and bleeding after intercourse. 2. Vaginal cancer presents symptoms of a watery vaginal discharge, a lump or mass in the vagina, painful urination, frequent urination, constipation, and pelvic pain. 3. Fibroadenoma is a solid, noncancerous breast lump. 4. Cystitis presents with symptoms of change in urination: dysuria, frequency, urgency, and change in urine character. It also can include suprapubic tenderness and mental status changes.

2. A clinician is performing sensory assessment on a patient presenting signs and symptoms of neurological disease. During the assessment, the clinician notices that the patient has a decreased sense of both pain and temperature. Which of the following may be affected? 1. Spinothalamic tract 2. Cranial nerve VII 3. Posterior column 4. Posterior horn

2. Answer: 1 Page: 330 Feedback 1. Abnormal pain and temperature sensory function may indicate a lesion on the spinothalamic tract. 2. Cranial nerve VII controls facial motor functions. 3. When position and vibration is involved, the posterior column is the affected area. 4. The posterior horn controls the sensation of touch.

2. The nurse practitioner is speaking to a group of student nurses about the causes of death of older adults. Which of the following statements is true regarding mortality and morbidity of people older than 65 years? 1. Seventy percent of the cohort older than 65 years dies of chronic conditions. 2. Conditions that contribute to significant chronic illness include fractures. 3. The number one cause of death of both older men and women is heart disease. 4. Dyslipidemia is the leading cause of mortality of people over 65 years of age.

2. Answer: 1 Page: 474 Feedback 1. Seven of every ten deaths in the United States are caused by chronic conditions. Heart disease, cancer, COPD, and stroke are the leading causes of death, with heart disease as the number one cause of death among both men and women. 2. Heart disease, cancer, COPD, and stroke are the leading causes of death, along with heart disease, diabetes, and Alzheimer's disease. 3. Heart disease is the number one cause of death among both men and women. 4. The most common chronic disease in the population age 65 years and older is dyslipidemia (41%).

2. Part of the abdominal examination the nurse practitioner will conduct is the assessment of skin color. As the nurse practitioner assesses the abdominal skin, the findings include which of these signs that are concerning? 1. Generalized pinkness. 2. Bluish discoloration on the flanks. 3. Yellow tint around the lower left quadrant. 4. Blue in the upper left quadrant.

2. Answer: 2 Page: 226 Feedback 1. Pinkness is normal for a Caucasian patient. 2. If the bluish discoloration is on the flanks, this is known as Grey Turner's sign and is often indicative of retroperitoneal bleeding, as with pancreatitis. 3. Jaundice in the area of the umbilicus is known as Ransohoff's sign and is a result of a ruptured common bile duct. 4. If a bluish discoloration is detected around the umbilicus, this is known as Cullen's sign, which is often found in patients with bleeding in the peritoneum.

3. In the initial screening for a diagnosis of AAA, which of the following is the best screening test? 1. CT scan 2. Complete blood count (CBC) 3. Ultrasound in the abdominal area 4. Angiography

3. Answer: 3 Page: 216 Feedback 1. CT screening is indicated when surgery is planned. 2. CBC may be a secondary screening when surgery is planned. 3. Ultrasound in the abdominal area is the best initial screening test for AAA. 4. Angiography screening is indicated when surgery is planned.

3. Mrs. M.W. asks the nurse practitioner what she is listening to in her belly and what it means. The nurse practitioner tells her that she is listening to determine: 1. That her bowels are moving properly. 2. If the blood vessels are not circulating blood. 3. Circulation and bowel function. 4. A diagnosis of her symptoms.

3. Answer: 3 Page: 226 Feedback 1. Studies have refuted the usefulness of auscultation when differentiating between patients with normal bowel sounds versus those with pathological bowel sounds. 2. Other sites to auscultate for bruits during an abdominal examination include the iliac arteries and the femoral arteries. 3. To auscultate for bruits during an abdominal examination, include the iliac arteries and the femoral arteries. When auscultating bowel sounds, take the time to listen for the presence of bowel sounds. 4. Nurse practitioners should not rely on auscultation alone for diagnostic purposes.

3. Mrs. Khan is 89 years old and is reported to have bouts of agitation. The nurse practitioner is counseling her family that these periods of agitation can be triggered by which of the following? 1. Silence and soft music. 2. Dim lights. 3. Approaches from other people. 4. A feeling of appetite satiety.

3. Answer: 3 Page: 429 Feedback 1. Environmental triggers, such as noise and visual cues from television and physical surroundings, can also lead to agitation. 2. Environmental triggers, such as light, can trigger agitation. 3. Psychosocial triggers, such as the approach taken by staff and interaction with other residents, can contribute to agitation. 4. Chronic bedrest, pain, and hunger can precipitate episodes of agitation.

4. A 58-year-old man is being seen by his nurse practitioner for a cardiac work-up. His cholesterol is 320 mg/d. The nurse practitioner includes teaching about lowering his cholesterol by discussing blood levels and dietary choices. The nurse practitioner says which of the following? 1. Your cholesterol level is above normal for your age and we need to address how to lower it. 2. We will discuss food choices that will raise your LDL levels. 3. We will discuss food choices that will lower your HDL levels. 4. The effects of high cholesterol levels will not be a factor in cardiac disease after you turn 60 years old.

4. Answer: 1 Page: 181 Feedback 1. The normal cholesterol level is less than 200 mg/d. 2. Efforts are often indicated to lower LDL levels. 3. Efforts are often indicated to raise HDL levels for cardiac protection. 4. The risk of the effects of an elevated cholesterol level persists from middle age to extreme old age.

4. The U.S. health-care system functions by managing and paying for illness by focusing on one problem at a time. The nurse practitioner analyzes this system and concludes it is ineffective because: 1. As the number of comorbidities increases, so do the risks. 2. Unintended outcomes of the one disease model include the collaboration of medical advice. 3. There is no knowledge gap regarding people with multiple chronic conditions (MCCs). 4. One in ten Medicare recipients has more than one chronic illness.

4. Answer: 1 Page: 475 Feedback 1. As the number of chronic conditions in an individual increase, there is a parallel risk for poor outcomes often related to the unintended consequences of a single disease approach. 2. Unintended consequences of a single disease approach include: poor functional status, unnecessary hospitalization, adverse drug events, conflicting medical advice from multiple providers and even death. 3. To explore the public health problem of MCCs, the U.S. Department of Health and Human Services created the Multiple Chronic Conditions Interagency Workgroup. One of the main goals of the framework is to generate research to bridge knowledge gaps about individuals with MCCs. 4. One in four adults has more than one chronic condition or MCC, and among the more than 20 million Medicare beneficiaries, 37% have five or more chronic conditions.

4. Mrs. Prentiss is diagnosed with symptomatic AAA. The initial treatment is aggressive blood pressure control. At which of the following readings should mean arterial pressure be maintained? 1. Between 100 and 120 mm Hg. 2. Between 90 and 120 mm Hg. 3. Between 60 and 70 mm Hg. 4. Between 80 and 100 mm Hg.

4. Answer: 3 Page: 217 Feedback 1. A reading of 100 to 120 mm Hg is too high. 2. A reading of 90 to 120 mm Hg is too high. 3. Good arterial pressure for those with AAA should fall between 60 and 70 mm Hg. 4. A reading of 80 to 100 mm Hg is too high.

5. Which medical specialist is the best referral for patients with symptomatic AAA? 1. Vascular surgeon 2. Neurosurgeon 3. Cardiologist 4. Internist

5. Answer: 1 Page: 217 Feedback 1. A vascular surgeon is a specialist who is highly trained to treat diseases of the vascular system. 2. A neurosurgeon is a physician who specializes in the diagnosis and surgical treatment of disorders of the central and peripheral nervous system. 3. A cardiologist is a doctor who specializes in the study or treatment of heart diseases and heart abnormalities. 4. Internists are specialists who apply scientific knowledge and clinical expertise to the diagnosis, treatment, and compassionate care of adults across the spectrum, from health to complex illness.

5. The nurse practitioner is examining an 85-year-old man with reports of abdominal pain, weakness, and loss of appetite. Which is the most likely condition to be tested for and ruled out? 1. Neoplasms and carcinomas 2. Partial seizure 3. Sarcopenia 4. Hirschsprung's disease

5. Answer: 1 Page: 4 Feedback 1. Certain diseases, such as neoplasms and carcinomas, are more common in the elderly, and an understanding of the epidemiology is critical in the interpretation. 2. Partial seizure is more common in early old age. 3. Sarcopenia is more common in early old age. 4. Hirschsprung's disease is most common in infancy.

5. Which of the following statements best describes "cough"? 1. It is a sign of a life-threatening condition. 2. It is often isolated to the respiratory system. 3. It is often isolated to the immune system. 4. It is often associated with acid reflux.

5. Answer: 2 Page: 43 Feedback 1. Cough is often associated with conditions that are not life threatening. 2. Cough is often isolated to the respiratory system. 3. Cough may be a secondary symptom associated with the immune system. 4. Cough is unlikely to be associated with acid reflux.

5. Which of the following is a neurological disorder characterized by leg cramping, pulling, and tugging sensations, and may have a genetic component? 1. Radiculopathy 2. Nerve compression 3. Restless legs syndrome (RLS) 4. Sleep impairment

5. Answer: 3 Page: 344 Feedback 1. Radiculopathy is a possible differential diagnosis for RLS. 2. Nerve compression is a possible differential diagnosis for RLS. 3. RLS is a neurological disorder characterized by cramping, gnawing, pulling, tugging, and other sensations, and may have a genetic component. 4. Sleep impairment may be a secondary outcome of RLS.

5. Mr. Jameson, 75 years old, comes to the clinic with complaints of severe sharp epigastric pain that radiates to his back. He has been nauseous and has vomited three times today. The nurse practitioner suspects which of the following? 1. Appendicitis 2. Gastrointestinal influenza 3. Acute pancreatitis 4. Insulin shock

5. Answer: 3 Page: 362 Feedback 1. Pancreatitis is manifested by mild-to-severe sharp epigastric pain, which possibly radiates to the back, chest, or flanks. 2. Other possible causes of acute pancreatitis are many and include viral infections (mumps, coxsackie B, mycoplasma pneumonia, and campylobacter). 3. The signal symptoms of acute pancreatitis include mild-to-severe sharp epigastric pain, which possibly radiates to the back, chest, or flanks. Nausea and vomiting accompany the pain in up to 90% of patients. 4. If not due to gallstones or alcohol use, two less common but useful to investigate causes in the older adult are drug-induced pancreatitis and hypertriglyceridemia-induced pancreatitis, typically with serum triglycerides over 1,000 mg/dL.

5. Ms. Rose, 88 years old, comes to the nurse practitioner with a complaint about a growth on her hand. She wants to have a biopsy done. The nurse practitioner asks the following question: 1. Have you injured your hand recently? 2. Are you using a different detergent? 3. Has this growth changed, bled, or is it painful? 4. Has this growth made it difficult to put on your rings?

5. Answer: 3 Page: 97 Feedback 1. An injury would not stimulate growth. 2. A reaction to a detergent would more likely be a rash. 3. Lesions that warrant biopsy are those that have changed, bleed, or are painful. 4. The ability to put on her ring is not the problem.

5. The nurse practitioner is conducting an admitting assessment of a 69-year-old man. He has diminished ability to ambulate and has difficulty with activities of daily living (ADLs). In the interview, the nurse practitioner asks him if he drinks alcohol. He says, "Yes." The practitioner follows up with a psycho-social evaluation because: 1. Physiological dynamics need to be ruled out as a possible contribution. 2. His drinking may be a function of the environment where he lives. 3. Criteria from the DSM-V is the same for people of any age. 4. His condition could be related to a long-term usage of alcohol or a new occurrence.

5. Answer: 4 Page: 431 Feedback 1. Neurotransmitter effects from alcohol and alterations in brain anatomy in patients with alcohol use disorders serve to reinforce the biological connection. 2. Several genetic markers have been studied, and a clear case for biological heritability has been established as one factor. 3. Older adults with alcohol problems often do not meet the American Psychiatric Association's Diagnostic and Statistical Manual of Mental Disorders, Fifth Edition (DSM-5) criteria for alcohol use disorder, but are impaired nonetheless. 4. It is unclear whether risky drinking in older people represents a pattern of continued use, a return to using after a period of abstinence, new onset of use, or a combination of all these patterns.

6. Mrs. Cameron is a 72-year-old woman who comes to see her nurse practitioner for pain in her right thigh. The nurse practitioner assesses that Mrs. Smith has bruises on her thigh and muscle weakness distally, near the knee. The nurse practitioner suspects which of the following? 1. Musculoskeletal lesions 2. Tendon rupture 3. Gout 4. Arthritis

6. Answer: 2 Page: 306 Feedback 1. Patients with presentation of soft tissue syndromes should be asked about the origin of the presentation of bruising. The pain may be related to an event. 2. Ecchymosis in the area of a joint may be indicative of tendon rupture. 3. Gout is polyarticular in older adults. 4. Arthritis is an ongoing synovial membrane attack that results in synovial proliferation; pannus formation; and destruction of bone, cartilage, and ligaments, leading to joint damage and deformity.

6. Which of the following statements is true for untreated symptomatic cystitis? 1. Untreated symptomatic cystitis can lead to pneumonia. 2. Untreated symptomatic cystitis can resolve on its own. 3. Untreated symptomatic cystitis can lead to pyelonephritis, sepsis, shock, and death. 4. Untreated symptomatic cystitis can lead to prostatic cancer, vaginitis, and sepsis.

6. Answer: 3 Page: 291 Feedback 1. Pneumonia is a complication in the respiratory system. 2. Treatment is indicated to resolve cystitis. 3. Untreated symptomatic cystitis can lead to pyelonephritis, sepsis, shock, and death. 4. Untreated symptomatic cystitis does not lead to prostatic cancer or vaginitis, but it can lead to sepsis.

6. Jane Smith is a 70-year-old Caucasian woman who comes to see her nurse practitioner for pain in her abdomen. The nurse practitioner is aware that Mrs. Smith is obese and has diabetes. Which of the following is a definitive diagnostic test for cholecystitis? 1. A flat plate of the abdomen. 2. Complete blood count (CBC). 3. Lipase, amylase, and liver function tests. 4. Real-time ultrasonography of the gallbladder and biliary tree.

6. Answer: 4 Page: 232 Feedback 1. A flat plate of the abdomen will not provide the information that a real-time ultrasound will. 2. Mild leukocytosis with increased band formation is the most common abnormality seen in laboratory studies; however, white blood cell counts may not be elevated in the older adult. 3. Elevation in lipase, amylase, and elevated liver function tests would prompt further evaluation for gallstone pancreatitis; however, laboratory values may be normal. 4. Real-time ultrasonography of the gallbladder and biliary tree is the diagnostic procedure of choice for both acute and chronic cholecystitis, showing gallstones, thickening of the gallbladder wall, and (if the common bile duct is obstructed) dilation of the biliary tract.

7. The son of a 72-year-old man brings his father into the practitioner to plan for treatment of the patient's alcoholism. The nurse practitioner has several options available. Which is the first choice? 1. Treat for anxiety. 2. Begin with higher doses of anti-anxiety medications. 3. The last choice for medication is serotonin uptake inhibitors. 4. Buspirone and gabapentin are used as first-line choices.

7. Answer: 1 Page: 435 Feedback 1. Treatment for anxiety should reduce symptoms and improve functioning. Simply listening, being compassionate, and showing respect are important to improving outcomes. 2. Start low and go slow with medication dosing to avoid risks from drug interactions. Older adults are more likely to take many medications and may have side effects from aging changes in absorption, metabolism, distribution, and excretion of medication. 3. First-line treatment includes the SSRIs because they have the least risk of drug interactions, side effects, or worsening existing medical conditions. 4. Buspirone and gabapentin are also used as secondary agents when first-line therapy fails and anxiolytic therapy is warranted.

7. In conducting a CGA, the nurse practitioner looks for any impairments in communication with the patient. The assessment includes: 1. Interviewing prior to the physical examination. 2. Asking questions of the accompanying family member. 3. Reading the history forms provided by the patient after the examination. 4. Observing physical impairments.

7. Answer: 1 Page:26 Feedback 1. Interviewing begins before the physical examination and continues throughout the examination. 2. Permission from the patient must be obtained before discussing anything with the family member. 3. When forms about the history of the patient are brought in, the nurse practitioner should review them before the examination. 4. Observation alone will not provide adequate information about communication impairments.

7. Kefka Jones, 60 years old, has diabetes and is now having urinary tract symptoms. The nurse practitioner orders a glomerular filtration rate (GFR) from a urine sample. Which of the following would indicate a stage 1 chronic kidney disease (CKD)? 1. GFR greater than or equal to 120mL/min. 2. GFR greater than or equal to 90 mL/min with other symptoms. 3. GFR of 45 to 59 mL/min. 4. GFR less than 15 mL/min.

7. Answer: 2 Page: 236 Feedback 1. GFR of 120 mL/min is not a criterion for stages of CKD. 2. Stage 1 is when GFR is greater than or equal to 90 mL/min with other evidence of CKD damage. 3. Stage 3a is GFR 45 to 59 mL/min. 4. Stage 5 is GFR less than 15 mL/min or on dialysis.

7. The nurse practitioner and a student are examining the apical pulse of an 82-year-old man. The student perceives that there is an S4 sound. The nurse practitioner interprets that this is likely: 1. An early ventricular filling. 2. Heart blockage. 3. Mitral regurgitation. 4. Normal, due to the man's age.

7. Answer: 4 Page: 153 Feedback 1. The early diastolic filling is reduced in healthy older adult persons, resulting in an end-diastolic volume maintained by an increase in atrial contribution to left ventricular filling. 2. A change in the loudness of S1, accompanied by a slow heart rate, may indicate heart blockage. 3. An S3 in an individual more than 50 years old is usually pathological and reflects an increased filling rate, indicating heart failure or mitral regurgitation. 4. This condition may cause an S4, which, in the absence of other findings, is considered normal in older adult persons.

8. Prostate cancer is the most common and leading cause of cancer death in men. Which of the following statements is true about prostate cancer? 1. There is usually a sudden onset of pain in the early stages. 2. Asian men are at highest risk for developing prostate cancer. 3. Development of prostate cancer has been linked with prior vasectomy. 4. There are usually no symptoms with early disease.

8. Answer: 4 Page: 299 Feedback 1. There are usually no symptoms with early disease. 2. African Americans have the highest incidence of prostate cancer in the world, with Asian and Hispanic men at lower risk than white men. 3. No link has been determined with prior vasectomy. 4. Once symptoms occur, there is usually progression of the disease.

8. Elisabeth, 67 years old, comes to the practitioner with her daughter, who is her caregiver. The nurse practitioner notices that Elizabeth is very busy and always moving and placing things in order within the waiting room. Along with this increase in energy, which of the following suggest bipolar with mania or hypomania? Select all that apply. 1. Ability to focus on a task 2. Flight of ideas 3. Rapid speech 4. Impulsivity 5. Thoughtfulness

8. Answers: 2, 3, 4 Page: 437 Feedback 1. According to the DSM-5, the individual must also experience increased energy while having distractibility. 2. According to the DSM-5, the individual must also experience increased energy while having these flights of ideas. 3. According to the DSM-5, the individual must also experience increased energy while having rapid speech (can be garbled). 4. According to the DSM-5, the individual must also experience increased energy while having thoughtlessness (impulsivity). 5. According to the DSM-5, the individual must also experience increased energy while having thoughtlessness.

9. John Brown, 60 years old, asks the nurse practitioner if a lack of functioning insulin is the only cause of diabetes. Which of the following is the nurse practitioner's reply? Select all that apply. 1. More recently, other hormones have been found to be part of diabetes. 2. Besides insulin, glucagon, amylin, and incretin are involved. 3. Finding other factors of diabetes has resulted in new treatments. 4. The pancreas is not the only organ involved in diabetes. 5. Genetic factors lead to type 1diabetes only.

9. Answer: 1, 2, 3, 4, 5 Page: 369 Feedback 1. Early understanding of diabetes has been expanded by the discovery of other hormones involved in the regulation of glucose. 2. Hormones involved in the regulation of glucose include insulin, glucagon, amylin, and incretin. 3. These discoveries have resulted in new classes of medications to treat diabetes. 4. These hormones include pancreatic hormones: insulin and amylin secreted by the β cells and glucagon from the α cells, as well as the gut incretin hormones: gastric inhibitory polypeptide (GIP) and glucagon-like peptide-1 (GLP-1). 5. Genetic predisposition for developing diabetes is more evident in type 2 diabetes.

9. Patient education is very important for individuals who have AAA. The patient and their family should be taught the importance of follow-up and the management of which of the following? Select all that apply. 1. Hypertension 2. Hypercholesterolemia 3. Smoking 4. Chondromalacia 5. Pain

9. Answer: 1, 2, 3, 5 Page: 220, 221 Feedback 1. Education about hypertension management is important for patients with AAA. Antihypertensive agents are used to reduce tension on the vessel wall in patients with AAAs who have elevated blood pressure. 2. Patients with AAA need to learn about control of hypercholesterolemia (also called high cholesterol). Hypercholesterolemia is characterized by high serum cholesterol levels and premature coronary atherosclerosis. Hypercholesterolemia is one of the factors promoting the arteriosclerotic process and is a major cause of aortic aneurysm. 3. Smoking cessation should be considered by patients with AAA. Smoking appears to increase the risk of aortic aneurysms. Smoking can be damaging to the aorta and weaken the aorta's walls. 4. Chondromalacia is a pain in the knee and has no relationship to AAA. 5. Pain is the most common symptom of an AAA. Pain associated with an AAA may be in the abdomen, chest, lower back, or groin area. The pain may be severe or dull. Sudden, severe pain in the back or abdomen may mean the aneurysm is about to rupture.

9. Tom Brown is 62 years old and plays golf twice a week. He experiences pain in his shoulder and goes to the urgent care center to be seen by his nurse practitioner. The nurse practitioner asks which of the following questions? Select all that apply. 1. Does the shoulder hurt when you are not playing golf? 2. Is there any swelling, redness, or pain? 3. Is there any family history of arthritis? 4. Is there a history of taking ibuprofen? 5. Does your pain affect your golf game?

9. Answer: 1, 2, 5 Page: 308 Feedback 1. Determining the extent of pain will help with the diagnosis. 2. Identify all presenting and associated symptoms such as swelling, erythema, pain, reported decrease in ROM, limitations on ADLs, and alterations in sensations of hot or cold. 3. Question patient about family history of soft tissue conditions, aspiration of bursa, recent fever, fluoroquinolone use, and chronic medical conditions. 4. Question patient about prior history of fluoroquinolone use. 5. Identify all presenting and associated symptoms, such as swelling, erythema, pain, reported decrease in ROM, and limitation on ADLs.

9. Steve came in for an office visit presenting complaints of difficulty speaking, gait problems, and dizziness. Which of the following should be noted as additional signs and symptoms in the focused interview? Select all that apply. 1. Paresthesias 2. Weight gain 3. Pertinent family history 4. Loss of consciousness 5. Use of tobacco

9. Answer: 1, 3, 4 Page: 328 Feedback 1. Paresthesias is an abnormal sensation, typically tingling or pricking ("pins and needles"), caused chiefly by pressure on or damage to peripheral nerves, and may be associated with neurological disorder or disease. 2. Weight gain is usually not associated with neurological disorder or disease. 3. Pertinent family history of any acute neurological disease will reveal the possibility of neurological disease. 4. Loss of consciousness may be associated with neurological disorder or disease. 5. Tobacco use does not relate to neurological disorder or disease.

2. The nurse practitioner knows that risk factors for coronary artery disease (CAD) include atherosclerosis. The nurse practitioner includes teaching on these factors that can be remediated to help prevent CAD: 1. Low blood pressure. 2. Age. 3. Advanced age and male gender. 4. Cholesterol levels and smoking.

2. Answer: 4 Page: 152 Feedback 1. One major remediable risk factor for CAD is high blood pressure. 2. Aging cannot be remedied, though it is a risk for CAD. 3. These two most important risk factors for atherosclerosis cannot be remedied. 4. Along with high blood pressure, cholesterol levels and smoking can be remedied.

3. Mr. Thomas comes in for a health examination with his nurse practitioner. When the nurse practitioner measures his blood pressure, the systolic is 150 mm and the diastolic is 80 mm. When Mr. Thomas asks if that is a good pressure for him, the nurse practitioner responds: 1. It is normal for your age. 2. Isolated hypertension is usually identified by a systolic blood pressure (SBP) higher than 160 mm Hg. 3. When evaluating blood pressure, a higher diastolic blood pressure (DBP) is risky. 4. There is a risk of a cardiovascular event for you with an SBP of 150 mm Hg.

3. Answer: 2 Page: 175 Feedback 1. Normal blood pressure is less than 120/less than 80.The SBP rises with age. 2. Isolated systolic hypertension is defined as an SBP greater than 160 mm Hg and a DBP of less than 90 mm Hg. 3. Cardiovascular disease in the geriatric age group is of greater possibility where there are systolic elevations rather than diastolic elevations. 4. The risk of experiencing a cardiovascular event is two to three times higher in those with significant hypertension (SBP over 160 mm Hg).

5. A patient is started on a low dose of a drug for safety but is never advanced to a therapeutic dose while another drug is added. Which of the following does this describe? 1. Preferences of care. 2. Risk-benefit ratio of drug. 3. Narrow therapeutic index of drug. 4. Clinical inertia.

5. Answer: 4 Page: 472 Feedback 1. Preferences of care are factors to weigh before prescribing. 2. Risk-benefit ratio of a drug is a factor to weigh before prescribing. 3. Narrow therapeutic index of a drug is a factor to weigh before prescribing. 4. Clinical inertia can occur when a patient is started on a low dose of a drug for safety but is never advanced to a therapeutic dose or is maintained on a drug that has no therapeutic benefit while another drug is added. Both situations contribute to polypharmacy and the potential for ADRs.

10. When looking for disorders, which of the following parts of the body should be inspected in the examination of the head, face, and neck? Select all that apply. 1. Sinuses 2. Neck 3. Lungs 4. Skin 5. Cervical spine

10. Answer: 1, 2 Page: 128 Feedback 1. The sinuses should be inspected and palpated, noting any gross tenderness or inflammation. 2. Inspection should include looking for symmetry, masses, scars, tracheal position, deviation, and other things. 3. Inspection of the lungs is only possible by performing diagnostic testing and not possible by sight and touch. 4. Inspection of the skin, although important, is not a focus in the examination for head, face, and neck disorders. 5. The cervical spine, or neck, begins at the base of the skull and through a series of seven vertebral segments connects to the thoracic, or chest, region of the spine. It houses the spinal cord, which sends messages from the brain to control all aspects of the body. This is not evaluated in examination of the head, face, and neck.

10. While examining John, who is 80 years old, the nurse practitioner observes that as he rises from the chair he pauses and holds on to the rail. Asking John if this happens often, he replies, "Yes, I get lightheaded and feel like I will fall." The nurse practitioner decides to conduct testing for orthostatic hypotension. This test includes which of the following? Select all that apply. 1. Having the patient lie down for 5 minutes, then measuring blood pressure and pulse. 2. Having the patient stand and wait a minute, then measuring blood pressure and pulse. 3. Having the patient jump in place for 2 minutes and then lie down. 4. Having the patient breathe deeply five times and then stand up. 5. Having the patient get up from his chair without using his arms, walk two to three steps, then back up to his chair and sit down.

10. Answer: 1, 2 Page:27 Feedback 1. Obtaining baseline blood pressures is important to assess any changes. 2. Measuring the blood pressure after John gets up will show a drop if he experiences hypostatic hypotension. 3. Jumping in place is not a normal activity and is risky for older people. 4. Breathing deeply does not affect blood pressure significantly. 5. Mobility can be screened with the Timed Get Up and Go Test. This test involves observing for unsteadiness as the patient gets up from a chair without using the arms, walks 10 feet, turns around, walks back, and resumes a seated position.

10. Mrs. Lee is experiencing moderate pain in her right leg and having difficulty sitting down on and standing up from a chair. She reports no other symptoms. Which of the following should be included in the assessment? Select all that apply. 1. Thorough medication history 2. Precipitating factors 3. Character of symptom 4. Family history 5. Social history

10. Answer: 1, 2, 3 Page: 34 Feedback 1. Use of medications should be considered in formulating diagnostic possibilities. 2. Precipitating factors should be included in the assessment process as main descriptors to guide the diagnosis. 3. Character of symptoms are necessary descriptors to guide the diagnosis. 4. Family history, although helpful, is a secondary factor in the assessment process. 5. Social history would not provide helpful information for this physical condition.

10. While counseling a patient about physical activity, the nurse practitioner considers which of the following? Select all that apply. 1. A physical activity prescription that is individualized to the patient. 2. Active hobbies that the patient enjoys. 3. Alternative activities such as yoga or tai chi. 4. Armchair activities for the frail older adult. 5. Patient's ability to pay for involvement in activities.

10. Answer: 1, 2, 3, 4 Page: 7 10. Answer: 1, 2, 3, 4 Page: 7 Feedback 1. Each person is unique and may not respond the same to exercise. 2. Hobbies are enjoyable and give pleasure, while at the same time increasing activity. 3. Stretching routines are done slowly and to the level of endurance. 4. When the frail older person cannot ambulate, they can move other parts of their body such as core, arms, head, and neck. They may also do non-weight-bearing movements of legs and feet. 5. Movement, activity, and exercise can be done anywhere, anytime the individual desires. There is no need to join a group.

3. The nurse practitioner knows that conducting a comprehensive geriatric assessment (CGA) is more beneficial for which of the following groups? 1. People between 60 and 70 years old. 2. People over 70 years old. 3. People classified as vulnerable older adults. 4. People over 55 years old.

3. Answer: 3 Page: 26 Feedback 1. People between 60 and 70 years old are likely to be healthy. 2. People over 70 years old have a variety of levels of health. 3. Vulnerable older adults and frail older adults will benefit most from a CGA because of lower physiological reserves and higher risk of iatrogenic complications. 4. People over 55 years old may not be considered the appropriate age for a CGA.

3. Which of the following conditions is signaled by bleeding from the nose? 1. Rhinitis 2. Telangiectasia 3. Retinopathy 4. Epistaxis

3. Answer: 4 Page: 130 Feedback 1. Rhinitis is inflammation of the nose. Symptoms of rhinitis include runny nose (rhinorrhea), nasal itching, nasal congestion, and sneezing. 2. Telangiectasia, also known as spider veins, are small dilated blood vessels near the surface of the skin or mucous membranes. These dilated blood vessels can develop anywhere on the body but are commonly seen on the face around the nose, cheeks, and chin. 3. Retinopathy is a disease of the retina that results in impairment or loss of vision. 4. Epistaxis is a condition signaled by bleeding from the nose and is the result of a spontaneous rupture of a blood vessel in the anterior septum. Causes include, but are not limited to, trauma, irritation or inflammation of the nasal mucosa, a septal defect, or paranasal tumors.

6. Oral cancer is a malignant tumor presented by a nonhealing sore in the mouth or lip that causes difficulty chewing or swallowing. Which of the following statements applies to an oral cancer diagnosis? 1. It cannot be prevented. 2. It involves unexplained lymph node swelling in the head and neck area. 3. It is three times more likely in women than men. 4. It involves unexplained lymph node swelling in the axillary area.

6. Answer: 2 Page: 141 Feedback 1. The most important way to control oral cancer is the combination of early diagnosis and timely and appropriate treatment. Preventions include lifestyle changes, periodic checkups, and protection from sun exposure. 2. Unexplained lymph node swelling in the head and neck area is one of the symptoms of oral cancer. Additionally, a nonhealing sore in the mouth or lip and difficulty chewing or swallowing are also symptoms. 3. Men are almost three times more likely to develop oral cancer than women. 4. Unexplained lymph node swelling in the head and neck area is a symptom, but lymph node swelling in the axillary area is not.

7. A 67-year-old female patient comes in for an office visit presenting symptoms of nasal congestion, itching, and sneezing and is diagnosed with rhinitis. Which of the following is true about the types of rhinitis? 1. Allergic rhinitis (AR) is a condition of adulthood and is more common in the older adult. 2. AR develops when the body's respiratory system becomes sensitized and overreacts to something in the environment. 3. Nonallergic rhinitis (NAR) is most common between ages 10 and 39, declining after age 40. 4. NAR is a condition of adulthood and is more common in the older adult.

7. Answer: 4 Page: 147 Feedback 1. AR is a condition of adulthood that declines in diagnosis after age 40. 2. AR develops when the body's immune system, not the respiratory system, becomes sensitized and overreacts to something in the environment that typically causes no problem in most people. 3. NAR is a condition of adulthood and is more common in the older adult. 4. NAR could be related to physiological changes in the older adult, as well as interactions from medications.

7. Challenges to increasing physical activity are different for everyone. For instance, some patients may be experiencing depression or loss, some may have had significant life changes, and others may be dealing with functional limitations. As relates to increasing physical activity, these are examples of what? 1. Patient excuses 2. Patient facilitators 3. Patient ignorance regarding what to do 4. Patient barriers

7. Answer: 4 Page: 20 Feedback 1. Depression, interpersonal loss, significant life event changes, and functional limitations are not choices that one makes so are not excuses. 2. Patient facilitators are things that encourage a patient to do what is needed for good health. 3. These examples are not the result of what one doesn't know. 4. These are examples of patient barriers because they discourage a patient from a desire to promote health.

8. When an assessment is conducted, which of the following needs to be recorded when a new concern is identified? Select all that apply. 1. Location and radiation. 2. Quantity and quality. 3. Aggravating and alleviating. 4. Signs and symptoms. 5. Effect on financial status.

8. Answer: 1, 2, 3, 4 Page:27 Feedback 1. A comprehensive assessment includes the location and radiation of the symptom. 2. A comprehensive assessment includes the quantity and quality of the symptom. 3. A comprehensive assessment includes aggravating and alleviating factors. 4. Associated signs and symptoms are included in a comprehensive assessment. 5. Financial concerns are considered after the findings of the assessment are understood.

8. A patient complains of dry, irritated, and itchy skin that has not been relieved with over-the-counter medications. On examination, the patient is also found to present cholestasis and jaundice. From which of the following conditions may the patient be suffering? 1. Lice 2. Scabies 3. Pruritus 4. Lichenification

8. Answer: 3 Page: 77 Feedback 1. Lice may cause itching in the scalp but is generally localized. 2. Scabies may cause itching of the skin but is generally localized. 3. Pruritus is an unpleasant, irritating, or itching sensation on the surface of the skin that may lead to a desire to scratch. Inflammation results from activation of the immune response. 4. Lichenification is the result of continued scratching and causes thickening of the skin.

12. Dizziness is an imprecise word used by patients to describe various subjective conditions. Care must be taken to establish what the patient means by dizziness in order to determine a diagnosis. Which of the following etiologies could be the cause of dizziness? Select all that apply. 1. Medications 2. Otolaryngologic conditions 3. Exercise 4. Psychiatric conditions 5. Hypothyroidism

12. Answer: 1, 2, 4 Page: 51 Feedback 1. Some medications can cause dizziness. 2. Otolaryngologic conditions can result in symptoms of dizziness. 3. Exercise does not usually result in dizziness unless it is associated with other pathologies. 4. Psychiatric conditions can produce symptoms of dizziness. 5. Hypothyroidism (underactive thyroid) is a condition in which the thyroid gland doesn't produce enough of certain important hormones. Signs and symptoms of this condition may include fatigue, increased sensitivity to cold, constipation, dry skin, and weight gain, but not dizziness.

13. Joint pain is a condition that is caused by joint damage or injury that can interfere with movement. From which of the following etiologies may joint pain arise? Select all that apply. 1. Bursitis 2. Muscle spasm 3. Gout 4. Fractured humerus 5. Muscle strain

13. Answer: 1, 3 Page: 72 Feedback 1. Bursitis is an inflammation of the fluid-field pads called bursa within the joint that can cause joint pain. 2. Muscle spasm is a condition that affects the muscles due to injury and does not necessarily cause joint pain. 3. Gout is a form of arthritis that causes joint pain. 4. Fractured humerus is an injury to the bone in the arm and does not necessarily cause joint pain. 5. A muscle strain is the stretching or tearing of muscle fibers. The tearing of the muscle can also damage small blood vessels, causing local bleeding (with or without bruising) and pain (caused by irritation of the nerve endings in the area). Pain is in the muscle and surrounding tissue rather than in the joint.

2. Which of the following terms refers to conditions that have multiple underlying factors and may involve multiple organs? 1. Multifaceted syndrome 2. Aging syndrome 3. CFM syndrome 4. Geriatric syndrome

2. Answer: 4 Page: 34 Feedback 1. Multifaceted syndrome is not the correct term, although conditions reflect the many contributing aspects to health care. 2. Aging syndrome is not the correct term, although aging may be a contributing factor to the condition. 3. CFM syndrome is not the correct term, although conditions may have cognitive, functional, and mobility impairments. 4. Geriatric syndrome is the term that refers to the conditions that have multiple underlying factors that may involve multiple organs.

4. Steven is a 65-year-old male diagnosed with Parkinson's disease. He is taking anti- Parkinson's drugs, calcium supplements, and NSAIDs for his joint pain. Which of the following conditions could be caused by his medications? 1. Constipation 2. Hematuria 3. Headache 4. Chest pain

4. Answer: 1 Page: 41 Feedback 1. Anti-Parkinson's drugs, calcium supplements, and NSAIDs are all known to cause constipation. 2. In older adult patients, the most common causes of hematuria are malignancy or benign prostatic hyperplasia. 3. Anti-Parkinson's drugs, calcium supplements, and NSAIDs typically do not cause headaches. Headache, however, may be a signal symptom for other neurological conditions. 4. Anti-Parkinson's drugs, calcium supplements, and NSAIDs typically do not cause chest pain. Chest pain may be a signal symptom for some cardiac conditions.

7. A health promotion topic included in the nurse practitioner visit is physical activity. Which of the following would be a good recommendation for the 75-year-old male? Select all that apply. 1. Skydiving 2. Horseback riding 3. Swimming 4. Dancing 5. Whatever the patient is willing and able to do

7. Answer: 3, 4, 5 Page: 7 Feedback 1. Skydiving is risky for older patients who may have cardiovascular issues. 2. Horseback riding could be risky for an older person who may have neurological or orthopedic issues. 3. Swimming is a good option if the patient is willing. 4. Dancing is a good option if the person is able. 5. When recommending physical activity, the person's functional abilities and desires need to be considered.

9. A 70-year-old man is seeing a nurse practitioner because he is feeling weak and dizzy. The nurse practitioner is discussing a care plan with him and makes which of the following statements? Select all that apply. 1. I want to order a test for your heart to evaluate its function. 2. I will perform a thorough examination of your functional abilities. 3. I will order a blood sugar test to check for diabetes. 4. Many older people feel this way. It is to be expected. 5. I will assess you for the presence of other conditions.

9. Answer: 1, 2, 3, 5 Page: 2, 3 Feedback 1. A complete assessment will help differentiate signs of aging from disease. 2. All systems interact and can affect an existing condition. 3. Identifying underlying conditions will avoid undertreatment. 4. Attributing symptoms to aging can contribute to depression in an older person. 5. There are changes in the sympathetic response which contribute to the orthostasis and falls, as well as lack of hypoglycemic response.

1. Mrs. Smith, 75 years old, reports that she is weak, has difficulty urinating, and is dehydrated. Although she is afebrile, the nurse conducts a thorough physical examination, including urinalysis and complete blood count (CBC). The total assessment is necessary because: 1. All body systems interact, and symptoms could indicate a variety of diagnoses. 2. The symptoms are vague and may be signs of aging. 3. There may be other signs or symptoms more indicative of the condition. 4. Mrs. Smith may not be reporting all significant information.

1. Answer: 1 Page: 2 Feedback 1. The clinician must be aware that all the systems interact and, in doing so, can increase the older person's vulnerability to illness/disease. 2. The nurse must not attribute symptoms only to the aging process. 3. There may be comorbidities accompanying this condition. 4. Assumptions of not reporting properly may not be true.

1. Which cranial nerve should be assessed in the examination for disorders of the face? 1. Cranial nerve III 2. Cranial nerve V 3. Cranial nerve VII 4. Cranial nerve IX

1. Answer: 3 Page: 127 Feedback 1. Cranial nerve III: oculomotor. This nerve is also known as the common ocular motor nerve. It controls eye movement and is also responsible for pupil size. It originates in the midbrain. 2. Cranial nerve V: trigeminal. This nerve's function is to carry sensitive information to the face and convey information for the chewing process. The sensory fibers convey sensations of touch, pain, and temperature from the front of the head, including the mouth and the meninges. 3. Cranial nerve VII: facial. This nerve should be assessed in the examination for disorders of the face. It consists of several nerve fibers that perform different functions, like ordering the muscles of the face to create facial expressions, as well as sending signals to the salivary and lacrimal glands. 4. Cranial nerve IX: glossopharyngeal. This nerve's influence lies in the tongue and pharynx. It collects information from the taste buds (tongue) and sensory information from the pharynx.

10. Mrs. Thomas is 82 years old and burned her hand while cooking. The nurse practitioner assesses second- and third-degree burns over approximately half of the back of her hand. The nurse practitioner chooses which of the following for initial treatment? Select all that apply. 1. Administer appropriate pain medication. 2. Rinse with cool tap water. 3. Clean with a strong detergent. 4. Remove any loose tissue but allow the blisters to remain. 5. Diagnose as first- and third-degree burns.

10. Answer: 1, 2, 4 Page: 100 Feedback 1. After administration of appropriate pain medication, wound management can begin. 2. Burn wounds should be immediately doused in cool tap water to disperse any remaining heat in the tissue. 3. Detergents and antibacterial soaps are not indicated. Burn wounds should be cleaned with mild soap and rinsed. 4. For small surface area burns, it is good to remove any loose tissue during cleansing and allow intact blisters to remain. 5. First-degree burns do not exhibit blisters, and third-degree burns do not exhibit pain.

11. An older woman is seen in the ambulatory clinic for a routine checkup. The patient asks about results of her blood work compared to last year's results. How is it best for the clinician to respond? Select all that apply. 1. They are likely to be the same if you have had no diseases since then. 2. Blood work results may be different just because of your aging process. 3. Your results may be different because the body changes with disease and medications you are taking. 4. Blood work is not a reliable indicator of health because many things can affect your health that may not show up in the results of your blood work. 5. There is likely no significant difference as your body remains stable in its functioning.

11. Answer: 2, 3 Page: 4, 5 Feedback 1. Biochemical individuality variation is often much smaller than variation within the larger group. 2. The aging process and decline in organ function may affect the health of elders. 3. Disease, nutrition, and medications affect the health of elders. 4. Although abnormal laboratory findings are often attributed to old age, rarely are they true aging changes. 5. Many factors can influence laboratory value interpretation in the elderly, including the physiological changes with aging, the prevalence of chronic disease, changes in nutritional and fluid intake, lifestyle (including activity), and the medications that are taken.

12. The treatment for cellulitis includes which of the following? Select all that apply. 1. Patients with mild cellulitis may be given oral antibiotics. 2. One drug of choice is dicloxacillin, 500 mg four times a day. 3. Treatment is dependent on the culture of the cells affected. 4. The drug of choice is given for a minimum of 3 days. 5. Administration of a tetanus booster injection.

12. Answer: 1, 2 Page: 103 Feedback 1. Oral antibiotics are sufficient for mild cellulitis and IV antibiotics for organisms such as MRSA. 2. There are several drugs effective with cellulitis; dicloxacillin is one of them. 3. Treatment of MRSA should be guided by wound culture results, but not cellulitis. 4. The drug of choice is typically given for 7 days. 5. If the wound is grossly contaminated and the patient's last tetanus booster was 5 to 10 years ago, the practitioner should consider giving another booster at this time.

14. Which of the following are the focus questions when assessing patients with pruritus? Select all that apply. 1. Did it come on suddenly or gradually? 2. How do you characterize the sensation? 3. What did you eat yesterday? 4. Is there anything that makes it worse or better? 5. What is the consistency of your bowel movements?

14. Answer: 1, 2, 4 Page: 77 Feedback 1. Symptoms of pruritis can begin gradually or suddenly, and can include a prickling, crawling, burning, or stinging sensation. 2. Having the patient characterize the sensation may help determine if the diagnosis is pruritus, as a feeling of prickling, crawling, burning, or stinging are associated with this disease. 3. Pruritus is not the result of foods you've eaten. 4. Knowledge of what makes it feel worse or better can help with treatment planning. 5. Consistency of bowel movement is not particularly associated with pruritus unless there is an underlying condition that is causing anal itching.

14. Mrs. Smith is 75 years old and has had type 2 diabetes for 30 years. She sees her nurse practitioner on a regular basis. Mrs. Smith asks the nurse practitioner why her treatment has not been changed, even though her laboratory values are seemingly out of normal range. Which of the following answers are best? Select all that apply. 1. A decision to make changes must be based on a pattern and not on one set of laboratory results. 2. The normal range charts are based on patients younger than 75 years. 3. Currently, it is too expensive to change your treatment. 4. Comparing new results with your prior results gives a more accurate picture. 5. Normal values for laboratory tests differ as one gets older.

14. Answer: 1, 4, 5 Page: 3 Feedback 1. Blood studies are more valuable when assessing for an increase or decrease in values. 2. It is not helpful to use aging as an explanation for possible abnormal results. 3. If a change in treatment is appropriate, the least expensive alternative should be sought. 4. Significant disturbances in the same individual may be detected through serial laboratory tests. 5. The clinician must determine whether a value obtained reflects a normal aging change, a disease, or the potential for disease.

2. Many adults find reasons or barriers for not being able to exercise. Which of the following is an example of a patient barrier? 1. Good health, no functional limitations. 2. Insurance incentive. 3. High body mass index (BMI). 4. Frequent contact with prescriber.

2. Answer: 3 Page: 20 Feedback 1. Good health, no functional limitations is an example of a patient facilitator because it is not counterintuitive to pursuing good health. 2. Insurance incentive is an example of a patient facilitator because it provides incentive for the patient. 3. High BMI is an example of a patient barrier because it is something that may seem insurmountable to a patient. 4. Frequent contact with the prescriber is an example of a patient facilitator because it provides accountability in the pursuit of good health.

2. The nurse practitioner knows that health promotion is accomplished on a broader scale by: 1. Maintaining current health services. 2. Maintaining federal legislation regarding health care at the current level. 3. Encouraging healthy lifestyle and healthy public policy formulation. 4. Encouraging the advancement of alternative health therapies.

2. Answer: 3 Page: 6 Feedback 1. Current health policies are ever-changing to improve services for citizens. 2. Federal legislation is being proposed to increase health services. 3. The World Health Organization has determined that healthy lifestyle promotion works best when coupled with supportive environments, community action, and healthy public policy formation. 4. This does not include preventive and health-protective measures, nor actualization of one's health potential.

3. Which of the following statements is true regarding diagnostic testing? 1. A test is ordered for a specific purpose. 2. A test is the most invasive available. 3. There is no need to discuss results with the patient. 4. If a test is needed, it should be ordered regardless of risk to the patient.

3. Answer: 1 Page: 3 Feedback 1. The nurse practitioner should have a plan for the use of each test result value obtained. 2. When considering which laboratory tests to order, it is worth remembering the doctrine primum no nnocere—first, do no harm. 3. Once laboratory tests are available for review, tests results should be discussed with the patient, with abnormal test results interpreted for the aging individual and addressed with the patient and caregivers. 4. Any risks involved in laboratory testing must be considered concerning the patient's clinical condition and weighed against the test's expected benefits.

4. Janey, 25 years old, may experience arthritis differently than 65-year-old Mrs. Johnson because: 1. The body undergoes physiological changes with aging. 2. A healthy body does not experience significant changes as one gets older. 3. Older patients do not feel any systemic symptoms, such as malaise and weight loss. 4. Even though the same joints are usually affected, age makes it feel different.

4. Answer: 1 Page: 5 Feedback 1. Knowledge of the bimodality of age onset of certain disease conditions will aid the advanced practice nurse in avoiding misdiagnosis or delay in diagnosis due to lack of recognition. 2. Symptoms of rheumatoid arthritis may be different depending on the age of the patient. 3. Younger patients may not experience constitutional symptoms such as fever, malaise, weight loss, and depression. 4. In late-onset rheumatoid arthritis, the joint involvement is more often in the larger joints.

4. A family member comes into the clinician's office with her older mother, the patient. In the process of the examination, the nurse practitioner identifies the patient as "frail older adult." The family member asks what that means. The nurse practitioner explains that this refers to: 1. Someone who is prone to pathological fractures. 2. An older person already exhibiting signs of decline. 3. An older person who has lost height and is under 5 feet tall. 4. An older person being susceptible to infections.

4. Answer: 2 Page: 26 Feedback 1. Frailty is not defined by only one symptom. 2. Frailty, or signs of declining health, can occur at any age. 3. Frailty is not related to the size of the patient. 4. Infections can be treated at any age.

5. Jane, 83 years old, comes to the clinic with her daughter for a geriatric assessment. The daughter asks about the necessity of this comprehensive assessment. The clinician explains that a CGA is a multidimensional process to: 1. Identify care needs and plans of care for older people. 2. Acquire information about past medical conditions. 3. Determine which diagnostic tests are necessary. 4. Satisfy requirements of Medicare.

5. Answer: 1 Page: 26 Feedback 1. The CGA is a multidimensional, interdisciplinary, and diagnostic process to identify needs and plan outcomes. 2. The CGA includes information from medical history, as well as the current assessment. 3. The CGA helps identify diagnostics, treatments, and other plan-of-care strategies. 4. The CGA is not a requirement of Medicare.

9. With the recognition that baby boomers will increase the over 65 population exponentially, guidelines for increasing physical activity in older adults have been issued. Which of the following programs is dedicated to health promotion in older adults? Select all that apply. 1. The Healthy People 2020 initiative 2. The Center for Medicare and Medicaid 3. The American Medical Association 4. The National Institute on Aging 5. The American Heart Association (AHA)

9. Answer: 1, 2, 4 Page: 19 Feedback 1. The Healthy People 2020 initiative has several sections dedicated to health promotion for older adults. 2. The Center for Medicare and Medicaid has programs to promote wellness and quality of life in older adults. 3. The American Medical Association promotes the art and science of medicine and the betterment of public health. 4. The National Institute on Aging has programs to promote wellness and quality of life in older adults. 5. The AHA publishes the scientific guidelines for cardiopulmonary resuscitation and emergency cardiovascular care, which form the foundation of lifesaving protocols used by health-care professionals, companies, and hospitals in the United States and throughout the world.

9. Mr. Thomas noticed tremors in his right hand 2 years ago and attributed this to his elbow injury from a fall. The tremors have gradually worsened, and he now has tremors under his lip, some gait problems, and occasional stomach spasms. His wife has recently noticed these things and suggested he have a medical consultation. Which of the following reasons might Mr. Thomas have had for ignoring these symptoms? Select all that apply. 1. Fear of loss of independence 2. Fear of doctors 3. Symptoms do not interfere with functions 4. Attributes symptoms to the aging process 5. Lack of transportation

9. Answer: 1, 3, 4 Page: 34 Feedback 1. Fear of loss of independence may result in older adults not seeking medical help for illnesses. 2. Fear of doctors is usually not a factor. 3. If symptoms do not interfere with one's ability to function, sometimes medical help will not be sought. 4. If an individual has a feeling that his symptoms are simply the results of aging, he may not feel there is a reason to seek medical help. 5. Not having transportation to get to the medical provider is not a factor because many facilities now have transportation services for patients who need it.

9. After examining Adam, who is 79 years old, the nurse practitioner uses which of the following criteria to adjust Adam's medications? Select all that apply. 1. The Beers Criteria published in 1991 2. Medication Appropriateness Index 3. STOPP/START Criteria 4. HHS Guide to Geriatric Medications 5. Hospital formulary

9. Answer: 2, 3 Page: 27 Feedback 1. The Beers Criteria, 1991, was revised in 2015. Other criteria were also developed to address weaknesses found in the original Beers Criteria. 2. The Medications Appropriateness Test was developed to address some of the weak areas of the Beers Criteria. 3. The STOPP/START Criteria were developed to determine when medications should be added and when another might be discontinued for an older patient. 4. The HHS Guide to Geriatric Medications does not guide in medication prescribing. 5. Hospital formulary provides a list of medications carried within that hospital.

1. Mrs. Williams is 76 years old and comes in to have a wound checked on her right leg. She fell a month ago and the wound has not healed. She is concerned that something is wrong. The nurse practitioner examines the wound and sees that it has been cleaned properly and has no signs of infection. The edges are approximated, but the skin around the wound is red and tender to touch. The best response regarding Mrs. Williams' concern is: 1. Wound healing for older people may take up to four times longer than it does for younger people. 2. Let us talk about what you are eating. 3. Had you come in earlier, I would have ordered medicine that would have healed that right up. 4. I will order an antibiotic to prevent infection.

1. Answer: 1 Page: 96 Feedback 1. Skin renewal turnover time increases to approximately 87 days in older adults, compared with 20 days during youth. 2. The perceived extended healing time is not related to diet. 3. This is false hope, as there is no medication that will heal this wound quickly. 4. Prophylactic antibiotics are not appropriate when there are no signs or symptoms of infection.

1. The nurse practitioner is conducting a health of older adults in-service with a group of older adults at a community center. The first concept she presents is a definition of chronic illness. Which of the following is the best definition? 1. A situation that considers the meaning of the more than one long-term illness on the patient, family, and provider. 2. A condition that requires medical attention for at least 6 months. 3. Any condition that requires time-limited adjustments. 4. There is only one definition of chronic illness.

1. Answer: 1 Page: 474 Feedback 1. Definitions of chronic disease also reflect the pathophysiology of disease and, more importantly, consider the meaning of chronic illness and the experience of the patient, family, and provider as they struggle to cope with the range of mildly complicated to extreme challenges. 2. One definition of chronic conditions is: "conditions lasting 1 year or more and requiring ongoing medical attention and the limiting of activities of daily living (ADLs)." 3. Chronic disease or a chronic condition is also defined as any condition that requires ongoing adjustments by the affected person and requires periodic interaction with the health-care system. 4. The literature does not support a single uniform definition for chronic disease, but the concept of chronicity includes the knowledge that patients experience persistent and recurring health problems.

1. Mrs. Keating is a 70-year-old woman suffering from arthritis. She is seeing the nurse practitioner for her regular checkup. For a person with chronic degenerative conditions, which of the following should the nurse suggest for health promotion? 1. Take herbal supplements. 2. Increase physical activity. 3. Take Aleve. 4. Get a massage.

1. Answer: 2 Page: 19 Feedback 1. Although there may be some herbal supplements that can help with arthritis, this is not the best suggestion because it does not promote increased physical activity. 2. Increased physical activity in older adults promotes wellness and quality of life. 3. Medication may be helpful for relief of symptoms but does not necessarily promote increased physical activity, which can help lead to better health. 4. Massage may provide temporary relief for arthritic joints, but physical activity is what can help lead to better health.

1. A patient comes to the health-care provider office for a checkup. The nurse practitioner notes that the patient is 67 years old and is presenting symptoms of dizziness, orthopnea, and edema. The nurse practitioner begins a focused assessment of the: 1. Cardiovascular system. 2. Respiratory system. 3. Neurological system. 4. Reproductive system.

1. Answer: 1 Page: 152 Feedback 1. Key symptoms of cardiovascular assessment include dizziness, syncope, orthopnea, angina, edema, and claudication. 2. Edema is not commonly found in respiratory conditions. 3. Dizziness is a symptom in various conditions besides neurological conditions. 4. Dizziness, orthopnea, and edema have no direct connection with the reproductive system.

1. In the assessment for peripheral vascular disorders, the clinician should begin with which of the following after the general history and physical examination? 1. Auscultation of the carotid arteries bilaterally. 2. Palpation of the carotid arteries bilaterally. 3. Inspection of the carotid arteries bilaterally. 4. Order diagnostic testing.

1. Answer: 1 Page: 215 Feedback 1. The clinician should begin with auscultation of the carotid arteries bilaterally because cardiac murmurs will usually radiate into the carotid arteries. 2. After auscultation of the carotid and subclavian arteries, the clinician should proceed to palpation of the brachial, radial, and ulnar arteries. 3. After auscultation and palpation, careful inspection should be performed of the distal fingers and the nail beds. 4. Diagnostic testing is usually ordered when areas of concern are found.

1. Mrs. M. W. comes into the primary care office for her regular yearly checkup. The nurse practitioner begins the assessment by asking if there are concerns Mrs. M. W. has that need to be addressed. The patient says that she has been experiencing periodic episodes of abdominal cramps and diarrhea that just comes and goes. The nurse practitioner continues the assessment with which of the following? 1. Asking about the sequence of events and events that triggered each episode. 2. Conducting a complete neurological examination. 3. Collecting a urine specimen. 4. Evaluating abdominal pain.

1. Answer: 1 Page: 225 Feedback 1. Determining the sequence of events that triggered each symptom and inquiring about precipitating factors such as a meal, position of the body, use of caffeine, or alcohol and smoking will lead to a differential diagnosis. 2. Information regarding episodes of anorexia, dyspepsia, dysphagia, heartburn, nausea, regurgitation, vomiting, painful or difficult defecation, diarrhea, tenesmus, or constipation can be clustered to form a differential diagnosis. 3. The physical examination is often unremarkable, and laboratory findings may not provide diagnostic information because the presentation of illness in an older adult is usually subdued. 4. Ask about the severity of pain by having the patient rate her pain on a scale of 1 to 10. Realize, however, that in older adults, pain may be blunted despite the underlying pathology.

1. Mrs. Borden is 67 years old and her primary care physician wants her to be seen by a nurse practitioner who specializes in geriatric issues. On completing the interview and examination, the nurse practitioner tells Mrs. Borden that she will order a blood laboratory specimen for thyroid function and pancreatic function. Mrs. Borden asks what the reason is. Which of the following is the nurse practitioner's best response? 1. Diabetes is more common in older patients. 2. Thyroid issues are less common in older patients. 3. These are common tests to be done on any new patient. 4. These tests are necessary to obtain baseline information.

1. Answer: 1 Page: 361 Feedback 1. The nurse practitioner recognizes that normal aging changes of the endocrine system primarily are related to a decrease in pancreatic function (inability to sufficiently secrete insulin), resulting in diabetes mellitus. 2. The nurse practitioner needs to screen older adults periodically for diabetes mellitus and thyroid disease because the incidence of these conditions increases with age. 3. The nurse practitioner managing the care of older adults needs to differentiate among four clinical states. One of those is an endocrine function that is altered relative to that of younger patients, but is an expected consequence of normal aging. 4. Overall, the nurse practitioner managing the care of older adults needs to differentiate among four clinical states: 1. An endocrine function that is altered relative to that of younger patients, but is an expected consequence of normal aging. 2. Altered endocrine function secondary to coincident nonendocrine disease, but is not of known pathological significance. 3. Iatrogenic changes in endocrine function that largely reflect the polypharmacy seen in the older adult population. 4. Authentic endocrinopathy.

1. Polypharmacy is a primary predictor for any undesired or unwanted consequences that can occur because of taking medications. Which is of the following is the correct term for these undesired or unwanted consequences? 1. Adverse drug reaction (ADR) 2. Prototype drug 3. Adverse drug event 4. Schedule II drugs

1. Answer: 1 Page: 470 Feedback 1. ADR is a reaction attributed to the drug. It is any undesired or unwanted consequence that occurs because of taking medications. 2. A prototype drug is the original drug model from which other medications in a pharmacological class have been developed. 3. An adverse drug event is any event that occurs during treatment and is not necessarily caused by the drug itself. It is a side effect that was revealed after usage of the drug and is reported by the patient or the doctor who has personal experience with the event. 4. Schedule II drugs are drugs with abuse potential, and psychic and physical dependence liability.

1. The nurse practitioner is serving in a clinic dedicated to patients over age 65 years. The nurse practitioner observes several common characteristics in the posture of elderly patients. Which of the following is common? 1. Increase in the length of the trunk in relationship to the upper extremities. 2. Head tips backward. 3. Disappearance of bony prominences. 4. Increase in the range of motion (ROM).

1. Answer: 2 Page: 305 Feedback 1. Aging often brings about a decrease in height, resulting from a decrease in the length of the trunk with respect to the length of the extremities. 2. An older person may tilt the head backward to compensate for the bend in the thoracic spine, producing the typical posture of those in this age group. 3. Because of the loss of subcutaneous fat caused by aging, bony prominences became more noticeable. 4. Without continued use, muscles stiffen and ROM becomes impaired as an older person continues to age.

1. Jean is a breast cancer patient given 2 to 3 years of survival. Her condition is up and down. Which of the following types of care is indicated for the patient? 1. Naturopathic care 2. Complementary care 3. Maintenance care 4. Palliative care

1. Answer: 4 Page: 485 Feedback 1. Naturopathic medicine and care is a distinct primary health-care profession emphasizing prevention, treatment, and optimal health through the use of therapeutic methods and substances that encourage individuals' inherent self-healing process. 2. Complementary medicine and care is used together with mainstream medical care. Examples include acupuncture, yoga, chiropractic and osteopathic manipulation, meditation, and massage therapy. 3. Maintenance care is care to prevent future relapses and maintain the condition after active care has been completed. 4. Palliative care is an interdisciplinary approach to care aimed at improving the quality of life of patients and their families facing a life-threatening illness.

10. The nurse practitioner is seeing patients in the respiratory clinic. An 85-year-old woman comes in with concerns about having difficulty breathing. When considering asthma, the nurse practitioner asks which of the following questions? Select all that apply. 1. Do you feel out of breath during the day? 2. Do you have a cough, especially at night? 3. Do you feel a tightness in your chest? 4. May I listen for wheezing in your breathing? 5. Do you feel any dyspnea?

10. Answer: 1, 2, 3, 4 Page: 155 Feedback 1. Chronic inflammation is associated with airway hyperresponsiveness that leads to breathlessness. 2. Chronic inflammation is associated with airway hyperresponsiveness that leads to coughing, particularly at night or early in the morning. 3. Chronic inflammation is associated with chest tightness. 4. Chronic inflammation is associated with airway hyperresponsiveness that leads to recurrent episodes of wheezing. 5. Older adults are less likely to sense dyspnea related to airway obstruction.

10. Mrs. Morrison is 85 years old and on Medicare with supplemental insurance. She is obese and has diabetes and heart failure. The nurse practitioner collaborates with a social worker and with a representative from Medicare about the cost of her treatment choices. The nurse practitioner analyzes treatment burden and the costs of care to develop a practical and cost-effective plan of care. Which of the following puts Mrs. Morrison at risk for a financial burden for health care? Select all that apply. 1. Chronic medical conditions. 2. Obesity and diabetes. 3. No history of hospitalization in the last 5 years. 4. Higher costs of prescriptions. 5. Lower total out-of-pocket expenses.

10. Answer: 1, 2, 4 Page: 478 Feedback 1. She is obese and has diabetes and heart failure. 2. The medical care costs of obesity in the United States continue to be overwhelming 3. Additional costs of chronic disease related to the multiple acute care hospitalizations experienced by severely chronically ill populations. 4. Individuals with MCCs also have substantial personal economic challenges related to higher costs for prescription drugs. 5. Individuals with MCCs also have substantial personal economic challenges related to higher costs of care and total out‐of‐pocket costs.

10. During a breast examination, the patient should be asked if she has which of the following? Select all that apply. 1. Breast pain 2. Breast atrophy 3. Breast rash 4. Breast trauma 5. Proper support

10. Answer: 1, 3, 4 Page: 280, 281 Feedback 1. A patient should be asked if she is experiencing any breast pain and should point out the location. Description of the pain is also important. 2. Breast atrophy is an observation to be made by the nurse practitioner and not a question to be asked of the patient. 3. A patient should be asked if she has observed any rash on or around the breast. Also ask about onset, location, and sensations of the rash. 4. Patients should be asked if there was any breast trauma, as this could lead to injury of the breast. 5. Asking about the use of proper support may be helpful for some individuals, but it is not generally a question that is asked during the breast examination.

10. Mrs. Rivets is an 80-year-old female who comes to the emergency department with complaints of a low-grade fever and pain in her lower leg that is aggravated by movement. Which early treatment does the nurse practitioner implement? Select all that apply. 1. Protection of the lower leg, rest, and application of an ice pack. 2. Salt compresses and an antibiotic. 3. Compression dressing and an antibiotic. 4. Compression dressing and warm compresses. 5. Elevation of the leg and pain medication.

10. Answer: 1, 5 Page: 309 Feedback 1. The treatment of acute soft tissue syndromes begins conservatively: protect with padding, braces, and changes in techniques; rest by avoiding activities that exacerbate pain, and use ice, as cryotherapy can relieve pain and decrease inflammation. 2. Salt compresses are not necessary, but compression dressings are helpful. If suspected infection/sepsis, then antibiotics are ordered. 3. Only when there is suspected infection/sepsis is a 4-week course of sensitivity-susceptible antibiotics is advised. 4. Compression and cold compresses will reduce swelling. 5. The treatment of acute soft tissue syndromes starts conservatively: elevation, raise the affected limb above the level of the heart; modalities, electrical stimulation or ultrasound; medications, such as NSAIDs, acetaminophen, and/or corticosteroid injections.

10. A patient who is an 80-year-old female comes to the emergency department with complaints of a low-grade fever and left lower quadrant pain aggravated by movement. She also reports that the pain is precipitated by eating and describes a colicky pain in the right side of the abdomen. Which diagnostic tests does the nurse practitioner order? Select all that apply. 1. A total body CT scan 2. Abdominal x-rays 3. CBC, amylase, lipase 4. Urinalysis 5. Sigmoidoscopy

10. Answer: 2, 3, 4 Page: 250 Feedback 1. Patients should be scheduled for a CT scan of the abdomen and pelvis. CT scans performed with oral, IV, and rectal contrast can enhance the accuracy of the diagnostic image. 2. Initially, plain abdominal radiographs can be ordered. 3. A CBC, amylase, lipase, urinalysis, complete C-reactive protein, and sedimentation rate need to be ordered to distinguish diverticulitis from other causes of acute abdominal pain. 4. Urinalysis may reveal sterile pyuria due to adjacent colonic irritation. 5. Sigmoidoscopy, colonoscopy, and barium enema are usually avoided during acute diverticulitis because these tests may cause further perforation or leakage of bowel contents.

10. Hospice services are an option for patients with end-stage life-threatening illness. These services are covered by Medicare, Medicaid, and most private insurance providers. According to studies, which of the following are the current leading causes of death? Select all that apply. 1. Elective surgeries 2. Falls and accidents 3. Malignant neoplasms 4. Cerebrovascular accidents 5. Diverticulitis

10. Answer: 3, 4 Page: 485 Feedback 1. Although complications may arise from elective surgeries and cause illness, it is rare that it leads to death. 2. Falls and accidents may cause death or long-term disabilities, but it is not one of the leading causes of death. 3. Malignant neoplasms are collectively known as cancers. Malignant neoplasms display aggressive characteristics, can invade and destroy adjacent tissues, and can spread to distant sites. According to studies, malignant neoplasms are one the leading causes of death. 4. Cerebrovascular accident (CVA) is the medical term for a stroke. A stroke is when blood flowing to a part of the brain is stopped either by a blockage or the rupture of a blood vessel. According to studies, CVA is one of the leading causes of death. 5. Diverticulitis is a gastrointestinal disease characterized by inflammation of abnormal pouches (diverticula) that develop in the wall of the large intestine. Symptoms typically include sudden-onset lower abdominal pain. Diverticulitis is not one of the leading causes of death.

10. Mark, 82 years old, is in a rehabilitation center for therapy post-knee replacement. The nurse practitioner is visiting Mark and finds him confused, disoriented, and with speech difficulty. The staff nurse says that his condition changed quickly over the last few hours. The nurse practitioner orders which of the following? Select all that apply. 1. Pulse oximetry 2. Evaluation for infections 3. Assessment of change in medications 4. Brain scan 5. Radiology

10. Answers: 1, 2, 3, 4 Page: 440 Feedback 1. A careful medical evaluation that includes attention to the level of oxygenation is necessary to determine the cause of delirium. 2. A careful medical evaluation that includes attention to the level of oxygenation and possible occult infection (e.g., urinary tract infection), is desired. 3. The role of medications is essential. Although many medications can be a causative factor, those with anticholinergic effects are frequently responsible. 4. The following neurological events can contribute to delirium: stroke, intracranial hemorrhage, head trauma, seizures, and undiagnosed pain. 5. Changes in brain function, multiple general medical problems, polypharmacy, reduced hepatic metabolism of medications, multisensory declines, and brain disorders, such as dementia, make the older adult particularly vulnerable to delirium.

11. A patient is being assessed for symptoms of headaches, changes in mental function, language deficits, and visual problems. Diagnostic testing reveals that he does not have a brain tumor. Which of the following may be the differential diagnosis for the patient's symptoms? Select all that apply. 1. Stroke 2. Subdural hematoma 3. Meningitis 4. Venous insufficiency 5. Diabetes mellitus

11. Answer: 1, 2, 3 Page: 332 Feedback 1. Stroke is the sudden death of brain cells due to lack of oxygen, caused by blockage of blood flow or rupture of an artery to the brain. Sudden loss of speech, weakness, or paralysis of one side of the body can occur. Stroke may be a differential diagnosis for symptoms of headaches, changes in mental function, language deficits, and visual problems. 2. Subdural hematoma is the bleeding into the space between the dura and the brain itself. Subdural hematomas can be caused by minor accidents to the head, major trauma, or the spontaneous bursting of a blood vessel in the brain. Subdural hematoma may be a differential diagnosis for symptoms of headaches, changes in mental function, language deficits, and visual problems. 3. Meningitis is the inflammation of the meninges, the three membranes that envelop the brain and the spinal cord. Meningitis can be caused by infection by bacteria, viruses, and protozoa. Meningitis may be a differential diagnosis for symptoms of headaches, changes in mental function, language deficits, and visual problems. 4. Venous insufficiency is a medical condition in which blood pools in the veins, straining the walls of the vein. It could cause swelling and ulcer formation in the limbs. Venous insufficiency may be a differential diagnosis for peripheral neuropathy with symptoms of paresthesia, sensory ataxia, and autonomic dysfunction. 5. Diabetes mellitus frequently affects the peripheral nervous system and is the most common cause of neuropathy.

11. Drug distribution in the body is affected by aging and higher body fat. Which of the following is true regarding drugs distributed in fat? Select all that apply. 1. There is a wider effect in drugs distributed in fat. 2. The effect of drugs is less when distributed in fat. 3. There is prolonged action in drugs distributed in fat. 4. There is a more intense effect of drugs distributed in fat. 5. There is a shorter effect of drugs distributed in fat.

11. Answer: 1, 2, 3 Page: 471 Feedback 1. Drugs distributed in fat have a wider distribution. 2. Drugs distributed in fat have a less intense effect. 3. Drugs distributed in fat have more prolonged action. 4. Drugs distributed in fat have a less intense effect. 5. Drugs distributed in fat have more prolonged action.

11. The nurse practitioner is conducting an initial history and physical examination, along with a health assessment on a new patient. Mr. Carter is African American and 68 years old. He presents with complaints of dizziness, unstable gait, and polyuria. This combination of symptoms is blamed for his frequent falling at home. The nurse practitioner analyzes the findings of the examination and suggests which of the following diagnostics? Select all that apply. 1. Blood pressure measurements, including measuring for hypostatic hypertension to rule out hypertension. 2. Fasting blood sugar and a glucose tolerance test to rule our diabetes. 3. Testing for chronic kidney disease (CKD). 4. Measurement of BMI. 5. Definitive testing for Alzheimer's disease.

11. Answer: 1, 2, 3 Page: 478 Feedback 1. Non-Hispanic African Americans are 40% more likely to have high blood pressure. 2. Diagnosed diabetes is 77% higher among non-Hispanic African Americans, 66% higher among Hispanics, and 18% higher among Asians than among non-Hispanic Caucasians. 3. Although CKD has not increased in overall prevalence, there remains a continued increase of CKD in non-Hispanic African Americans. 4. Native Americans and Alaska Natives are 60% more likely to be obese than non-Hispanic Caucasians. 5. The literature shows consistent disparities among African Americans and Hispanics compared to non-Hispanic Caucasians in the prevalence and incidence of Alzheimer's disease.

11. Which of the following statements is true regarding palliative care and hospice care? Select all that apply. 1. Palliative care can begin at diagnosis, and at the same time as treatment. 2. Palliative care may be offered by hospitals, home care agencies, cancer centers, and long-term care facilities. 3. Hospice care begins after treatment of the disease is stopped and when it is clear that the person is not going to survive the illness. 4. Only hospice care provides comfort. 5. Only a patient's family can give the best care.

11. Answer: 1, 2, 3 Page: 485, 486 Feedback 1. The goal of palliative care is to help people with serious illnesses feel better. It prevents or treats symptoms and side effects of disease and treatment. Palliative care can be given at the same time as treatments meant to cure or treat the disease. Palliative care may be given when the illness is diagnosed, throughout treatment, during follow-up, and at the end of life. 2. Any health-care provider can give palliative care, but some providers specialize in it. Palliative care may be offered by hospitals, home-care agencies, cancer centers, and long-term care facilities. 3. Hospice care begins after treatment of the disease is stopped and when it is clear that the person is not going to survive the illness. Hospice is a program of care designed for the last 6 months of a person's life. 4. Both hospice and palliative care provide comfort. 5. Palliative care is a collaborative effort of medical providers. Although a patient's family can provide this care, most times the family also needs the services of palliative care to offer support and help them cope during the patient's illness and in their own bereavement.

11. The nurse practitioner is preparing discharge instructions for a 79-year-old man with newly diagnosed asthma. Which of the following are important instructions to include? Select all that apply. 1. Infections can lead to an occurrence of asthma. 2. Experiencing strong emotions like anger can trigger an asthma episode. 3. As you get older, you are more likely to experience difficulty breathing. 4. Symptoms of asthma may include a runny nose or postnasal drip. 5. There is no need to follow up with your doctor unless your symptoms get worse.

11. Answer: 1, 2, 4 Page: 155 Feedback 1. Symptoms may occur in conjunction with a respiratory infection, weather changes, or contact with environmental allergens. 2. Symptoms may occur in conjunction with strong emotional reactions, animal fur, mold, or exercise. 3. Older adults are less likely to sense dyspnea related to airway obstruction. 4. Other symptoms may include rhinorrhea with postnasal drip. 5. Regular monitoring 1 to 3 months after initiation of treatment, and then every 3 to 12 months depending on the level of control and severity. Patients should be seen within 1 week after an asthma exacerbation, and within 2 to 6 weeks when changes are made in the stepwise management.

11. Mr. Jamison, a 67-year-old male, is being seen in the clinic for complaints of pain in his thigh. There is swelling in the soft tissue, dislocation, altered ROM, bruising, local tenderness, and pain with any motion. The nurse practitioner suspects a femoral neck fracture. What could have contributed to this fracture? Select all that apply. 1. Direct injury to the femur. 2. Sleeping on his side. 3. Osteoporosis. 4. Heavy lifting. 5. Bending over at the waist.

11. Answer: 1, 3 Page: 310 Feedback 1. Common sites for fractures include proximal humerus, distal radius, pelvic ramus, proximal femur, proximal tibia, and thoracic and lumbar vertebral bodies. 2. Symptoms do not indicate soreness from sleeping on the side. 3. Fractures most attributed to osteoporosis include femoral neck fractures, pathological fractures of the vertebrae and lumbar spine, and thoracic spine vertebral fractures. 4. Vertebral fractures may result from an activity, such as heavy lifting. 5. Vertebral fractures may result from an activity, such as bending over, that puts sudden stress on the spine.

11. In treating AAA, arterial monitoring is recommended. Which of the following IV antihypertensive agents should be used to rapidly and consistently maintain blood pressure in the mean arterial pressure ranges? Select all that apply. 1. Nicardipine 2. Sertraline 3. Esmolol 4. Nitroglycerin 5. Carbidopa

11. Answer: 1, 3, 4 Page: 217 Feedback 1. Nicardipine is an IV antihypertensive agent that should be used to rapidly and consistently maintain blood pressure in mean arterial pressure ranges. 2. Sertraline is used to treat depression, panic attacks, obsessive compulsive disorder, post-traumatic stress disorder, and social anxiety disorder. 3. Esmolol is an IV antihypertensive agent that should be used to rapidly and consistently maintain blood pressure in mean arterial pressure ranges. 4. Nitroglycerin is an IV antihypertensive agent that should be used to rapidly and consistently maintain blood pressure in mean arterial pressure ranges. 5. Carbidopa is a medication used with a combination to treat symptoms of Parkinson's disease or Parkinson-like symptoms.

11. Eye health education should be provided to maintain or minimize visual problems. Which of the following should be included in patient education? Select all that apply. 1. The importance of protecting the eyes from sunlight. 2. Eye examinations should be performed every 5 years. 3. Eating more protein with meals will help eye health. 4. Information on age-related visual changes 5. The importance of wearing eye goggles when swimming.

11. Answer: 1, 4 Page: 129 Feedback 1. The importance of protecting the eyes from sunlight with UV-blocking sunglasses should be included in the patient eye health education. 2. Recommending periodic eye examinations every 2 years should be included in the patient eye health education. 3. The patient should be instructed to maintain a nutritionally balanced diet including green, leafy vegetables. 4. Because age-related changes do occur with vision, giving patients this information will help them be aware of what to look for. 5. Though not generally a part of patient education, the recommendation to wear eye goggles when swimming may be helpful to minimize eye irritation from chlorine, salt water, or any other contaminants.

11. Bill James, 67 years old, is being seen in the clinic for complaints of regurgitation, pyrosis, hoarseness, chronic cough, and atypical chest pain. The nurse practitioner suspects gastroesophageal reflux disease (GERD). As home-going instructions are developed, the nurse practitioner includes which of the following? Select any that apply. 1. Raise the head of the bed 2 inches. 2. Avoid tight, restrictive clothing. 3. Avoid smoking and ingestion of fatty foods. 4. Avoid eating a meal for at least 20 minutes before becoming recumbent. 5. Avoid caffeine and acidic foods.

11. Answer: 2, 3, 5 Page: 262 Feedback 1. Patients with reflux esophagitis should be instructed to raise the head of the bed 4 to 6 inches with shock blocks or use a foam wedge that can be placed at the head of the bed. 2. While there has been limited evidence to suggest that making lifestyle changes (such as avoiding tight, restrictive clothing to decrease abdominal pressure) counteracts the symptoms of GERD, nurse practitioners should encourage patients to adopt this strategy. 3. The patient should avoid smoking and ingestion of fatty foods, coffee, chocolate, mints, citric juices, alcohol, and large quantities of fluids with meals. 4. Patients should avoid eating a meal for at least 3 hours before becoming recumbent. 5. Important to the management of GERD are the lifestyle changes or nonpharmacological measures that patients need to incorporate along with the medication regimen. These include avoidance of smoking, alcohol, and food products such as chocolate, mints, spicy or acidic foods, and caffeine.

11. When other symptoms are presented with atrophic vaginitis, a biopsy is used as the diagnostic test to rule out which of the following diagnoses? Select all that apply. 1. Urinary tract infection (UTI) 2. Sexually transmitted infection (STI) 3. Squamous cell hyperplasia 4. Lichen sclerosis 5. Lichen planus

11. Answer: 3, 4, 5 Page: 282 Feedback 1. UTI presents with the symptom of pain with urination and is usually treated with an antibiotic. No biopsy is indicated to diagnose. 2. STIs present symptoms of pruritis, burning, and a thick white discharge. No biopsy is indicated to diagnose. 3. Squamous cell hyperplasia presents symptoms of pruritus, which results in scratching and inflammatory changes in squamous cells of the vulvar area. These are seen on a biopsy. 4. Lichen sclerosis presents with whitish lesions on the vulva and squamous cells. A biopsy is needed to diagnose. 5. Lichen planus shows papular, purple lesions that are pruritic and thought to be immunological in origin. A biopsy is needed to diagnose.

12. Mr. John has recently been given a diagnosis of diabetes. He asks the nurse practitioner if can try something nonpharmacological to control his diabetes. Which of the following would the nurse practitioner propose? Select all that apply. 1. Weight loss and exercise. 2. Self-monitoring blood glucose. 3. A moderate increase in activity. 4. Frequent follow-up appointments with the nurse practitioner. 5. Enrolling in a high-intensity physical activity program.

12. Answer: 1, 2, 3, 4 Page: 371 Feedback 1. One goal of treatment for patients with type 2 diabetes is weight management. 2. The frequency and timing of self-monitoring of blood glucose levels (SMBG) need to be individualized based on several factors, including age, ability to adhere, and oral versus insulin therapies. SMBG results in promoting glycemic control and reinforcing adherence to therapy. Even patients maintaining glucose levels by nonpharmacological means may benefit from intermittent either postprandial or fasting SMBG. 3. Lifestyle modifications of weight loss and exercise are particularly important. 4. After 3 to 6 months, if nonpharmacological treatment fails or the hyperglycemia is severe (fasting plasma glucose 200 to 300 mg/dL or a casual plasma glucose 250 to 350 mg/dL), oral agents may be added to the treatment regimen. 5. Although weight loss in frail older adults is not recommended due to the risk of sarcopenia, exercise of even a modest nature can be beneficial in decreasing insulin resistance.

12. Nosebleeds are generally minor and nonrecurring, but when a patient is seen for this, a consultation with an ear, nose, and throat (ENT) specialist is indicated when which of the following conditions occurs? Select all that apply. 1. Bleeding that is not controlled after 15 minutes of compression. 2. Second episode occurs within 1 week. 3. Itching sensation after the bleeding stops. 4. Sneezing after the bleeding stops. 5. Massive bleeding.

12. Answer: 1, 2, 5 Page: 131 Feedback 1. Most nosebleeds do not require medical attention; however, the individual should seek medical attention if a nosebleed lasts longer than 15 minutes, or if it occurs after an injury. This may be a sign of a posterior nosebleed, which is more serious. 2. If a second episode occurs within 1 week, a consultation with an ENT specialist is indicated, as it may have underlying conditions associated with the recurrence of symptoms such as sinusitis, septal hematoma/perforation, or mucosal pressure necrosis. 3. Dry air is the most common cause of nosebleeds. Living in a dry climate and using a central heating system can dry out the nasal membranes, which are tissues inside the nose. This dryness causes crusting inside the nose. Crusting may itch or become irritated if the nose is scratched or picked, and it can bleed. 4. Sneezing is an allergic reaction and can also dry out the nasal membranes and cause nosebleeds. Frequent nose blowing is another cause of nosebleeds. 5. ENT consultation is indicated when there is evidence of massive bleeding that cannot be stopped, as there can be a large amount of blood loss.

12. Miss Benton is a 65-year-old female who complains of a painful big toe on the right foot. She asks the nurse practitioner if this is normal for older people. The nurse practitioner explains that this appears to be gout, and gives her what information? Select all that apply. 1. Gout is an inflammatory condition of joints, tissues, and kidneys. 2. Gout is a problem with a protein called purine. 3. Gout is rare in older people. 4. Gout is localized to the big toe. 5. Long-term use of some medications can lead to gout.

12. Answer: 1, 2, 5 Page: 312 Feedback 1. Gout can inflame not only various joints, but periarticular tissues, subcutaneous tissues, and kidneys as well. 2. Gout is the result of malfunctioning metabolism of purine, leading to overproduction or underexcretion of uric acid. 3. Gout is the most prevalent inflammatory condition occurring in older adults. 4. Gout can affect the joints, periarticular tissues, subcutaneous tissues, and kidneys. 5. Gout can be caused by long-term use of some medications, especially diuretic medications.

12. Jessie is a 54-year-old patient with a history of atrophic vaginitis. Which of the following should be included in her patient education? Select all that apply. 1. Use of over-the-counter (OTC) medications for pain and discomfort. 2. Use of water-soluble lubricants. 3. Benefits of regular sexual activity. 4. Identifying and addressing age-related changes. 5. Benefits of sexual-aid stimulators.

12. Answer: 2, 3, 4 Page: 283, 284 Feedback 1. First-line therapies to alleviate atrophic vaginitis symptoms include vaginal lubricants and moisturizers, plus regular sexual activity. 2. The use of water-soluble lubricants may alleviate symptoms. 3. Regular sexual activity can be beneficial in alleviating symptoms. 4. Knowledge of age-related changes is beneficial for understanding and alleviating the symptoms of atrophic vaginitis. 5. The use of a stimulator with sexual activity is not typically discussed with patients diagnosed with atrophic vaginitis.

12. Which of the following statements is true relating the diagnosis of Parkinson's disease (PD)? Select all that apply. 1. The disease should be addressed aggressively using all medications to minimize progression. 2. Drug therapy focuses on correcting the imbalance of dopamine and acetylcholine. 3. Exercise is a contraindication to avoid falls. 4. Depression is common in PD and can contribute to worsening of symptoms. 5. PD is accompanied by symptoms of radiculopathy.

12. Answer: 2, 4 Page: 334, 335 Feedback 1. Always start low and slow with all medications. 2. Drug therapy focuses on correcting the imbalance of dopamine and acetylcholine, which is indicative of PD. 3. Treatment must include exercise from the day of diagnosis because it promotes neuroplasticity. Neuroplasticity is the brain's ability to reorganize and repair throughout a lifetime. 4. Depression is common in PD and can contribute to worsening of symptoms. 5. PD is manifested with symptoms of tremor, rigidity, akinesia, and impaired postural reflexes.

12. One of the things affecting drug distribution is the bioavailability of the drug. Which of the following factors may affect the amount of a drug that reaches the systemic circulation? Select all that apply. 1. Handling of the drug. 2. Dosage of the drug. 3. Solubility of the drug. 4. Route of administration. 5. Generic drug.

12. Answer: 3, 4 Page: 471 Feedback 1. Doctors, nurse practitioners, and other health-care providers are trained in how to give medication safely. Administration of medication requires a thorough understanding of the drug, including proper storage, handling, and disposal. 2. Though dosages affect the amount of a drug in the body, it does not change the bioavailability of the drug. 3. The amount of the drug that reaches the systemic circulation may be increased or decreased, depending on the solubility of the drug. Aqueous solutions are available more quickly than oily ones. 4. Route of administration is important to consider because inhalants and drugs given intravenously and topically are usually more readily available to the body than drugs administered intramuscularly, subcutaneously, orally, or rectally. 5. Generic drugs have no significant difference from brand drugs in rate or extent of absorption when administered at the same dose under similar conditions.

12. Lucy is 65 years old and has been coping with irritable bowel syndrome-diarrhea (IBS-D) for over 10 years. She has tried various treatments with minimal effectiveness. She asks the nurse practitioner if there is anything else that might be helpful to control this diarrhea. The nurse practitioner suggests which of the following? Select any that apply. 1. Antispasmodics 2. Caffeinated beverages 3. Antidiarrheal medications 4. Eluxadoline,75 mcg, twice daily with food 5. Rifaximin

12. Answer: 4, 5 Page: 267 Feedback 1. Antispasmodics are not recommended for treatment of IBS-D in older adults because of the anticholinergic side effects of these medications. 2. Foods that may exacerbate IBS-D should be avoided (e.g., caffeinated beverages). 3. Antidiarrheal medications are not beneficial in treating global IBS-D symptoms. 4. A newer agent recently approved for the treatment of IBS-D is a combination mu-opioid receptor agonist and delta-opioid receptor antagonist called eluxadoline in doses of 100 mcg and 75 mcg to be given twice a day with food. This has been shown effective in improving both diarrhea and abdominal pain symptoms associated with IBS-D. 5. Early studies point to the use of rifaximin, an antibiotic, for consideration of the treatment of IBS without constipation and can be considered for use with patients who have not had success with other treatments.

12. Mr. Dan is 75 years old and suffering from depression. He tells the nurse practitioner that he is tired of feeling so low. He asks if depression is a normal part of aging. The nurse practitioner tells him that depression is which of the following? Select all that apply. 1. Not a normal part of aging. 2. Related to nutritional deficiencies, especially vitamin B12 and vitamin D. 3. Triggered by one life event. 4. Learned helplessness, loss, and bereavement 5. A state of balanced neurotransmitters in the brain.

12. Answers: 1, 2, 3 Page: 451, 452 Feedback 1. Depression is not a normal part of the aging process. Causative factors for depression include physiological influences such as medication side effects, neurological disorders, cardiac disease, neuroendocrine disturbances, and electrolyte and hormonal disturbances. 2. Nutritional deficiencies in vitamin B12 and vitamin D strongly correlate with depression, raising the risk of depression two-fold. 3. Depression can be triggered by a single major life event or a combination of events and stressors. Loss of any kind may contribute to depression and in the older adult may include loss of a spouse/loved one or loss of mobility and independence. 4. Psychosocial and cognitive theories postulate internalized loss with ego dysfunction, learned helplessness, loss and bereavement, and cognitive distortions with negative attitudes and thoughts as contributors to depression. 5. Biological theories point to impaired synthesis, deficiencies, increased uptake, and increased metabolism or breakdown of the neurotransmitters serotonin, norepinephrine, and dopamine as causative factors in depression.

13. Sam is 64 years old and has been feeling weak, lightheaded, and slightly nauseous. He goes to the urgent care center where the nurse practitioner initiates an assessment. Which of the following suggest an arrhythmia? Select all that apply. 1. Chest x-ray showing an increased size of the left atria. 2. History of alcohol drinking. 3. Lack of family history of any acute heart conditions. 4. No history of heart pounding, racing, or skipping beats. 5. Palpitations and syncope.

13. Answer: 1, 2 Page: 161 Feedback 1. An age-related factor associated with tachyarrhythmias is increased left atrial size. 2. Drugs that can cause an arrhythmia include digitalis and other antiarrhythmics, aminophylline, and alcohol. 3. The family history of sudden cardiac death may indicate a predisposing factor for rhythm disorder, such as hypertrophic obstructive cardiomyopathy, congenital prolonged QT syndrome, or the presence of an aberrant conduction pathway. 4. In the history, the patient may describe sensations that accompany abnormal cardiac rhythm, such as pounding, racing, or skipped beats. 5. Older adults are less likely to complain of palpitations and more likely to present with manifestations of heart failure or hypoperfusion (i.e., impaired mental function, dizziness, syncope). There appears to be a peak in syncopal episodes above age 65 years in both men and women.

13. Mark is 60 years old. He reports a swelling in his right side, loss of weight, and a slight discoloration of his skin. The nurse practitioner conducts a history and physical examination and orders which of the following to rule out cancer of the liver? Select all that apply. 1. CBC to identify anemia. 2. Serum alkaline phosphatase, AST, ALT for liver damage. 3. Ultrasound to identify any enlargement. 4. Magnetic resonance imaging (MRI). 5. Radiology for lesions found in the liver.

13. Answer: 1, 2, 3, 4, 5 Page: 269 Feedback 1. Patients with liver disease may have mild anemia and electrolyte disturbances. 2. Serum alkaline phosphatase, AST, ALT, and gamma-glutamyl transpeptidase (GGT) are often abnormal in a nonspecific pattern. 3. Identification of a liver nodule that is smaller than 1 cm should be reimaged with an ultrasound in 3 months. 4. Nodules that are larger than 1 cm should be reimaged with contrast-enhanced MRI. Nodules that demonstrate arterial hypervascularity and venous or delayed phase washout are consistent with hepatocellular carcinoma (HCC). Those nodules that do not demonstrate arterial hypervascularity and venous or delayed phase washout should be reimaged with another contrast-enhanced study such as a CT scan or MRI. 5. Lesions that demonstrate arterial hypervascularity and venous or delayed phase washout are diagnostic for HCC. Lesions that do not demonstrate arterial hypervascularity and venous or delayed phase washout should undergo percutaneous biopsy.

13. Devon visits the urgent care facility for sudden symptoms of pain to his lower extremity. The initial diagnosis is acute arterial ischemia. Aside from pain, which other symptoms may arise from this diagnosis? Select all that apply. 1. Pulselessness 2. Paresthesia 3. Pallor 4. Purulence 5. Paralysis

13. Answer: 1, 2, 3, 5 Page: 219 Feedback 1. Pulselessness is a symptom for acute arterial ischemia. Checking pulses is notoriously unreliable. Arterial Doppler signals should be checked in anyone with suspected acute limb ischemia. 2. Paresthesia is present in over 50% of cases. Sensory nerves are smaller than motor nerves and more sensitive to ischemia so tend to be affected first. 3. Pallor is especially useful in comparison to the opposite limb; it is also useful to check venous filling. Acutely ischemic limbs are classically white rather than blue. Chronic critically ischemic limbs may appear pink due to compensatory vasodilation, the so-called sunset foot. 4. Purulence is not a symptom that may arise from acute arterial ischemia diagnosis. 5. Paralysis is a poor prognostic sign and indicates an element of irreversible ischemia.

13. Mrs. Koslowski, 95 years old, comes to the nurse practitioner with her daughter to discuss her medical regime. She complains that she thinks some of the therapies are doing more harm than good. Mrs. Koslowski wants to discontinue some of the medical orders. The nurse practitioner reviews her medical plan and discusses a new model of care called minimally disruptive medicine (MDM). She explains to the daughter and to Mrs. Koslowski that this model does which of the following? Select all that apply. 1. Recognizes the difference between patient-centered and disease-centered treatment. 2. May suggest limitations on some traditional therapies for the frail patient with MCCs. 3. Revises clinical practice guidelines to consider minimally disruptive medicine. 4. Recognizes that older adults have improved results when treated with disease-specific orders. 5. Has clinicians use approaches that include patient values so that better decisions are made.

13. Answer: 1, 2, 3, 5 Page: 479 Feedback 1. MDM acknowledges the limitations of disease-centered treatment recommendations for patients with MCCs. 2. MDM acknowledges the limitations of disease-centered treatment recommendations for patients with MCCs. 3. It is essential to evaluate the MDM guidelines to determine if they are appropriate for frail, older adults with MCCs. 4. Older adults with MCCs have poorer outcomes when treated according to disease-specific guidelines. 5. To improve decision making, clinicians require unique data about the frail older adults and alternative approaches that include patient and family values, communication and coordination with specialty providers and other members of the team, and a restructured reimbursement system.

13. Breast cancer is more prevalent in the female gender, with a median age of 61 years. Which of the following are modifiable risk factors that may contribute to the diagnosis of breast cancer? Select all that apply. 1. Postmenopausal obesity 2. Alcohol intake 3. Early menarche 4. Menopause hormone therapy 5. Ethnicity

13. Answer: 1, 2, 4 Page: 285 Feedback 1. Addressing postmenopausal obesity by encouraging healthy eating and exercise can reduce the risk factor of breast cancer. 2. Minimizing alcohol intake can help reduce the risk of developing breast cancer. 3. Early menarche is a nonmodifiable risk factor of breast cancer. 4. Menopause hormone therapy is a modifiable risk factor for breast cancer. Patients need to be informed about the risk of using combined estrogen and progestin menopause hormone therapy. 5. Ethnicity is a nonmodifiable risk factor, along with age and family history.

13. Hearing loss is a decreased ability or inability to hear. The loss may involve the external, middle, or inner ear and can be unilateral or bilateral. Which of the following etiologies may result in hearing loss? Select all that apply. 1. Sensorineural 2. Vascular occlusive disease 3. Conductive 4. Cerebral neoplasia 5. Aphasia

13. Answer: 1, 3 Page: 136 Feedback 1. A lesion in the organ of Corti or in the central pathways, including the cranial nerve VIII and auditory cortex, causes sensorineural hearing loss. Presbycusis, noise-induced hearing loss, and ototoxic drug-related hearing loss all are sensorineural. 2. Vascular occlusive disease is predominantly a disease of the lower extremities and not an etiology of hearing loss. 3. Conductive hearing loss is caused by a lesion involving the outer and middle ear to the level of the oval window. Various structural abnormalities, cerumen impaction, perforation of the tympanic membrane, middle ear fluid, damage to the ossicles from trauma or infection, middle ear tumors, temporal bone fractures, injuries related to trauma, and congenital problems are some of the causes. 4. Cerebral neoplasia is a brain tumor that may affect body organs; however, it is not a primary etiology that leads to hearing loss. 5. Aphasia is a loss of ability to understand or express speech caused by brain damage.

4. Which of the following conditions presents nonproliferating lesions and does not affect future cancer risk? 1. Atypical lobular hyperplasia 2. Fat necrosis 3. Radial scar 4. Papilloma

4. Answer: 2 Page: 287 Feedback 1. Atypical lobular hyperplasia is a proliferating lesion with atypia and has an increase in cancer risk of four to five times. 2. Fat necrosis is a nonproliferating lesion and does not affect future cancer risk. 3. Radial scar is a proliferating lesion without atypia and has an increase in cancer risk of approximately one to two times. 4. Papilloma is a proliferating lesion without atypia and has an increase in cancer risk of approximately one to two times.

13. It is important to know the pharmacodynamic influences of the drug. Which of the following statements are true regarding pharmacodynamics? Select all that apply. 1. A drug's pharmacodynamics describes the effect at the site of action. 2. Pharmacodynamics is concerned with the movement of drugs within the body. 3. A drug's pharmacodynamics describes the time and intensity of the drug effect. 4. Pharmacodynamics describes the rate and pathways by which drugs are eliminated from the body by metabolism and excretion. 5. Pharmacodynamics' fundamental concept is drug clearance.

13. Answer: 1, 3 Page: 471 Feedback 1. There is a general trend of assuming greater pharmacodynamic sensitivity in the older adult, however, this sensitivity is not universal. 2. Pharmacokinetics is concerned with the movement of drugs within the body. 3. An example of this is the difference in effect at the site on the older adult. Older adults tend to exhibit enhanced responses to drugs affecting the central nervous system (e.g., benzodiazepines), and this is attributed to greater tissue sensitivity caused by aging. 4. Pharmacokinetics describes the rate and pathways by which drugs are eliminated from the body. 5. A fundamental concept in pharmacokinetics is drug clearance; that is, elimination of drugs from the body (analogous to the concept of creatinine clearance).

13. Peripheral neuropathy is a disorder of the peripheral nervous system resulting in sensory, motor, and autonomic dysfunction. Which of the following conditions are the treatable causes of peripheral neuropathy? Select all that apply. 1. Radiculopathy 2. Alcohol use 3. Diabetes mellitus 4. Hypnic jerks 5. TBI

13. Answer: 2, 3 Page: 337 Feedback 1. Radiculopathy is caused by compression of the nerves as they exit the spine and may be a differential diagnosis for RLS. 2. Moderate to heavy use of alcohol causes peripheral neuropathy due to the adverse neurotoxic effect and associated nutritional deficiencies. 3. Diabetes mellitus frequently affects the peripheral nervous system and is the most common cause of peripheral neuropathy. 4. Hypnic jerks are sudden jerking motions of the legs or whole body that occur when falling asleep and may be a differential diagnosis for RLS. 5. TBI is a complex injury with a broad spectrum of symptoms and disabilities, but rarely causes peripheral neuropathy.

13. Annie is 89 years old and appears apathetic, unwilling to eat and drink, and shows functional decline. The nurse practitioner discusses with the family the possibility that Annie is experiencing a failure to thrive (FTT). The family asks how this happened. The nurse practitioner replies with which of the following statements? Select all that apply. 1. Adult FTT is a part of aging. 2. FTT can be associated with increased infections and diminished immunity. 3. FTT patients have often experienced multiple hospitalizations and several comorbidities. 4. FTT is complex, with many components. 5. The goal of early assessment is to prevent a decline in health.

13. Answer: 2, 3, 4 Page: 377 Feedback 1. Adult FTT is not considered a normal part of aging. 2. FTT is associated with increased infections and diminished cell-mediated immunity. 3. This population of gravely ill and impaired older adult patients has experienced multiple hospitalizations, has multiple diagnoses, and can be viewed as a paradigm of the very sick and frail older adults. 4. It can be concluded that patients given a diagnosis of FTT present with a complex picture of multifaceted problems across a spectrum of physiological ailments, psychological deficits, and social and environmental needs. 5. The goal of early assessment is to identify the precise needs of the patient in determining appropriate intervention strategies, providing necessary supports, and optimizing rehabilitation.

14. There is no specific diagnostic test for hordeolum or chalazion unless it becomes recurrent. Which of the following may be a differential diagnosis for these conditions? Select all that apply. 1. Orbital cellulitis 2. Contact dermatitis 3. Acute dacryocystitis 4. Strabismus 5. Amblyopia

14. Answer: 1, 2, 3 Page: 138 Feedback 1. Orbital cellulitis is an infection of the soft tissues of the orbit posterior to the orbital septum. It is most often caused by extension of infection from adjacent sinuses, or by direct infection accompanying local trauma or contiguous spread of infection from the face or teeth. 2. Contact dermatitis is a red, itchy rash caused by direct contact with a substance or an allergic reaction to it. The rash isn't contagious or life threatening, but it can be very uncomfortable. 3. Acute dacryocystitis is an infection of the tear (lacrimal) sac usually due to a blockage in the tear (nasolacrimal) duct. The patient presents with pain, redness, and edema around the lacrimal sac. 4. Strabismus is a visual problem in which the eyes are not aligned properly and point in different directions. 5. Amblyopia is the medical term used when the vision in one of the eyes is reduced because the eye and the brain are not working together properly. The eye itself looks normal, but it is not being used normally because the brain is favoring the other eye. This condition is also sometimes called lazy eye. Many people make the mistake of saying that a person who has a crossed or turned eye (strabismus) has a lazy eye, but lazy eye (amblyopia) and strabismus are not the same condition. Strabismus can cause amblyopia.

14. Bob is 85 years old and is suspected of experiencing adult FTT. His family asks the nurse practitioner to evaluate what exactly is going on with him. The nurse practitioner knows to order which of the following? Select all that apply. 1. Complete blood count (CBC), serum albumin level, and thyroid-stimulating hormone (TSH). 2. Cholesterol levels and comprehensive metabolic panel. 3. Blood cultures. 4. A nutritional assessment. 5. Depression screening with Beck's PP depression scale.

14. Answer: 1, 2, 3, 4 Page: 377 Feedback 1. Consider ordering a CBC, serum albumin level, and TSH initially in patients presenting with signs of malnutrition over time. 2. Consider ordering cholesterol level and comprehensive metabolic panel initially in patients presenting with signs of malnutrition over time. 3. For patients with suspected infection or inflammation associated with malnutrition, consider blood cultures. 4. Conduct a nutritional assessment with the Mini Nutritional Assessment. A score of 17 or less indicates malnutrition. 5. Depression screening with the Geriatric Depression Scale should be included. A score of 5 to 9 is suggestive of depression and 10 or more is significant for depression.

14. Ms. Alma, 69 years old, seeks medical advice because she is having trouble breathing. She is experiencing dyspnea, chronic cough with sputum production, decreased activity tolerance, and wheezing. She states that she had not been near anyone with any upper respiratory infections and wonders why this is happening. The nurse practitioner asks which of the following questions? Select all that apply. 1. Have you ever smoked cigarettes? 2. Have you ever worked in a factory or foundry? 3. How long have you experienced these symptoms? 4. Do you have any past history of respiratory difficulties? 5. Have your symptoms increased as you've gotten older?

14. Answer: 1, 2, 3, 4, 5 Page: 164, 165 Feedback 1. The most significant risk factor for chronic obstructive pulmonary disease (COPD) is cigarette smoking. 2. Occupational/industrial gases or fumes, and indoor/outdoor air pollution are all contributing risk factors for COPD. 3. Advancing age is a factor, with symptoms often presenting after the age of 40 to 50 years old. Individuals aged 65 to 74 years and 75 years and older have the highest incidence of COPD. 4. Inadequate lung growth and development during childhood contribute to risk factors for COPD. 5. Advancing age is a risk factor for COPD, although the evidence is unclear if healthy aging leads to COPD or if it is a result of the cumulative sum of exposures throughout life.

14. The nurse practitioner is asked to order medication for pain and disease control for a 60-year-old patient who has suffered with RA for years. Which is the most common treatment? Select all that apply. 1. Corticosteroids 2. NSAIDs 3. Analgesics 4. Biological disease-modifying antirheumatic drugs (DMARDs) 5. Tumor necrosis factor (TNF) receptor antagonists

14. Answer: 1, 2, 3, 4, 5 Page: 324 Feedback 1. The drug management of RA is one of symptom and disease control. Symptom control includes corticosteroid treatment. 2. The drug management of RA is one of symptom and disease control. Symptom control includes NSAIDs. 3. The drug management of RA is one of symptom and disease control. Symptom control includes analgesics. 4. Biological DMARDs are newer therapies that target specific cytokines of the inflammatory response; they are administered subcutaneously, intravenously, and by mouth. 5. Some of the TNF inhibitor biological agents approved for use are etanercept, adalimumab, infliximab, certolizumab, and golimumab. Other biological therapies commonly used are rituximab and abatacept.

14. The family of Mr. Moore, 90 years old, meets with the nurse practitioner to discuss a change in the care of Mr. Moore. He appears to be less comfortable, more irritable, and has less appetite. He has diabetes, chronic obstructive pulmonary disease (COPD), kidney disease, and chronic colitis. The family and practitioner agree that Mr. Moore is not near end of life, and yet there is not an apparent cure for his conditions. The nurse practitioner discusses palliative care for Mr. Moore. Why might palliative care be a good option? Select all that apply. 1. It will provide relief from his symptoms. 2. It can manage symptoms until treatment is found to provide a cure. 3. Nearly half of Medicare patients can qualify for palliative care when they have two to four chronic care conditions. 4. The health-care providers are aware of updated treatment that can alleviate his symptoms. 5. It may provide a better quality of life and mood enhancement.

14. Answer: 1, 2, 5 Page: 481 Feedback 1. Palliative care focuses on providing relief from the symptoms and the stress of the severe illness with the goal of improving the quality of life for both the patient and the family. 2. Palliative care is used to alleviate symptoms—either in combination with or instead of curative treatment. 3. Palliative care should be considered for close to half (45%) of all Medicare beneficiaries who have four or more chronic conditions for which palliative care services may be clinically indicated. 4. Palliative care can serve as an intervention to manage chronic illness, not just at the end of life, but also in the early phases of illness to support patients and caregivers. 5. Patients who receive early palliative care show significant improvements in quality of life and mood, and may survive longer.

14. Thom is 60 years old and has been treated for nonalcoholic fatty liver disease (NAFLD) for over 10 years. He still suffers from fatigue, abdominal pain, and an enlarged liver. The nurse practitioner discusses which of the following home-care plans to assist him in adjusting to his disease? Select all that apply. 1. Monitor weight and stay within optimal weight limits. 2. Avoid anti-lipid medications, such as atorvastatin. 3. Obtain a hepatitis A and B vaccine, if not immune. 4. May drink alcohol socially, but not daily. 5. If obese, set a goal for rapid weight loss.

14. Answer: 1, 3 Page: 272 Feedback 1. Overweight or obese patients with NAFLD should consider a weight-loss program. It has been shown that weight loss and exercise reduce liver enzyme levels and steatosis. 2. Treatment for hyperlipidemia (atorvastatin, gemfibrozil) has been shown to improve liver enzymes and liver steatosis. 3. The hepatitis A and B vaccine should be given to patients without serological evidence of immunity. 4. Patients with NAFLD should avoid alcohol consumption. 5. Weight loss should not exceed 1 to 2 pounds a week. Patients should avoid rapid weight loss.

14. A patient sees the nurse practitioner for a post-hospitalization follow-up and presents new complaints with change in functional status. Which of the following should the nurse practitioner do? Select all that apply. 1. Evaluate for ADR. 2. Prescribe patient medications for the new symptoms. 3. Consult current resources for precautions before prescribing. 4. Prescribe antibiotics for the patient. 5. Leave as status quo.

14. Answer: 1, 3 Page: 472 Feedback 1. Whenever a patient presents with a new chief complaint, a change in cognitive or functional status, or a newly recognized health problem, evaluate for an ADR as the cause before prescribing. 2. Prescribing patient medications for new symptoms without evaluating for an ADR may result in the use of unnecessary drugs and increase the chance for ADRs. 3. Several categories of drugs can be problematic in older adults. Prescribers should consult current resources for precautions before prescribing. 4. Prescribing antibiotics for the patient without evaluating for an ADR may result in unnecessary drugs being added and increasing the chance for ADRs. 5. Leaving the situation status quo is never advisable, as symptoms may worsen if not addressed. A patient must be evaluated and possible ADRs identified.

14. Which of the following are the main laboratory clinical manifestations of cystitis? Select all that apply. 1. White blood cell (WBC) count 2. Bacteriuria 3. Pyuria 4. Fungi 5. E. coli

14. Answer: 2, 3 Page: 289 Feedback 1. WBCs, also called leukocytes or leucocytes, are the cells of the immune system that are involved in protecting the body against both infectious disease and foreign invaders. Increased or decreased WBC count can indicate infection but is not the main laboratory clinical manifestation of cystitis. 2. Bacteriuria is the presence of bacteria in urine and is one of the main laboratory clinical manifestations of cystitis. 3. Pyuria is the presence of pus in the urine and is one of the main laboratory clinical manifestations of cystitis. 4. The presence of fungi does not lead to a diagnosis of cystitis. 5. The presence of E. coli is not a main laboratory clinical manifestation of cystitis. E. coli is a bacterium found in the environment, foods, and intestines of people and animals.

14. Treatment to slow the progression of PAD should include which of the following? Select all that apply. 1. Immobilization 2. Pharmacological 3. Surgical 4. Conservative 5. Heat

14. Answer: 2, 3, 4 Page: 220 Feedback 1. Immobilization does not slow the progression of PAD. 2. Pharmacological treatment is provided with conservative treatment and used to slow the progression of PAD. 3. Surgical treatment of PAD involves revascularization of the affected extremity and is used to slow the progression of PAD. 4. Conservative treatment involves modification of risk factors, including smoking. It also includes blood pressure and diet implementations to slow the progression of PAD. 5. Heat is not a treatment used to slow the progression of PAD.

14. Pain is an unpleasant sensory and emotional experience associated with actual or potential tissue damage. Which of following is true regarding pain? Select all that apply. 1. Pain is mostly perceived by the individual and often not real. 2. Pain is always treatable. 3. The patient's own report is the best indicator of pain. 4. Pain is a common end-of-life symptom in the older adult. 5. Pain is not an indicator of an illness.

14. Answer: 3, 4 Page: 490 Feedback 1. Both real and imagined pain will register very similarly in the brain. According to studies, pain-related areas of the brain can be activated without any injury. 2. Pain may not always be treatable. Unrelieved pain is associated with functional impairment, falls, mood disorders, sleep and appetite disturbances, and decreased socialization. Pain escalation may indicate advancing disease. 3. Pain is whatever the experiencing person says it is, whenever they are experiencing it. The patient's own report is the best indicator of pain. 4. Pain is a common end-of-life symptom in the older adult. Most older adults can effectively communicate pain, but a more detailed assessment of pain can be challenging. 5. Pain is a signal in the nervous system that something may be wrong. It is an unpleasant feeling, such as a prick, tingle, sting, burn, or ache. Pain may be sharp or dull. It may come and go, or it may be constant. Pain can be helpful in diagnosing a problem and may be caused by an illness or injury.

14. Mrs. Smith is 90 years old and has a daytime caregiver. Her daughter stays with her during the evening and nighttime. The daughter suspects her mother is being abused. Which of the following statements would confirm this for the nurse practitioner? Select all that apply. 1. My mother complains of physical pain and has bruises on her arm. 2. My mother is losing weight and is always hungry in the evening. 3. My mother has lost her wallet and funds are going out of her bank account. 4. My mother has been withdrawing from others. 5. My mother has burn marks on her body in the shape of a cigarette.

14. Answers: 1, 2, 4, 5 Page: 457 Feedback 1. Unusual bruising in areas that are not usually thought to be accidental can raise suspicion of possible abuse. Any bruising around breasts or genital area and unexplained sexually transmitted diseases might indicate sexual abuse. 2. Neglect is a type of abuse due to failing to provide food, shelter, health care, or protection for a vulnerable older adult. 3. The National Institute of Justice warns sudden changes in finances, unexplained changes to wills or trusts, unexplained bank withdrawals, and loss of property may indicate a problem of exploitation of an older adult. 4. Emotional abuse can be suspected if a patient withdraws from his or her normal activities or has an unusual change in his or her level of alertness or any other change in behavior that has not been previously observed. 5. Physical signs include slap marks and unexplained burns or blisters, especially circular, as from a cigarette.

15. Sharon is a 70-year-old patient with venous ulcers to her right lower extremity. She has a history of multiple pregnancies and surgeries, and has a sedentary lifestyle. Which of the following may be possible contributing factors to her diagnosis given her history? Select all that apply. 1. Obesity 2. Reduced mobility 3. History of breast cancer 4. Degenerative disease 5. Parkinsonism

15. Answer: 1, 2 Page: 221 Feedback 1. Obesity, or being overweight, can increase the risk of developing a venous leg ulcer, as this increases the pressure in the leg veins. 2. Decreased or reduced mobility in the ankle contributes to the diagnosis of venous leg ulcer. 3. History of breast cancer is not a contributing factor to venous leg ulcer. 4. Degenerative disease is not a contributing factor to venous leg ulcer. 5. Parkinsonism is not a contributing factor to venous leg ulcer.

15. Mrs. Owen, 70 years old, is being discharged from the rehabilitation center to go home. Her RA pain is under currently control. What does the nurse practitioner include in the follow-up instructions? Select all that apply. 1. Get plenty of rest during the day. 2. Exercise and move throughout the day. 3. Learn about the side effects of your medication. 4. Read literature for antidotes that cure RA. 5. Keep your regular health-care visits, as there are no support organizations for RA.

15. Answer: 1, 2, 3 Page: 325 Feedback 1. Patients should be taught the importance of incorporating periods of rest into their daily lives. 2. Patients should be taught the importance of incorporating periods of exercise into their daily lives. 3. Medication education is important in RA because of the potential for side effects associated with a complicated drug regimen. 4. Because there is no cure for RA, patients must be skeptical about antidotes promised for RA and check with their health-care providers if they have any concerns about treatments. 5. Patients can contact the Arthritis Foundation (1-800-282-7800) for information; a booklet, Overcoming Rheumatoid Arthritis, is available.

15. Dan is 70 years old and has been having frequent bouts of kidney stones. They are painful, and he has had one procedure done for nephrolithiasis. Dan asks the nurse practitioner if there is anything he can do to prevent these. He says his brother also has them, so he wonders if it is just familial. How does the nurse practitioner respond? Select all that apply. 1. Increase the amount of fluids you drink daily. 2. Avoid eating foods such as dairy, spinach, and nuts. 3. Some medications can contribute to stone formation. 4. Kidney stones are not familial. 5. Report signs of possible urinary tract infections to a physician for treatment.

15. Answer: 1, 2, 3, 5 Page: 270 Feedback 1. A major contributing factor to the development of calculi is decreased fluid intake leading to a high concentration in urine. 2. Certain food substances that augment the formation of kidney stones include dairy products, chocolate, green leafy vegetables (calcium oxalate stones), and eggs, fish, poultry, peanuts, and wheat (cystine stones). 3. Certain medications, such as triamterene, indinavir, acetazolamide, acyclovir, and sulfa can contribute to the development of nephrolithiasis. 4. Family history has also been found to contribute to stone formation, as well as increased oxalate absorption. 5. Chronic urinary tract infections may be precursors to struvite stone formation.

15. Mr. Waterson is 76 years old and involved in a weight-loss program. He weighs 250 pounds and is 5 feet and 5 inches tall. He asks his nurse practitioner what he should do for exercise to enhance his modified eating style. The nurse practitioner advises him with which of the following statements? Select all that apply. 1. Anything you do is better than nothing. 2. Try moderate walking for 2 ½ hours a week. 3. Try to increase your moderate activity to 600 minutes a week. 4. Perform muscle-strengthening activities that are moderate or high intensity on 4 or more days a week. 5. Do exercises that improve balance, which may prevent falls.

15. Answer: 1, 2, 5 Page: 382 Feedback 1. All older adults should avoid inactivity. Some physical activity is better than none, and older adults who participate in any amount of physical activity gain some health benefits. 2. For substantial health benefits, older adults should do at least 150 minutes (2 hours and 30 minutes) a week of moderate-intensity, or 75 minutes (1 hour and 15 minutes) a week of vigorous-intensity aerobic physical activity, or an equivalent combination of aerobic activity that is of moderate-to-vigorous intensity. Aerobic activity should be performed in episodes of at least 10 minutes, and preferably, it should be spread throughout the week. 3. For additional and more extensive health benefits, older adults should increase their aerobic physical activity to 300 minutes (5 hours) a week of moderate-intensity, or 150 minutes a week of vigorous-intensity aerobic physical activity, or an equivalent combination of aerobic activity that is of moderate-to-vigorous intensity. 4. Older adults should also do muscle strengthening activities that are moderate or high intensity and involve all major muscle groups on 2 or more days a week, as these activities provide additional health benefits. 5. Older adults should do exercises that maintain or improve balance if they are at risk of falling.

15. Which of the following is true regarding STOPP/START criteria for prescribing medications? Select all that apply. 1. Lists potential alternative drugs that can safely be used in older adults. 2. Valuable tool for prescribers, but they are not meant to be mandates for prescribing. 3. Screening tool to alert doctors to correct treatment. 4. Screening tool of older persons' potentially inappropriate prescriptions. 5. It is the Beers criteria that mandates prescribing.

15. Answer: 2, 3, 4 Page: 472 Feedback 1. The Beers criteria lists potential alternative drugs that can be safely used in older adults. 2. Clinical judgment, knowledge of the patient who has the disease, clinician and patient shared goals, risk-benefit considerations, and quality of life all factor into decision making for individual patients. 3. START is a screening tool to alert doctors to right treatment and is used in conjunction with STOPP. 4. STOPP is a screening tool of older persons' PIMs and is used in conjunction with START. 5. The Beers criteria are valuable tools for prescribers, but they are not meant to be mandates for prescribing. Clinical judgment, knowledge of the patient who has the disease, clinician and patient shared goals, risk-benefit considerations, and quality of life all factor into decision making for individual patients.

15. Mr. Hupp, 84 years old, lost his wife 2 months ago. He goes to his nurse practitioner and complains of the same symptoms that his wife had before she died. Possible treatment options available to the nurse practitioner include which of the following? Select all that apply. 1. Educate him on normal signs of grieving. 2. Take time to remember his wife and discuss memories. 3. Provide emotional support. 4. Monitor current chronic medical conditions. 5. Encourage him to stay quiet at home and decrease his physical activity.

15. Answers: 1, 2, 3, 4 Page: 459 Feedback 1. Sleep disruptions are common in the first 2 weeks; education on sleep hygiene and assessing for pre-existing conditions such as obstructive sleep apnea may be necessary. Referral to a bereavement support group may help some individuals, but not all. 2. Reminiscence is helpful to many. 3. Provide emotional support, allowing the older adult to express feelings. 4. Those with pre-existing chronic medical conditions, particularly cardiac, will need to be monitored for adverse events; studies have demonstrated the occurrence of acute coronary syndrome (ACS/MI) after the death of a significant person. 5. Encourage patients to return to their normal routine as soon as possible. Daily physical exercise can help patients cope with the depression that accompanies grief.

2. Which of the following is related to the incidence of gastric acidity declining with age because of decreased intestinal blood flow? 1. Drug metabolism 2. Drug distribution 3. Drug absorption 4. Drug elimination

2. Answer: 3 Page: Feedback 1. Drug metabolism is affected with decreased liver size and blood flow, which can result in decreased first-pass metabolism. Drug activity for some medications is prolonged because drugs are metabolized and eliminated more slowly. 2. Drug distribution is affected by aging, particularly in individuals of smaller body size, those who have decreased body water, and those with higher body fat. 3. Drug absorption is affected when gastric acidity declines with age because of decreased intestinal blood flow and fewer absorbing cells in the gastrointestinal (GI) tract. It is also affected with the presence of food and other drugs in the stomach at the same time. 4. Drug elimination is affected by the changes in the kidney functions.

2. A patient with renal disease has blood work drawn, and the results show an increase in serum creatinine. The nurse practitioner needs to know which of the following laboratory values before ordering medications? 1. CBC 2. Culture and sensitivity of the urine 3. Creatinine clearance 4. Uric acid levels

2. Answer: 3 Page: 3 Feedback 1. A CBC will not evaluate kidney function for a patient with renal disease. 2. A culture and sensitivity test reflects the presence of an infection and the antibiotic to which the organism is sensitive. 3. The calculation of creatinine clearance provides an estimation of renal function. 4. Uric acid level is elevated in the presence of gout.

2. While conducting a thorough examination of the musculoskeletal system, the patient asks why such detail is necessary. Which of the following is the nurse practitioner's best response? 1. I don't want to miss any signs of something serious. 2. The purpose of conducting this examination is to regulate your medications. 3. I am looking for symptoms that form a pattern. 4. This is routine for all patients over 65 years of age.

2. Answer: 3 Page: 305 Feedback 1. The nurse practitioner is seeking symptoms that form a pattern, and in the thorough examination to do that, she will find any abnormalities or areas of concern. 2. Medications will be taken into consideration after examination findings. 3. A goal of the nurse practitioner in gathering information from the older adult is to try and determine if there is a pattern of symptoms. 4. In an adult over 65 years old who is not presenting any musculoskeletal issues, this type of examination may not be necessary.

5. Mr. Angulo is an 89-year-old patient who was recently an inpatient for pneumonia and was bedridden for 1 week. He is now presenting dysuria, suprapubic tenderness accompanied by fever, and increased confusion. Based on his recent history, which of the following conditions may be developing? 1. Cystitis 2. Distended bladder 3. Prostate stones 4.Subacute prostatitis

5. Answer: 1 Page: 289 Feedback 1. Symptoms are indicative of cystitis, and because the patient was bedridden and may have had placement of a catheter, this increases the likelihood. 2. Distended bladder is less likely to be the condition the patient is developing, given the history and symptoms. 3. Prostate stones are less likely to be the condition the patient is developing, given the history and symptoms. 4. Subacute prostatitis is less likely to be the condition the patient is developing, given the history and symptoms.

2. Mr. Jones is 70 years old and sees the nurse practitioner for a yearly checkup. He states that he needs to urinate a lot and seems to always be thirsty. He also finds himself gaining some weight. In the process of the examination, the nurse practitioner detects that he is vague in his answers. Which of the following may be the reason? 1. Mr. Jones is cold and bundled in a sweater. 2. There is no family history of diabetes. 3. The symptoms seem to have appeared slowly. 4. He does not want to admit to eating too much.

2. Answer: 3 Page: 361 Feedback 1. Explore with the patient any difficulty with temperature regulation, changes in skin texture, or distribution of body hair. 2. Reevaluate the patient's family history for endocrine and metabolic disease. 3. Because changes in the endocrine system may appear subtle to the older person or atypical as compared to younger patients, it may be difficult to pinpoint the onset of the presentation. 4. Review with the patient any episodes of unexplained weight loss or gain, new or increased fatigue, weakness, malaise, and recent infections.

6. According to studies, which of the following is true regarding cerebrovascular disease? 1. It accounts for 20% to 50% of seizures in older adults. 2. It accounts for 10% of seizures in older adults. 3. Sixty percent of patients with stroke are at risk of developing seizures within 5 years. 4. Transient ischemic attacks have a relatively high incidence of seizures.

6. Answer: 1 Page: 347 Feedback 1. Cerebrovascular disease accounts for 20% to 50% of seizures in older adults. 2. Cerebrovascular disease accounts for 20% to 50% of seizures in older adults. 3. Cerebrovascular disease accounts for 20% to 50% of seizures in older adults. 4. Transient ischemic attacks have a relatively low incidence of seizures.

2. Which of the following medical specialists deals with the diagnosing of visual impairments and conditions? 1. Ophthalmologist 2. Optometrist 3. Otolaryngologists 4. Podiatrist

2. Answer: 1 Page: 129 Feedback 1. An ophthalmologist is a medical doctor who specializes in eye and vision care. An ophthalmologist diagnoses and treats all eye diseases, performs eye surgery, and prescribes and fits eyeglasses and contact lenses to correct vision problems. 2. An optometrist is an eye doctor who has earned the doctor of optometry (OD) degree. Optometrists examine eyes for both vision and health problems, and correct refractive errors by prescribing eyeglasses and contact lenses. 3. Otolaryngologists are physicians trained in the medical and surgical management, and treatment of patients with diseases and disorders of the ear, nose, and throat, and related structures of the head and neck. 4. A podiatrist is a medical doctor devoted to the study and medical treatment of disorders of the foot, ankle, and lower extremities.

2. The nurse practitioner conducts a comprehensive geriatric assessment of the older individual because: 1. Health of older people is affected by several factors. 2. A focused assessment will provide depth of specific information. 3. A comprehensive assessment will provide more subjective information. 4. A comprehensive assessment will result in a list of current and prior medications taken.

2. Answer: 1 Page: 26 Feedback 1. A comprehensive approach to geriatric assessment is recommended because the physical health of the older adult is inextricably related to functional ability, psychosocial health, and a safe and enabling environment. 2. A focused assessment will provide a limited amount of information about the whole person. 3. A comprehensive assessment will provide both objective and subjective information. 4. A list of medications is only one component of a comprehensive assessment.

2. Mrs. Smith is 90 years old and a frail, older woman with functional limitations, cognitive impairment, and lack of family or social support. Which type of care is best for the patient? 1. Hospice care 2. Palliative care 3. Skilled nursing care 4. Physical therapy care

2. Answer: 2 Page: 485 Feedback 1. Hospice is a program of care designed for the last 6 months of a person's life and this scenario does not indicate that. 2. Palliative care can include individuals whose health if failing and who have no social support. 3. Skilled nursing provides the palliative and other kinds of care. 4. Physical therapy provides the palliative and other kinds of care.

2. The nurse practitioner is conducting patient rounds in a long-term care facility. As she talks with Mrs. Jones, she notices that her arms and elbows are excoriated and the skin is shearing. The nurse practitioner explains to the staff that Mrs. Jones needs frequent assessment of her skin and protection provided to prevent skin breakdown because: 1. Her lack of activity causes the skin to tear. 2. Fat has redistributed to the abdomen and thighs, leaving bony surfaces in areas such as the face, hands, and sacrum. This can result in injury. 3. She has lost weight and is in jeopardy of falling. 4. She picks at herself and causes skin breakdown.

2. Answer: 2 Page: 96 Feedback 1. Lack of activity alone does not cause skin breakdown. 2. Fat is redistributed to the abdomen and thighs, leaving bony surfaces, such as the face, hands, and sacrum, exposed to potential injury, especially skin tears from shearing, friction forces and pressure ulcer development. 3. Although losing weight may be a risk factor for falling, it is not directly related to skin breakdown. 4. There is no evidence that she is picking at herself, as there is nothing reported anywhere else on her arms.

2. Mr. Slobinsky, 80 years old, is unable to leave his home due to mobility problems. The nurse practitioner conducts a social support assessment because Mr. Slobinsky: 1. May not want to see anyone. 2. Is not missing the cultural traditions of community activities. 3. Has made frequent appointments for consultation with the practitioner. 4. Looks forward to sharing his loneliness with the nurse practitioner.

2. Answer: 3 Page: 428 Feedback 1. Older adults who are alone and lonely may also be depressed. 2. Specific to older adults, ethnic traditions and expectations are factors that should be addressed in the psychosocial assessment. 3. A positive correlation exists between the need for social supports and the use of health-care services by older adults. 4. Older adults may be reluctant to reveal social or emotional concerns and, in some cultures, may feel that it is unacceptable to share personal problems with outsiders. However, they may feel comfortable discussing a physical manifestation, such as pain or sleeplessness, with a health-care provider.

2. Which of the following conditions is signaled by symptoms of swelling of the affected body part, usually the limb, because of impaired flow of lymph fluid? 1. Abdominal aortic aneurysm (AAA) 2. Venous ulcers 3. Peripheral vascular disease (PVD) 4. Lymphedema

2. Answer: 4 Page: 218 Feedback 1. AAA presents the signal symptoms of persistent or intermittent pain in the middle or lower abdomen, radiating to the lower back. 2. Venous leg ulcers present signal symptoms of swelling that subside with elevation of lower extremities, eczematous skin changes, dull ache in lower extremities, and presence of varicosities. 3. PVD is a disease that alters blood flow to or from the extremities and vital organs other than the heart. It presents signal symptoms of pain, intermittent claudication of the feet, and tissue loss in affected leg/arm. 4. Lymphedema presents the signal symptoms of swelling of the affected body part, usually the limb, because of impaired flow of lymph fluid.

6. The focus of advanced nurse practitioners is primary care. This is defined as activities that focus on: 1. Preventing the occurrence of a disease or condition. 2. Treating an illness after symptoms appear. 3. Enabling the patient to recover and convalesce. 4. Enriching the patient's appreciation of life.

6. Answer: 1 Page: 7 Feedback 1. Primary preventive strategies focus on immunization, well-checks, and other health maintenance activities. 2. Secondary preventive strategies focus on prevention and treatment of illness. 3. Tertiary preventive strategies focus on rehabilitation. 4. Nurse practitioners focus on the wellness-illness continuum.

3. Mrs. Wilbur reports the presence of pain in her lower and upper back. The nurse practitioner responds by doing which of the following? 1. Determining where the pain is coming from. 2. Ignoring any history of injury. 3. Ruling out the presence of anything inflammatory. 4. Determining that this pain does not affect activities of daily living (ADLs), as the patient came in independently.

3. Answer: 1 Page: 305 Feedback 1. Initially, the nurse practitioner will need to determine if the presenting symptoms arise from the joints, tendons, muscles, or periarticular structures, such as bursae. 2. Past trauma may now be manifesting itself as an articular degeneration. Because patients who have had a structural deformity or amputation typically place excessive strain on the joints for years, as older individuals they may now experience degeneration of the bone and surrounding musculature. 3. Inflammatory and noninflammatory conditions can coexist in patients. 4. It is important to ask questions and determine how the pain and stiffness are affecting function and quality of life, regardless of how it appears.

6. Mary Jane is seeing a nurse practitioner for a CGA and asks which parts of her life will be examined. The nurse practitioner says the domains of the CGA include: 1. Internal organ health. 2. Social habits. 3. Physical and psychological health, as well as other facets of life. 4. History of family illness.

6. Answer: 3 Page: 27 Feedback 1. The CGA includes more than internal health. 2. Learning about social habits is not the only component in determining health. 3. Domains of CGA include physical health, psychological health, socio-environmental supports, mobility, functional status, and a measure of the quality of life. 4. Although family health history provides information about genetics, the CGA is complete and includes current health status in several domains.

6. For individuals over 65 years old, the most common morbidities are related to: 1. Heart disease, arthritis 2. Respiratory problems, cancer 3. Diabetes, stroke 4. All of these are common morbidities.

6. Answer: 4 Page: 5 Feedback 1. Heart disease is one of the common morbidities. 2. Cancer is common in the general population; however, specific types are more common in the older patient. 3. Diabetes is common in patients over 40 years of age. 4. Heart disease, cancer, and diabetes combined are the most common morbidities in older patients.

3. Ms. Allen, 72 years old, is being evaluated by the nurse practitioner for type 2 diabetes. Because symptoms of endocrine disorders in older adults are vague, which of the following questions is best to ask? 1. Do you have any trouble carrying out your daily activities? 2. Can you tell me what you ate yesterday? 3. Have you seen a dentist in the last year? 4. Have you noticed that you have less hair?

3. Answer: 1 Page: 361 Feedback 1. Given the vagueness or atypical presentation of endocrine, metabolic, and nutritional disorders in older adults, specific questions directed at these conditions are imperative during the review of systems. Any alteration in the ability to carry out activities of daily living (ADLs) or instrumental ADLs due to fatigue or subjective weakness. 2. Given the vagueness or atypical presentation of endocrine symptoms in older adults, ask the patient or significant other(s) if there have been any acute changes in memory or mood. 3. Given the vagueness or atypical presentation of endocrine symptoms in older adults, ask the patient about any mouth ulcerations. 4. Given the vagueness or atypical presentation of endocrine symptoms in older adults, ask the patient about any noticeable dryness to hair or loss of hair.

3. Geriatric patients in end-of-life conditions suffer from many symptoms. The most prevalent symptoms include delirium, dyspnea, and pain. Which of the following is a common additional symptom for patients in end-of-life conditions? 1. Insomnia 2. Speech impairment 3. Hearing impairment 4. Tremors

3. Answer: 1 Page: 486 Feedback 1. Insomnia is common with geriatric patients in end-of-life conditions. In the context of palliative care, insomnia often coexists with other symptoms, such as pain, depression, and anxiety, and the presence of one often exacerbates the other, contributing to a decrease in quality of life 2. Depending on the condition, speech impairment may be involved but is not a common symptom associated with end-of-life conditions. 3. Depending on the condition, hearing impairment may be involved but is not a common symptom associated with end-of-life conditions. 4. Depending on the condition, tremors may be involved but is not a common symptom associated with end-of-life conditions.

3. Gayle is a 55-year-old woman who is suspected of having a brain tumor. The clinician orders diagnostic testing for thyroid-stimulating hormone (TSH) and magnetic resonance imaging (MRI) to confirm this. Which of the following statements is true regarding a brain tumor? 1. The symptom of headache is rare. 2. A brain tumor can be a malignant or benign neoplasm. 3. The etiology of brain tumors is often known. 4. Brain tumors occur more commonly in females than males.

3. Answer: 2 Page: 331 Feedback 1. Headaches occur more than half of the time with a brain tumor. 2. A brain tumor can be malignant or benign. 3. The etiology of brain tumors is largely unknown. 4. Brain tumors occur more commonly in males than females.

3. Biotransformation means chemical alteration of things such as nutrients, amino acids, toxins, and drugs in the body. In which organ does biotransformation occurs? 1. Intestines 2. Liver 3. Pancreas 4. Bile

3. Answer: 2 Page: 470 Feedback 1. The intestines are vital organs in the GI tract of our digestive system and their functions are to digest food and enable the nutrients released from that food to enter the bloodstream. 2. Biotransformation occurs in all body tissues, but primarily in the liver, where enzymatic activity alters and detoxifies the drug and prepares it for excretion. 3. The pancreas plays an essential role in converting the food we eat into fuel for the body's cells. 4. The bile is critical for digestion and absorption of fats and fat-soluble vitamins in the small intestine.

3. Which of the following is the single most signal symptom for possible breast cancer? 1. Nipple retraction 2. Breast discharge 3. Breast mass 4. Breast redness

3. Answer: 3 Page: 286 Feedback 1. Other than a mass in the breast, there are no other early signal symptoms. 2. Other than a mass in the breast, there are no other early signal symptoms. 3. Breast mass is the only early signal symptom of breast cancer. 4. Other than a mass in the breast, there are no other early signal symptoms.

3. While teaching a family about the risks of chronic illness for James, who is 69 years old and the patriarch of the family, the nurse practitioner includes which of the following information? 1. The American Heart Association reports that there is an increase in cardiovascular disease with low levels of smoking, excluding second-hand smoke. 2. Drinking high levels of alcohol contributes to an increase in chronic obstructive lung disease. 3. Obesity is correlated with a higher risk for type 2 diabetes. 4. The lower the body mass index (BMI) the lower the risk of chronic illness.

3. Answer: 3 Page: 475 Feedback 1. The American Heart Association tells us that there is a sharp increase in cardiovascular disease risk with even low levels of exposure to cigarette smoke, including secondhand smoke. 2. Binge or heavy drinking can lead to high-risk sexual behavior, unintentional injuries (e.g., motor vehicle crashes), falls, violence, and suicide. Excessive alcohol consumption can also lead to the development of high blood pressure, liver disease, some cancers, dementia, and alcohol dependence. 3. Obese individuals have an increased incidence of type 2 diabetes, sleep apnea, osteoarthritis, gallbladder disease, respiratory problems, some types of cancer, and depression. 4. The higher the BMI (and waist circumference), the higher the risk of cardiovascular disease, including hypertension, coronary artery disease, and stroke.

4. Mr. Jones, a 70-year-old male, visits the urgent care and presents symptoms of unilateral eye pain, visual blurring with halos around lights, red eye, and photophobia. He also reports nausea and vomiting. Visual acuity shows a loss in the affected eye. He is immediately referred for a complete ophthalmic examination. Which of the following conditions may Mr. Jones be experiencing? 1. Acute glaucoma 2. Conjunctivitis 3. Hyphema 4. Uveitis

4. Answer: 1 Page: 132 Feedback 1. Symptoms for acute glaucoma include unilateral eye pain, visual blurring with halos around lights, conjunctival injection, and photophobia. This requires immediate referral for a complete ophthalmic examination. 2. Symptoms for chronic glaucoma include tunnel vision, night blindness, and halos around lights. The treatment goal is to minimize the progression of the disease and preserve vision. 3. Hyphema is defined as the presence of blood within the aqueous fluid of the anterior chamber. The blood may cover most or all of the iris and the pupil, blocking vision partially or completely. The most common cause of hyphema is trauma. 4. Uveitis is a form of eye inflammation. It affects the middle layer of tissue in the eye wall (uvea). Symptoms include eye redness; eye pain; light sensitivity; blurred vision; dark, floating spots in the field of vision (floaters); and decreased vision. Possible causes of uveitis are infection, injury, or an autoimmune or inflammatory disease.

4. Parkinson's disease (PD) is a chronic progressive neurodegenerative disorder resulting in the loss of dopamine-producing cells in the brain. Which part of the brain is affected with PD? 1. Substantia nigra 2. Cingulate gyrus 3. Hippocampus 4. Hypothalamus

4. Answer: 1 Page: 333 Feedback 1. PD is a chronic progressive neurodegenerative disorder resulting in the loss of dopamine-producing cells in the substantia nigra. 2. The cingulate gyrus is a component of the limbic system and involved in processing emotions and behavior regulation. This part is not involved with PD. 3. The hippocampus is part of the limbic system and functions in the consolidation of information from short-term memory to long-term memory, as well as in spatial memory that enables navigation. This part is not involved with PD. 4. The hypothalamus links the nervous system to the endocrine system via the pituitary gland. This part is not involved with PD.

4. The nurse practitioner is considering ordering medications to help control an older patient's agitation. She chooses which of the following? 1. Psychotropic medications 2. Seizure medications 3. Stimulants 4. Depressants

4. Answer: 1 Page: 429 Feedback 1. There are two types of treatment for agitation: psychotropic medications and behavioral interventions. Psychotropic medications include antipsychotics, anxiolytics, and antidepressants. 2. Antiseizure drugs are used for manic-like symptoms. Evidence supports the use of antiseizure medications instead of antipsychotics. 3. Anxiolytics treat the symptoms of anxiety that often accompany agitation. 4. If there is evidence of depression, an antidepressant may be indicated.

4. The nurse practitioner assesses a patient's skin and finds an infectious lesion on the lower leg. The lesion is considered a secondary lesion. The nurse practitioner explains that a secondary lesion is one that: 1. Arises from changes to a primary lesion. 2. Is a complication of an underlying disease. 3. Is difficult to treat. 4. Is a normal sign of aging.

4. Answer: 1 Page: 97 Feedback 1. Secondary lesions (infections) arise from changes to the primary lesion. 2. Secondary lesions are not necessarily the result of an underlying disease. 3. Secondary lesions can be treated with medications or surgery. 4. Secondary lesions arise as a condition not normal to aging.

4. Mrs. Smith, 80 years old, has been vomiting and having bouts of loose stools for several days. She is taking several medications that are prescribed. The nurse practitioner conducts a medication review because: 1. An accounting of all prescribed and over-the-counter (OTC) medications may uncover reasons for her symptoms. 2. There could be drug-drug interactions leading to her symptoms. 3. Side effects of the gastrointestinal tract are not seen in many medications. 4. The administration of some of her medications may be appropriate.

4. Answer: 2 Page: 362 Feedback 1. A thorough review of all medications, including OTC and home remedies, is important to discern for alterations in the absorption of medications. 2. It is important to discern if drug-drug interactions are responsible for alterations in the absorption of medications. 3. Certain medications may be responsible for a new onset of anorexia, diarrhea, or constipation. 4. Essential, too, is a thorough review of how the patient and medical facility are administering the medications (timing the doses to not conflict with food). Finally, review patient adherence and understanding of the medication regimen.

4. Which of the following is most affected by pharmacokinetic change? 1. Distribution of drugs. 2. Elimination of drugs. 3. Absorption of drugs. 4. Metabolism of drugs.

4. Answer: 2 Page: 471 Feedback 1. Drug distribution relies on the bioavailability of the drug. 2. Kidney function changes, normal with aging, may prolong the half-lives of drugs. This is particularly important for drugs that are excreted unchanged in the urine and for drug categories known to be particularly nephrotoxic in older persons. 3. Drug absorption is generally thought to have a less significant impact on pharmacokinetics. 4. Being familiar with age-related pharmacokinetics of drugs is of the utmost importance when determining the initial and maintenance dosages, especially in the metabolism of drugs. When prescribing for an older patient, it is critical to understand whether the drug inhibits or induces the cytochromes P450 (CYP) enzymes.

4. When assessing a patient with a possible musculoskeletal condition, it is important to: 1. Know how much sleep the patient gets per night. 2. Be aware that older patients report any and all pains. 3. Determine ROM in all limbs. 4. Not refer to past injuries or conditions.

4. Answer: 3 Page: 306 Feedback 1. The amount of sleep the patient gets may be affected by pain, but knowing this will not affect the diagnosis. 2. It is important to remember that within the current cohort of older adults, a stoic attitude toward pain may be displayed despite the presence of an acute or chronic musculoskeletal condition. 3. Patients presenting with a functional limitation should be asked to demonstrate active ROM. Active ROM should be performed smoothly and effortlessly. 4. Ask if the patient has experienced any severe trauma in the past that may now be manifesting itself as an articular degeneration.

4. Mr. Person comes to the health-care clinic with a presenting symptom of not urinating as much as usual. The nurse practitioner assesses Mr. Person by asking which of the following? 1. How much do you drink per day? 2. Is your urine a clear yellow color? 3. Do you experience urgency and frequency of urination? 4. What medications have you taken recently or are you currently taking?

4. Answer: 4 Page: 228 Feedback 1. Patients may have nausea, vomiting, and diarrhea, leading to volume depletion. 2. The patient may have noticed a change in his urine consistency or color. 3. The patient may have symptoms directly resulting from alterations in kidney function, such as decreased to no urine output. 4. Specific questions regarding the use of medications that can cause renal injury, including NSAIDs and antihypertensive medications, such as ACE inhibitors and angiotensin receptor blockers, as well as recent antibiotic use, need to be asked of the patient.

4. Mr. Goldberg is diagnosed with late-stage Hodgkin's lymphoma. His wife, Barbara, noticed that his attention and awareness abilities are decreasing, fluctuating during the course of the day, and worsening at night. From which of the following could Mr. Goldberg be suffering? 1. Confusion 2. Dyspnea 3. Depression 4. Delirium

4. Answer: 4 Page: 486 Feedback 1. A state of confusion may be present with delirium. 2. Dyspnea or breathlessness derives from interactions of physiological, psychological, social, and environmental factors that may induce secondary physiological and behavioral responses. It is a distressing symptom at the end of life and affects quality of life. 3. Depression is a mood disorder that causes a persistent feeling of sadness and loss of interest. It is a distressing symptom at the end of life and affects quality of life. 4. Delirium is disturbance of attention and awareness accompanied by a change in baseline cognition, developing over a short period of time, and fluctuating during the course of a day, often worsening at night.

5. J. T. is a 69-year-old Caucasian male who presents with dysuria. During the examination, he asks the nurse practitioner if there is a chance he has cancer. The nurse practitioner tells him that before she can determine what is causing the problem she will need to perform a physical examination and order which of the following diagnostic tests? 1. Computed tomography (CT) scan of the abdomen and pelvis 2. IV pyelogram 3. Urine dipstick and cytology 4. Ultrasound

5. Answer: 1 Page: 230 Feedback 1. CT scans should include both the abdomen and pelvis; scans need to be done with and without contrast, and they should include delayed images to identify defects in the collecting system. 2. CT scan is replacing the IV pyelogram as the procedure of choice. 3. Urine dipstick, cytology, and screening for tumor-specific molecular markers in the urine are not recommended for screening asymptomatic patients due to low specificity, low sensitivity, and cost, respectively. 4. Although CT scan provides better visualization of tumors than ultrasound, it may miss tumors less than 1 cm in size.

5. The nurse practitioner whose practice consists of geriatric-aged patients must consider which of the following when working with patients who experience multiple chronic illnesses? 1. Addictions in older adults are predicted to decline in the next 20 years. 2. Mental illness or behavior problems must be included as chronic conditions. 3. Fragility and disability typically have multiple chronic conditions. 4. Co-occurrence of mental and medical conditions is not necessarily disabling.

5. Answer: 2 Page: 476 Feedback 1. The number of older adults who require management of addictions is projected to double from 1.7 to 2.8 million in 2000 to 4.4 to 5.6 million in 2020. 2. High-risk individuals with MCCs include those with severe mental illness or behavioral health problems. 3. MCCs can contribute to frailty and disability, and older persons who are frail or disabled typically have MCCs. 4. Physical illness is often accompanied by mental health problems, and mental disorders are associated with an increased risk of a wide range of chronic physical conditions.

5. Which of the following is the best statement regarding dyspnea? 1. Smoking causes dyspnea. 2. Dyspnea is one of the most common reasons for visits to the emergency room for individuals with advanced disease. 3. Dyspnea is a rare symptom as patients approach the end of life. 4. Females are more likely to develop symptoms of dyspnea.

5. Answer: 2 Page: 488 Feedback 1. Smoking and comorbidities can contribute to the development of dyspnea. Smoking alone is rarely a cause in the development of dyspnea unless associated with a condition. 2. Dyspnea is one of the most common reasons for visits to the emergency room for individuals with advanced disease because it is distressing and occurs in 94% of patients with serious, advanced illness. 3. Dyspnea is the most reported symptom as patients approach the end of life. 4. There are no gender-specific differences in the development of dyspnea.

5. Mr. Borden is 79 years old and lives with his daughter. The daughter provides excellent care for her father and wants to know which changes in his heart and circulatory system are due to aging and which are due to lifestyle choices. The nurse practitioner replies: 1. The size of the heart increases with age. 2. The left ventricle hypertrophies in response to increasing in age. 3. The activity of the heart decreases in its ability to deal with stress, as well as to provide physical strength as one ages. 4. The patient's diet affects the ability of the heart to balance blood pressure.

5. Answer: 3 Page: 153 Feedback 1. The size of the heart remains essentially unchanged, although some increase in left ventricular wall thickness has been demonstrated even in older individuals who do not have cardiovascular disease. 2. Left ventricular hypertrophy is usually due to increased cardiac demand, most likely caused by an increase in peripheral resistance. 3. Structural and physiological changes in the aging cardiovascular system cause decreased capacity to endure stresses and a decline in ability pertaining to physical activity performance. 4. Baroreceptors become less sensitive with age, and the response to changes in blood pressure is often blunted.

5. Mr. Cummins, a 69-year-old Caucasian male, presents with an inability to move his right arm sufficiently. The nurse practitioner tells him that she will perform a physical examination and order tests to rule out which of the following? 1. Gout 2. Rheumatoid arthritis (RA) 3. Muscle weakness from a systemic condition 4. Osteoarthritis (OA)

5. Answer: 3 Page: 307 Feedback 1. Because gout is an inflammatory disease that results in deposits of sodium urate crystals in the joints, periarticular tissues, and kidneys, and because the patient is only having a problem with one joint, it is not likely that gout is the diagnosis. 2. RA is a chronic systematic inflammatory process evidenced by symmetrical polyarthritis; it is the most common inflammatory arthropathy. 3. To determine if a patient is having true muscle weakness, the patient should perform against the examiner's resistance. One side should be compared to the other and a numerical value for tested muscle strength recorded. Flexor and extensor muscles should be tested for strength. 4. OA encompasses both symptoms and the structural remodeling of articular cartilage with inflammation of synovitis and ligament.

6. The nurse practitioner is working in the emergency department and is called to assist with the diagnosis of a patient who just came in. Mr. Barn is 76 years old and complains of abdominal pain, and his serum amylase and lipase are three times the normal limit. The nurse practitioner confirms pancreatitis, as this meets which criteria? 1. CT severity index 2. Atlanta classification 3. APACHE II 4. Ranson criteria

6. Answer: 3 Page: 364 Feedback 1. The CT severity index developed by Balthazar (2002) is the only scoring system to use evidence of necrotic pancreas as criteria. 2. To rate severity, it uses prognostic signs (a score of 3 or higher on the Ranson criteria and an 8 or higher on the APACHE II) and presence of local pancreatic necrosis, peripancreatic fluid collection, etc. and organ failure. 3. The Atlanta classification is both used to diagnose acute pancreatitis and rate the severity of pancreatitis. Diagnosis is made based on the Atlanta classification when two of three criteria are met: abdominal pain consistent with acute pancreatitis, laboratory abnormalities (serum amylase and lipase three times the normal limit or higher), and imaging findings consistent with acute pancreatitis. 4. The Ranson criteria was the first system and remains the best known and most widely used, but has drawbacks in that it requires 2 days' worth of objective measures for the total score, 5, to be obtained on the first day and evaluated within 48 hours of the onset of pain. In addition, sensitivity is only 73% and 77% to predict mortality, and the threshold for abnormal values depends on whether the pancreatitis is caused by alcohol or gallstones.

6. There are over 35 assessment tools currently available to assist providers with the prescribing of medications. Which of the following consists of a list of potentially inappropriate medications (PIMs) to be avoided in older adults? 1. STOPP 2. START 3. Beers criteria 4. ADRs

6. Answer: 3 Page: 472 Feedback 1. STOPP is a screening tool to determine potentially inappropriate prescriptions for older people. It consists of 65 clinically important criteria that relate to PIMs. 2. START is a screening tool to alert doctors regarding correct treatment. It contains 22 criteria that are supported by evidence. It reminds prescribers to consider certain drugs that are appropriate for specific conditions but that may be omitted. 3. Beers criteria consists of a list of PIMs to be avoided in older adults; they are listed by drug category and by diagnosis. 4. ADRs are what we are seeking to avoid with the different screening tools for prescribing medications.

6. Mrs. Jones, 80 years old, comes to the clinic for management of heart failure, diabetes, and depression. The nurse practitioner assesses that this patient is not compliant with the medication plan and has missed a couple of recent appointments. The nurse practitioner analyzes this information and concludes that perhaps Mrs. Jones is experiencing treatment burden because: 1. At the age of 80 years, she may not understand the workload she is carrying daily. 2. The comorbidities Mrs. Jones is dealing with are similar and do not necessitate many different specific tasks. 3. She is supposed to deal with several activities regarding her health daily. 4. The depression may inhibit her from proper self-care.

6. Answer: 3 Page: 476 Feedback 1. That "work" might include medication management, self-monitoring of blood pressure or blood sugars, visits to providers, laboratory tests, and lifestyle changes. 2. Many chronic conditions, such as heart disease, diabetes, and arthritis, share common challenges associated with their self-care management. 3. Patients coping with MCCs are especially vulnerable to a sense of burden with their treatment regimen because they are required to engage in a complex array of self-care activities on a daily basis. 4. Adjusting to the psychological and social demands, and decreased energy and function, while trying to engage in productive interactions with health-care providers, is a challenge.

6. Pain is categorized into different types. Which of the following type of pain originates in the thoracic, abdominal, and pelvic viscera and presents diffused pain that often radiates? 1. Neuropathic pain 2. Somatic pain 3. Visceral pain 4. Mixed pain

6. Answer: 3 Page: 490 Feedback 1. Neuropathic pain is nerve pain due to dysfunction of the nervous system. 2. Somatic pain originates from cutaneous, bone, and musculoskeletal tissues. 3. Visceral pain originates in the thoracic, abdominal, and pelvic viscera. It is diffuse pain and often radiates. It is a poorly localized "achy and dull" sensation in the abdomen or pelvis. 4. Mixed pain is a combination of nociceptive and neuropathic pain. Mixed pain syndromes are very common and include a large proportion of low back pain, cancer pain, chronic postoperative pain, as well as most forms of osteoarthritis.

6. The daughter of a 70-year-old woman asks the nurse practitioner why her mother has had a change in functional ability recently. The mother has regularly been drinking for 40 years, however, her behavior is changing. The nurse practitioner replies that: 1. The concentration of alcohol in her body remains the same as she ages. 2. Her change in behavior has nothing to do with any drugs she is taking. 3. There are different types of alcoholism in older adults, and the change in behavior is related to the specific type. 4. Physical changes due to aging affect the body's metabolism of alcohol differently.

6. Answer: 4 Page: 433 Feedback 1. The changes associated with aging may increase the concentration of alcohol in the blood. 2. Another major concern associated with alcohol misuse in the older adult is the increased occurrence of drug-alcohol interactions. 3. Patterns of alcohol use disorder in older adults have been divided into two categories: early onset and late onset. 4. Concerns about alcohol consumption in the older adult are directed primarily toward the physiological changes that accompany aging and the problems posed by regular alcohol consumption.

7. PVD is a disease that alters blood flow to or from the extremities and vital organs other than the heart. According to studies, which of the following remains the most important risk factor for PVD? 1. Smoking 2. Hypertension 3. Family history 4. Hypercoagulopathy

7. Answer: 1 Page: 219 Feedback 1. Smoking remains the most important risk factor to PVD. 2. Hypertension is an associated risk factor to PVD. 3. A strong family history of the disease is an associated risk factor to PVD. 4. Hypercoagulopathy is an associated risk factor to PVD.

7. Women developing ovarian cancer may be asymptomatic, making it hard to detect until the condition has worsened. Which of the following is the best initial diagnostic tool for suspected ovarian cancer? 1. Transvaginal ultrasonography 2. Pelvic examination 3. Laparoscopy 4. Complete blood count (CBC)

7. Answer: 1 Page: 294 Feedback 1. Transvaginal ultrasonography is the best initial study for suspected ovarian cancer. 2. Pelvic examination gives limited diagnostic results, as ovarian enlargement cannot always be palpated. 3. Laparoscopy is performed after a histopathological tissue report has been confirmed. 4. A CBC is also part of the initial evaluation, but secondary to transvaginal ultrasonography.

7. The nurse practitioner calls Mr. Smith to come into the examining room. As he is coming in, the nurse practitioner observes his gait to assess for musculoskeletal conditions. One abnormal sign is: 1. Unbalance in gait. 2. Lack of a lurch. 3. Symmetrical leg movement. 4. Ability to sit in and get out of a chair without using arms.

7. Answer: 1 Page: 306 Feedback 1. An unbalanced gait is abnormal and a risk for falling. 2. Note if the patient is having difficulty in shifting weight from one leg to the other and instead relies on shoulder movement from side to side. This is called a lurch and is abnormal. 3. Observe for any limping or asymmetrical leg. 4. Another important aspect of the gait examination is to note the patient's ability to sit in and rise from a chair. If a patient relies on a rocking motion and/or arm strength to stand, this is abnormal and could point to quadricep weakness.

7. Ms. Jenny is 72 years old and has managed her type 2 diabetes for 30 years. This condition is costly to her in more than one way. Which complications are common? 1. Financial challenges. 2. Good visual acuity. 3. Good renal functioning. 4. Increased circulation to lower extremities.

7. Answer: 1 Page: 370 Feedback 1. Diabetes is costly and has high morbidity and mortality rates. The total direct and indirect costs of treating diagnosed diabetics in 2012 was 245 billion dollars. This translates to a cost 2.3 times higher than care for those without diabetes. 2. Morbidity includes blindness. 3. Morbidity includes end-stage renal disease. 4. Morbidity includes lower extremity amputations.

7. Mr. Smith, 82 years old, is seen by a nurse practitioner and is potentially diagnosed with treatment burden. The nurse practitioner recommends the Chronic Disease Self-Management Program (CDSMP). This program addresses which of the following skills? Select all that apply. 1. Techniques to deal with problems. 2. Decision making. 3. Using resources available. 4. Patient-practitioner relationship. 5. Taking action.

7. Answer: 1, 2, 3, 4, 5 Page: 476 Feedback 1. The CDSMP assists with five core skills of self-management that include problem solving. 2. The CDSMP assists with five core skills of self-management that include decision making. 3. The CDSMP assists with five core skills of self-management that include resource utilization. 4. The CDSMP assists with five core skills of self-management that include the patient-provider partnership. 5. The CDSMP assists with five core skills of self-management that include taking action.

7. Mr. Fitzgerald has seizure conditions due to a recent accident causing traumatic brain injury (TBI). A nurse practitioner is tasked with giving the patient and his family information on home care. Which of the following should be included in the patient/family education? 1. Referral to an internist. 2. Lifestyle considerations. 3. Use of protective gear. 4. Referral for group therapy.

7. Answer: 2 Page: 351 Feedback 1. Referrals are given as needed, but not as inclusive in education on home care for patients with seizure conditions. 2. Lifestyle considerations are important to the safety and well-being of the patient. 3. Use of protective gear may be helpful, but is not a part of educating for home care of a patient with seizure conditions. 4. Group therapy will not help with home care for a patient with seizure conditions.

7. After consultation with the patient, which of the following is prevented by prescribing only necessary drugs? 1. Pharmacodynamics 2. Polypharmacy 3. Pharmacokinetics 4. Pharmacy

7. Answer: 2 Page: 472 Feedback 1. Pharmacodynamics is the branch of pharmacology concerned with the effects of drugs and the mechanism of their action. 2. Polypharmacy can be prevented by implementing parsimonious prescribing. 3. Pharmacokinetics is the branch of pharmacology concerned with the movement of drugs within the body. 4. Pharmacy is the science and technique of preparing and dispensing drugs.

7. Marie is a fibromyalgia sufferer. Her treatment program includes pain relief, antidepressant medications, and an exercise program. From which of the following types of pain might she be suffering? 1. Neuropathic pain 2. Somatic pain 3. Visceral pain 4. Mixed pain

7. Answer: 2 Page: 490 Feedback 1. Examples of neuropathic pain include diabetic neuropathy, neuropathy due to herpes zoster, and peripheral vascular disease. Psychological pain related to depression, anxiety, personality disorders, somatization disorders, and post-traumatic stress disorder also fit this category. 2. Examples of somatic pain include muscle pain, such as fibromyalgia or myofascial pain; inflammatory pain such as that resulting from acute injury or chronic inflammatory pain, including arthritis; and mechanical pain related to fractures, dislocations, or compression of tissue by bony structures. 3. Examples of visceral pain include appendicitis, gallstones, chronic chest pain, diverticulitis, and pelvic pain. 4. Mixed pain may arise from acute painful conditions with acute inflammatory mechanisms that may transition to a chronic condition in which more neuropathic mechanisms may ultimately predominate.

8. Mr. Newman developed a sudden onset of severe headache, weakness on one side of his body, difficulty speaking, and loss of balance. He was immediately admitted to the hospital with the diagnosis of hemorrhagic stroke. Which of the following is the manifestation of a hemorrhagic stroke? 1. Leak or rupture in an artery wall. 2. Leak or rupture in a venous wall. 3. Blockage in the heart. 4. Blockage of an artery.

8. Answer: 1 Page: 352 Feedback 1. A leak or rupture in an artery wall is the manifestation of hemorrhagic stroke. 2. A lead or rupture in a venous wall does not result in a stroke. 3. A blockage in the heart does not lead to a stroke. 4. Blockage of an artery is the manifestation of ischemic stroke.

8. Initial and ongoing assessment should be the protocol when prescribing medications. Designating and training office staff to do the initial medication review can save time but still requires review by the provider. Which of the following terms refers to a patient bringing in all current medications for review? 1. Brown bag 2. OTC 3. Rational polypharmacy 4. Magic bag

8. Answer: 1 Page: 472 Feedback 1. "Brown bag" is when the patient brings in all current medications for review. This may be helpful, but it is time consuming. Designating and training office staff to do the initial medication review can save time, but still requires review by the provider. 2. OTC stands for over-the-counter medications that do not require a prescription. 3. Rational polypharmacy refers to polypharmacy that is justified. 4. Magic bag is not a term that applies to polypharmacy.

8. Jose Mirrare is a 65-year-old male who is being treated for cirrhosis of the liver. Which of the following does the nurse practitioner include in his discharge teaching? Select all that apply. 1. Eliminate all alcohol consumption. 2. Attend an alcohol treatment program. 3. Do not self-medicate with over-the-counter (OTC) medications, including herbal products. 4. Eat three large meals of a balanced diet containing 4 to 5 mg protein/kg body weight per day. 5. Be cautious when driving, as you have encephalopathy.

8. Answer: 1, 2, 3 Page: 236, 237 Feedback 1. Chronic alcohol consumption, combined with a poor nutritional intake, contributes to cirrhosis. Drug-induced cirrhosis can occur in patients taking large doses of vitamin A, Aldomet, isoniazid, and methotrexate. Certain infectious diseases (tertiary syphilis, brucellosis, schistosomiasis) have predisposed patients to a risk of developing cirrhosis. 2. Recommend an alcohol treatment program and provide the telephone number for the nearest chapter of Alcoholics Anonymous. 3. Patients should be requested not to self-medicate with OTC medications, including herbal products. 4. Patients with cirrhosis should eat small, frequent meals of a balanced diet containing 1 to 1.5 mg protein/kg body weight per day, unless contraindicated by advanced disease. 5. Patients with encephalopathy should not be driving.

8. Mr. Weisner is a 65-year-old male who is experiencing pain in his joints. The nurse practitioner assesses which of the following? Select all that apply. 1. Movement of the joint in a relaxed position. 2. Movement of the joint in flexion. 3. Movement of the joint in extension. 4. Appearance of the limb being normal for his age. 5. Absence of any curvatures in the limb.

8. Answer: 1, 2, 3, 4, 5 Page: 307 Feedback 1. This is one facet of examination of the joints. 2. This is one facet of examination of the joints. 3. This is one facet of examination of the joints. 4. Enlargement and irregularity may indicate sequelae of childhood rickets, osteoporotic fractures, Paget's disease, OA, tophaceous gout, or bone tumors. 5. Excessive curvature and irregularity may indicate sequelae of childhood rickets, osteoporotic fractures, Paget's disease, OA, tophaceous gout, or bone tumors.

8. The nurse practitioner is ordering laboratory testing to confirm her suspicion of acute pancreatitis. Which of the following findings would confirm her suspicions? Select all that apply. 1. Trypsinogen activation peptide (TAP). 2. Blood urea nitrogen (BUN), creatinine, glucose. 3. An abdominal x-ray. 4. Amylase and lipase amount three times the normal. 5. Stool culture.

8. Answer: 1, 2, 3, 4, 5 Page: 363 Feedback 1. Another potential marker of acute pancreatitis is TAP that becomes markedly increased and may be useful in the detection of early acute pancreatitis. 2. Other useful laboratory tests are a metabolic panel to look at BUN, creatinine, and glucose levels. 3. The radiological evidence is used to confirm or exclude the clinical diagnosis, establish a cause, assess severity, and guide therapy. An abdominal x-ray is done first and helps to exclude other causes of abdominal pain, such as an obstruction or bowel perforation. 4. Amylase and lipase amounts are three times the normal amount of acute pancreatitis and are the most common laboratory markers used to establish a diagnosis of acute pancreatitis. Amylase is the most frequently ordered test, which rises within 6 to 12 hours of onset of pain, peaks around 24 hours, returns to normal within 3 to 5 days, and has a sensitivity of 81 to 95. Lipase has a higher sensitivity for acute pancreatitis (85% to 100%), rises within 8 hours of onset of symptoms, peaks at 24 hours, and returns to baseline within 8 to 14 days; it is now the preferred diagnostic blood test. 5. Other useful laboratory tests are a metabolic panel to look at BUN, creatinine, glucose, and calcium levels, as well as liver function tests, triglycerides, CBC, arterial blood gases, and a urinalysis.

8. The nurse practitioner identifies a late cardiac murmur in an older adult patient. A student nurse asks what condition this suggests. The nurse practitioner responds that this kind of murmur is most often due to which of the following? Select all that apply. 1. Weakening valves 2. Papillary muscle dysfunction 3. Mitral annular calcification 4. Heart blockage 5. Mitral stenosis

8. Answer: 2, 3 Page: 153 Feedback 1. An apical late systolic murmur occurs frequently in older persons, resulting from floppy valves that become regurgitant over time. 2. One of the two most common causes of mitral regurgitation is papillary muscle dysfunction. 3. One of the two most common causes of mitral regurgitation is mitral annular calcification. 4. Diastolic murmurs, which are almost always pathological in older persons, may be caused by heart blockage. 5. Diastolic murmurs, which are almost always pathological in older persons, may be caused by mitral stenosis.

8. Mrs. Hupp, 91 years old, lives with her daughter, Kim. Kim loves her mother and does everything she can to assist in providing a good quality life for her mother. Recently, Kim has been telling her nurse practitioner that she is experiencing lack of sleep, higher blood sugars, and anxiety. The nurse practitioner discusses which of the following with Kim? Select all that apply. 1. The mother's treatment burden. 2. Caregiver burden. 3. The possibilityof referring the mother to a long-term facility. 4. The possibility of the caregiver not practicing self-care and not experiencing good health. 5. Caregivers experience varying degrees of burden in caring for their parents.

8. Answer: 2, 3, 5 Page: 477 Feedback 1. Caregivers' tasks can include assistance with ADLs, instrumental ADLs, medication and treatment management, monitoring signs and symptoms, or navigating health-care appointments and health-care systems. 2. The demands of caregiving can create additional stress in the caregiver's life, commonly referred to as caregiver burden. 3. Caregiver burden also directly affects decisions to place care recipients in long-term care facilities. 4. Caregiver burden is associated with anxiety, depression, insomnia, poor self-care management, poor self-rated health, and higher mortality. 5. About 40% of caregivers report high burden, 18% report moderate burden, and 41% report a relatively low burden (National Alliance for Caregiving, American Association of Retired Persons).

8. Which of the following differential diagnoses should be considered with the diagnosis of peripheral arterial disease (PAD)? 1. Marfan syndrome 2. Atelectasis 3. Raynaud's phenomenon 4. Carpal tunnel syndrome

8. Answer: 3 Page: 219, 220 Feedback 1. Marfan syndrome is a genetic disorder that affects the body's connective tissue. It is not related to the diagnosis of PAD. 2. Atelectasis is a condition where some, or all, of the air-filled sacs (alveoli) inside the lungs collapse, thereby reducing the lungs' capacity to deliver oxygen to the body. It is not related to the diagnosis of PAD. 3. Raynaud's phenomenon is a type of vascular disease characterized by a pale to blue to red sequence of color changes of the digits, most commonly after exposure to cold, and should be considered with the diagnosis of PAD. Buerger's disease is also a differential diagnosis for this disease. 4. Carpal tunnel syndrome is a common condition that causes pain, numbness, and tingling in the hand and arm. It is not related to the diagnosis of PAD.

8. Physiological changes of aging and multiple comorbidities influence the dying process. The growing acceptance of hospice and palliative care is creating a more natural dying experience. Which of the following is true regarding the process of death? 1. Urinalysis is usually performed as a diagnostic test for further infection. 2. Age has no bearing on the risk of death. 3. The meaning and experience of death are often influenced by society. 4. The meaning and experience of death are often influenced by one's culture and ethnicity.

8. Answer: 4 Page: 493 Feedback 1. Diagnostic testing is not applicable for patients who are near end-of-life conditions. 2. Risk of death increases with advancing age. 3. Although society may sometimes influence the process of dying, the real meaning and experience of death are influenced by one's culture and ethnicity. 4. The meaning and experience of death are often influenced by one's culture and ethnicity. It is important to honor the patient's cultural beliefs, traditions, rites, and rituals.

9. The nurse practitioner has conducted a thorough health assessment for Mr. York, who is 83 years old. As the nurse practitioner synthesizes the information and creates the discharge instructions, consideration must be given to which of the following? Select all that apply. 1. His level of understanding of his health condition. 2. His ability to provide self-care. 3. His willingness to obtain preventive services. 4. His number of medication errors. 5. His basic reading level.

9. Answer: 1, 2, 4, 5 Page: 477 Feedback 1. Research indicates that low health literacy contributes to some health-related problems, including a poor understanding of health conditions. 2. Research indicates that low health literacy contributes to the inability to provide self-care. 3. Research indicates that low health literacy contributes to the low use of preventive services. 4. Due to the limited knowledge of their health problems, they make more medication errors, use more inpatient and emergency department care, and have worse overall health status and higher health care costs. 5. Health literacy involves using reading, writing, verbal, and numerical skills to obtain, process, and understand necessary health information to make appropriate decisions.

9. Ms. Jameson, 72 years old, comes to the clinic with concerns about her breathing. She states that she is having trouble with mucus collection and difficulty coughing it out. The nurse practitioner suggests which of the following? Select all that apply. 1. As you get older, the tissue in your lungs is not as flexible to aid in exhaling. 2. The muscles that help to inhale become stronger as you age. 3. The body contains less water as you age, and the mucus begins to dry out. 4. As you get older, there is an increase in the exchange of gases between the lungs and the blood vessels. 5. The cough reflex is normal, but the mucus is too thick.

9. Answer: 1, 3 Page: 154 Feedback 1. The aging process is characterized by a loss of elasticity and flexibility in collagen and elastin tissue components, which impedes the normal expiratory recoil of the lung. 2. A part of aging is the weakening of inspiratory muscles. 3. The concurrent decrease in body water composition dries mucous membranes. 4. Loss of elastin also affects the alveoli and the basement membrane of the capillary wall, where gas exchange occurs. A thickening occurs in both areas, limiting the amount of diffusion. 5. Because of decreased muscular strength, the cough reflex is not as forceful or as effective. This is one of the changes that occurs gradually as one ages and is hardly noticeable unless a physiological challenge or stress arises.

9. Ms. Jenny is 80 years old and is diagnosed with Clostridium difficile-associated diarrhea (CDAD). As she discusses her condition with her nurse practitioner, she asks how she obtained this disease. The nurse practitioner tells her which of the following? Select all that apply. 1. It used to be hospital-acquired only disease, but now this infection is in the community. 2. Risk factors include antibiotic exposure, being an older adult, and church or synagogue exposure. 3. Inflammatory bowel disease and compromised immunity can lead to CDAD. 4. Lesser known risk factors include prior hospitalization within several months and a state of malnourishment of the body. 5. The source of CDAD is currently unknown.

9. Answer: 1, 3, 4 Page: 242 Feedback 1. CDAD is no longer a hospital-acquired infection only, as outpatient incidences are rising. 2. Risk factors include antibiotic exposure, older adults, and health-care facility exposure. 3. The presence of inflammatory bowel disease and compromised immunity also play a role in increasing the risk for CDAD. 4. Lesser-known risk factors include intensive care admission, prior hospitalization within several months, ventilatory support, enteral feedings, use of histamine blockers and proton pump inhibitors, and malnourished states. 5. Bacterial spores are spread via the fecal-oral route and thrive on surfaces such as toilets, hospital equipment, and door handles. These spores have the potential to survive for months, even in harsh circumstances.

9. The aging process may alter the dynamic processes that drugs undergo to produce therapeutic effects. Which of the following are terms that refer to these changes? Select all that apply. 1. Pharmacodynamics 2. Prophylaxis 3. Pharmacology 4. Pharmacokinetics 5. Pharmacist

9. Answer: 1, 4 Page: 470, 471 Feedback 1. Pharmacodynamics refers to what the drug does to the body. It is the study of the relationship between the concentration of drug at the site of action and the biochemical and physiological effect. 2. Prophylaxis refers to treatment or drug given to prevent a condition or disease. 3. Pharmacology is the branch of science specific to the study of drugs. 4. Pharmacokinetics refers to what the body does to the drug. It describes the processes of drug absorption, drug distribution, drug metabolism, and drug excretion. 5. A pharmacist is a person who is professionally qualified to prepare and dispense medicinal drugs.

9. A nurse practitioner is performing an annual checkup for a female patient. In the physical examination of the breast, for which of the following should the nurse practitioner assess? Select all that apply. 1. Temperature 2. Symmetry 3. Dimpling 4. Lumps 5. Size

9. Answer: 2, 3, 4 Page: 280 Feedback 1. Although changes in temperature may signal abnormalities, it is secondary to the physical examination of the breast. 2. Asymmetry can present any abnormalities that contribute to breast conditions. 3. Dimpling or puckering of the skin on the breast is important to recognize, as these may be signs of breast cancer or other breast conditions. 4. A lump on the breast is important to recognize, as this is the single early sign of breast cancer. Lumps may also be indicative of other breast conditions. 5. Size alone is not important, but asymmetry in size is important to recognize, as it may be a sign of breast cancer or other breast conditions.

9. In a life-threatening illness, which of the following are tools to help minimize and relieve suffering? Select all that apply. 1. Establish an exercise program to keep the patient active. 2. Offer a support system to help patients live as actively as possible until death. 3. Provide relief from pain and other distressing symptoms. 4. Integrate psychological and spiritual aspects of patient care. 5. State that everything will be okay.

9. Answer: 2, 3, 4 Page: 485 Feedback 1. In this stage of illness, the patient is most likely in pain and bedridden. 2. Offering a support system to help patients live as actively as possible until death is one of the tools of palliative care. Support includes helping the family cope during the patient's illness and in his or her own bereavement. 3. Providing relief from pain and other distressing symptoms of death is one of the goals of palliative care. Patients at this stage are likely very uncomfortable. 4. Integrating the psychological and spiritual aspects of patient care in facing death is one of the goals of palliative care. It helps the patient and family stay in a positive frame of mind. 5. Affirming life and regarding death as a normal process is part of palliative care.

9. The nurse practitioner is assessing a 69-year-old man for possible bipolar disorder. Which of the following will contribute to diagnosing this patient? Select all that apply. 1. Complete blood count (CBC) and comprehensive metabolic panel, toxicology screen, urinalysis, thyroid function tests. 2. Electroencephalogram (EEG) and magnetic resonance imaging (MRI). 3. Mini-Mental State Examination. 4. Saint Louis University Mental Status (SLUMS). 5. Total body computed tomography (CT) scan.

9. Answers: 3, 4 Page: 437 Feedback 1. Diagnostic studies include a CBC and comprehensive metabolic panel, toxicology screen, urinalysis, thyroid function tests, rapid plasma reagin, HIV, electrocardiogram (EKG), and other individualized testing as indicated by the individual patient presentation and anticipation of treatment modalities. 2. In patients with new onset of psychosis, an EEG and MRI or CT scan may be appropriate to rule out medical pathologies. 3. Other screening tests for cognitive disorders include the Mini-Mental State Examination. 4. The Saint Louis University Mental Status (SLUMS) is more sensitive to detecting cognitive dysfunction. 5. CT scan may be appropriate to rule out medical pathologies.

1. Geriatric assessments can be challenging for the health-care provider due to delay of recognition and/or reporting of symptoms by the patient. Which of the following best describes what is used by health-care providers to address this differential? 1. Clinical decision-making tools. 2. When symptoms were noticed. 3. Change in function and impact of symptoms. 4. Symptoms from comorbidities.

1. Answer: 3 Page: 34 Feedback 1. Using clinical decision-making tools can be complicated because of different factors, such as patient denial of symptoms and their occurrences. 2. Using the time symptoms were noticed as the basis for assessment cannot be deemed an effective tool due to inaccuracies regarding the report of when symptoms first occurred. 3. Change in function as a measure of the impact of a symptom on the patient is an effective means of sorting out the differential. 4. Comorbidities can result in a delayed or atypical presentation.

1. Mr. and Mrs. Smith are both 75 years old and are seeing their nurse practitioner for their annual checkups. Mrs. Smith wonders why they each have trouble digesting different foods. It is inconvenient for her to fix different meals that they both can tolerate. The nurse practitioner explains that: 1. Men have more difficulty digesting vegetables. 2. Women have more difficulty digesting dairy products. 3. Men and women have different enzymes in their gastrointestinal tracts. 4. Organ systems progress differently for each person regardless of age.

1. Answer: 4 Page: 26 Feedback 1. There are no gender differences in types of food digested. 2. People of either gender may have an intolerance to dairy. 3. People of either gender have the same types of enzymes in their digestive systems. 4. People age at different rates, and within each person organ systems age at different rates.

10. The nurse practitioner is leading a class of seniors over age 65 years and is teaching about nutritional needs. One of the men asks why, even though he eats correctly according to the standards presented, he still feels weaker than he did 10 years ago. He also wonders why he gets more infections than he used to. Which of the following are helpful answers? Select all that apply. 1. I suggest that you exercise a little more than you are currently doing. 2. Some people experience a decrease in reserve energy. 3. For some people, the immune system weakens. 4. More viruses are being spread throughout the community. 5. I suggest that you see your primary caregiver for extensive testing.

10. Answer: 2, 3, 5 Page: 2, 3 Feedback 1. Improved nutrition may not be the factor affecting health of elders. 2. Many factors can influence the health of elders, including lifestyle and medications. 3. Many factors can influence the health of elders, including changes in the immune system. 4. Viruses and other infections are not the only considerations for infections. 5. Biochemical individuality is important in detecting asymptomatic abnormalities in older adults. Significant homeostatic disturbances in the same individual may be detected through serial laboratory tests, even though all individual test results may lie within normal limits of the reference interval for the entire group.

11. Nutrition counseling is part of health promotion. The nurse practitioner considers a teaching plan that includes which of the following information? Select all that apply. 1. Level of involvement in community activities. 2. Baseline information on current dietary intake. 3. Current activity patterns. 4. Current height and weight. 5. Health status information.

11. Answer: 2, 3, 4, 5 Page: 7, 8 Feedback 1. Active involvement in community activities does not reflect what the older person is eating. 2. The nurse practitioner needs the current state of nutrition to develop a plan for increasing or maintaining positive nutrition. 3. Knowing how much activity the person has affects the need for calories and nutrients. 4. Knowing the current height and weight aids in determining the body mass index, which is an indicator of normal weight or overweight. 5. Before initiating counseling about diet, obtain baseline information on other health status information.

11. Mr. Watson,75 years old, comes to the urgent care center with complaints of fever, fast heartbeat, a swollen gland under his right arm, and redness in his upper left arm that has hurt for 2 to 3 days. The patient says that he has had the redness in his arm for months without any difficulty. The nurse practitioner suspects which of the following? Select all that apply. 1. Influenza 2. Upper respiratory infection 3. Cellulitis of upper left arm 4. Necrotizing fasciitis 5. Lymphangitis

11. Answer: 3 Page: 103 Feedback 1. Influenza is systemic and not localized in any one area. 2. The patient has no respiratory symptoms. 3. Signs of cellulitis include worsening of erythema, edema, tenderness, and pain that has occurred for a few days. Symptoms are usually sudden. Systemic symptoms which indicate serious toxicity include fever, hypotension, and tachycardia. 4. Necrotizing fasciitis exhibits diffuse swelling of an arm or leg with bullae. 5. Systemic symptoms that indicate serious toxicity include fever, hypotension, tachycardia, leucocytosis, lymphadenopathy, and lymphangitis.

12. A major concern for older adults is safety. The nurse practitioner addresses safety issues when assessing the following. Select all that apply. 1. Presence of a gun in the home. 2. Fear of falling. 3. Strength of the patient. 4. Presence of rugs and other hazards in the home. 5. Urinary incontinence.

12. Answer: 1, 2, 3, 4, 5 Page: 8 Feedback 1. Possession of a firearm combined with depression, caregiver stress, irreversible illness, or decline in functional abilities can invite self-inflicted injury, suicide pacts, or other acts of violence. 2. Fear of falling causes a person to walk guardedly and unevenly. 3. Potential recommendations include exercise programs to build strength. 4. Rugs, furniture, and pets can become obstacles for the older person to stumble over. 5. Urinary incontinence, day or night, causes the person to rush to the bathroom and increases the risk of falling.

12. An elderly couple is discussing health concerns with the nurse practitioner. The couple is concerned that each of their individual blood studies show different results. One set of test results shows a significant decrease in blood sugar, but the test results of the partner does not show an equal decrease. Their physician does not seem concerned, and the couple is wondering why. How is it best for the nurse practitioner to respond? Select all that apply. 1. Studies show that each person's pattern of chemical make-up is different. 2. There is no difference between one person's range of blood results and that of any other person. 3. Each person's chemical make-up is the same as that of others of the same gender and age group. 4. Many factors affect an individual's chemical make-up. 5. Ranges of the values provided by the laboratory are correct for any age.

12. Answer: 1, 4 Page: 2, 3 Feedback 1. Individuals experience smaller variations in laboratory work than from the others in the same age group. 2. There are wider variations of laboratory results within a group of older people. 3. Laboratory values are determined by more than age and gender. 4. Laboratory values may vary as a result of nutrition, activity, and emotional status. 5. The reference values presented for the older adult cohort are not necessarily correct for the individual due to biochemical individuality.

12. Incorporating exercise into patients' lifestyles can be effectively achieved by including which of the following in your discussions with patients? Select all that apply. 1. Talking about the importance of exercise. 2. Encouraging goal setting and self-monitoring by the patient. 3. Recommending the purchase of exercise equipment to use at home. 4. Informing the patient about resources that are available in the community for group exercise. 5. Giving an illustration of the exercises.

12. Answer: 2, 4 Page: 21 Feedback 1. Handing a program or exercise prescription to the patient is more effective than just talking about it. 2. Goal setting and self-monitoring by the patient is very effective in persuading the patient to adopt a healthy lifestyle. 3. Recommending the purchase of exercise equipment to use at home may not be the best approach for different reasons, such as financial ability. 4. Informing patients about the resources that are available in the community for group exercise is a valuable adjunct to counseling. 5. Though not listed as something that could promote exercise, giving the patient illustrations for the exercise could be helpful when it is demonstrated and explained.

13. The goal of increasing physical activity is to promote a healthy lifestyle. Which of the following options help reduce the risk of injuries? Select all that apply. 1. Choosing physical activities that are appropriate for their current fitness level and health goals. 2. Remaining under the care of a health-care provider if they have chronic conditions or symptoms. 3. Choosing to do physical activities their friends are doing. 4. Enlisting spousal supervision and support. 5. Hiring a personal trainer.

13. Answer: 1, 2 Page: 22 Feedback 1. Choosing to do types of physical activities that are appropriate for a patient's current fitness level and health goals helps reduce the risk of injuries. 2. Being under the care of a health-care provider if a patient has chronic conditions or symptoms helps reduce the risk of injuries. 3. Choosing to do types of physical activities one's friends are doing is not the best option in reducing risk for injuries because of differences in physical ability or medical conditions. 4. Being under the supervision of one's spouse for support is a good facilitator, but it is not the best option in reducing the risk for injures because a spouse may not understand the impact of physical activity on physical and medical conditions. 5. A personal trainer may not have the proper training for more complicated conditions.

13. Albert is 72 years old and complains of burning upon urination. He has recently returned from a vacation to another country. The nurse practitioner conducts a physical examination and observes that there is a white discharge from his penis. What does the nurse practitioner include in the treatment plan? Select all that apply. 1. Teaching on safe sex practices. 2. Teaching about the incidence of contracting sexually transmitted infections (STIs), even at an older age. 3. Teaching that frequent testing and screening is needed if he continues to be sexually active. 4. Assuring the patient that while traveling there are limited opportunities for sexual encounters. 5. There is no need to assess an older patient's sexual preferences and practices.

13. Answer: 1, 2, 3 Page: 8 Feedback 1. Older adults are less likely to practice safe sex and use condoms. 2. Older people may be at increased danger from STIs because of the decreased perception of risk. 3. Current sexual history may determine the need for continued screening for STIs, HPV, and cervical cancer. 4. Encounters may be with fellow travelers, locals, or commercial sex workers. 5. Using the patient's sexual history, explore patient needs, preferences, and medical or psychological obstacles to sexual expression.

13. Bertha is an 85-year-old grandmother who has multiple morbidities. Family members report that Bertha is not eating much and does not seem as active as usual. They want the nurse practitioner to order tests to identify what is happening to her. The nurse practitioner considers which of the following before ordering any tests? Select all that apply. 1. Will the test results alter any of the diagnoses that she is currently addressing? 2. Will the results alter management of her health regimen? 3. What is the cost/benefits ratio? 4. Is the test the least invasive? 5. Is this testing acceptable to the patient?

13. Answer: 1, 2, 3, 4, 5 Page:28 Feedback 1. One of the considerations in ordering tests for diagnostic purposes is whether the test result will alter the diagnosis. 2. One of the considerations in ordering tests for diagnostic purposes is whether the test result will alter management of a condition. 3. The cost and benefits need to outweigh the risks. 4. Less invasive testing alleviates negative reactions to invasive procedures. 5. Evidenced-based practice includes the consideration of the patient's wishes.

13. A 59-year-old female was admitted to the hospital for malaise, headache, fever, and flu-like symptoms. She has a decreased appetite and is having trouble sleeping. After a couple of days, she complains to the nurse practitioner of itching, burning, and tingling pain around her waist. The nurse practitioner advises the nursing staff to observe for vesicles for a few days. The patient asks why she is so sick. What would be the nurse practitioner's best response? Select all that apply. 1. We are not certain at this point, however, these symptoms often occur before a break-out of herpes zoster. 2. You have some very general systemic symptoms, so we are waiting for more specific symptoms to appear. 3. Because you had chicken pox as a child, and you now have a depressed immune system, the chance of developing herpes zoster is high. 4. These symptoms are probably a strong case of influenza. 5. Herpes zoster is more common in people 55 years old and older.

13. Answer: 1, 3 Page: 106 Feedback 1. Patients usually experience itching, burning, or tingling pain at the site 4 to 5 days before the eruption appears. 2. Although there are general systemic symptoms, there is also itching, burning, and pain in the waist, which is leading to a herpes zoster diagnosis. 3. The patient has initial contact with VZV in the form of chicken pox. Individuals who are immunosuppressed are more likely to develop herpes zoster. 4. Herpes zoster is characterized by pain along the dermatomes and vesicles, which is not symptomatic of influenza. 5. This infection is most common in adults over 55 years old. The risk of herpes zoster increases with age.

13. Adam, 70 years old, is admitted for possible myasthenia gravis (MG). The nurse practitioner knows that MG is commonly seen in women between 20 and 40 years old. Adam, however, is experiencing an ocular form of MG, has dysphonia, and does not have any thymus abnormalities. Adam asks why the nurse is concerned about MG since he is older and this is a "young person's disease." Which of the following are the best answers? Select all that apply. 1. We need to assess for anything, just in case. 2. Some diseases show up in two different age and gender groups. 3. Older people often experience weakness, and we must rule out other conditions. 4. Your insurance will cover this, and we want to rule out as much as possible. 5. Signs and symptoms of illness are the same for each disease regardless of age.

13. Answer: 2, 3 Page: 5 Feedback 1. Ordering more tests than needed violates the principle of "Do no harm." 2. MG is bimodal and can be found in younger women, as well as in both men and women. 3. Knowledge of the bimodality of age onset of certain disease conditions will aid the nurse practitioner in avoiding misdiagnosis or delay in diagnosis due to lack of recognition. 4. There should be a need for ordering a test and value attributed to the results. 5. The manifestations of illness and disease in the elderly can be very different, even if the underlying pathological process is the same as in younger individuals.

14. The AHA Council on Nutrition, Physical Activity, and Metabolism identified patient facilitators to increasing physical activity in older adults. Which of the following is an example of patient facilitators? Select all that apply. 1. Frequent contact with prescriber. 2. Planned program. 3. Making time. 4. Purchasing new equipment. 5. Ignorance of exercise.

14. Answer: 1, 2 Page: 20 Feedback 1. Frequent contact with a prescriber has been seen to be a patient facilitator. 2. A planned program has resulted in facilitation toward health motivation. 3. Making time is a patient choice rather than a facilitator toward making changes. 4. Purchasing new equipment is not an example of a patient facilitator and may not be doable for all patients. 5. Ignorance of what to do is not a patient facilitator but a barrier to increasing physical activities.

14. Mary, 72 years old, goes to the pharmacy to pick up her prescriptions. The pharmacist asks if she has had her immunizations. Mary replies, "I had all my childhood shots, so I do not need any now." What should the nurse practitioner teach her about senior immunizations? Select all that apply. 1. Because the immune system may be weakened in older persons, immunizations are suggested. 2. Persons over the age of 50 years should have a yearly influenza vaccine. 3. The new Shingrix vaccine will help protect her from shingles. 4. The side effects of immunizations outweigh the benefits of immunizations. 5. The pneumococcal vaccine is given as a one-time dose.

14. Answer: 1, 2, 3 Page: 12 Feedback 1. The older person is susceptible to organisms that cause infection due to decreased immunity, nutrition, hydration, and other factors. 2. The influenza virus mutates each year and an updated version of the vaccine is needed to provide immunity. Influenza vaccine is now recommended annually for all adults over 50 years old, unless contraindicated. 3. Recently released Shingrix has been found to be more effective than the Zostavax, which was given previously. 4. Side effects are most often mild and localized. Viruses can cause serious or even lethal conditions. 5. Pneumococcal vaccine is recommended as follows: Administer one-time dose to PCV13-naïve adults at age 65 years, followed by a dose of PPSV23 12 months later.

14. In assessing functionality, the nurse practitioner first considers activities of daily living (ADLs), which include which of the following? Select all that apply. 1. Basic self-care. 2. Mobility. 3. Continence. 4. Ability to cook a meal. 5. Ability to interact with others within the community.

14. Answer: 1, 2, 3 Page:30,31 Feedback 1. The ability to bathe and dress oneself is a basic daily need. 2. The ability to move around and walk safely is a basic daily need. 3. The ability to maintain urinary and bowel control is a basic daily need. 4. It is not a basic daily need to be able to cook. It is possible to eat food that does not require cooking. 5. Industrial ADLs measure community interactions within the home and outside the home.

14. Mrs. Person, 82 years old, comes to the well clinic to see a nurse practitioner for a bump on her ear. This growth has been there for almost a year but has recently grown. The area around the growth appears inflamed. Why would the nurse practitioner suggest a biopsy? Select all that apply. 1. The growth is elevated and increasing in size. 2. The ear has high exposure to the sun. 3. There is inflammation around the growth. 4. The patient is 82 years old and reports having lived in the south of the United States for many years. 5. There is no concern about familial tendencies.

14. Answer: 1, 2, 3, 4 Page: 118 Feedback 1. Signs of malignancy include elevation; the original lesion may also have enlarged in size. 2. Common locations for skin cancers are the scalp, ears, lower lip, and dorsal side of the hands. 3. Signs of malignancy include inflammation of the lesion. 4. The incidence of all types of skin cancers increases with age and the degree and intensity of sun exposure. 5. Certain genetic predispositions can contribute to the development of skin cancer, and there is a familial tendency to develop melanoma.

15. The nurse practitioner is making patient rounds in a long-term care facility and is visiting Mr. Smith, 95 years old, who has a large amount of fungus growing from his toenails. The staff nurse asks what can be done to help alleviate this nail fungus. What does the nurse practitioner advise? Select all that apply. 1. Wash and completely dry the feet and toes daily. 2. Keep the patient's feet cool and dry. 3. Use aluminum acetate solution (Burow's solution). 4. Have the patient wear occlusive footwear. 5. Use clotrimazole (ointment, cream, or lotion).

15. Answer: 1, 2, 3 Page: 123 Feedback 1. The key to prevention of recurrence is to keep the area dry. Use a hairdryer to thoroughly dry the area after bathing. 2. The key to prevention of all types is to keep the skin cool and dry. 3. The use of aluminum acetate solution (Burow's) and the application of antifungal or absorbent powder have all been shown to prevent recurrence. 4. Avoiding occlusive footwear, wearing absorbent materials, and practicing good hygiene offer the best primary prevention. 5. Newer agents are more likely to cure tinea pedis than the older generation of antifungals, including clotrimazole, which is fungistatic, whereas terbinafine is fungicidal.

15. Mr. Adams is 90 years old. In the last few months he appears unable to comply with the health-care plan developed by the nurse practitioner. The nurse practitioner considers which of the following reasons for noncompliance when updating his home care plan? Select all that apply. 1. Polypharmacy 2. Treatment burden 3. Attending multiple appointments 4. Affording complex drug regimens 5. Cognitive ability only

15. Answer: 1, 2, 3, 4 Page: 5 Feedback 1. As a patient ages, this correlates with more medications having been ordered. 2. Patients with multimorbidity are known to have a treatment burden in terms of understanding and self-care management of their conditions. 3. Patients with multimorbidity are known to have a treatment burden in terms of understanding and self-care management of their conditions. 4. This burden entails affording complex drug regimens. 5. Though cognitive ability may be a factor, there is much more to consider when noncompliance is an issue.

15. The nurse practitioner performs a holistic assessment on an older patient. One component of this assessment is spirituality. Which of these options is appropriate to use to assess any spiritual needs? Select all that apply. 1. Use the HOPE assessment tool: source of Hope, a participant in an Organized religious group, Personal spirituality, Effect of spirituality on health care. 2. Ask the patient, "Are you at peace?" 3. Call a chaplain to conduct the assessment. 4. Wait until a sign of distress appears and then evaluate the source. 5. Administer the FICA assessment of spiritual needs.

15. Answer: 1, 2, 5 Page:31 Feedback 1. HOPE is an acceptable assessment tool for spiritual distress. 2. Asking a direct question that is not directed at a specific faith or religion is appropriate. 3. Chaplains are helpful, but the nurse practitioner can make a basic assessment of a patient. 4. An early assessment can help to avoid distress by implementing comfort measures early. 5. FICA (Puchalski & Romer, 2000): Faith or belief, Importance or influence, Community Address is an acceptable assessment tool.

15. Mr. and Mrs. Jones are preparing to take a trip to Europe. The nurse practitioner includes the following teaching about taking care of their health while gone. Select all that apply. 1. Shorter airplane flights provide more chance to ambulate and relieve pressure on the back and legs. 2. Generally speaking, the air in Europe is cleaner than the air in the United States, so you should have no problems with your respiratory conditions. 3. Plan to fill your prescriptions when you are in a larger city. 4. Depending on the specific areas you will visit, certain immunizations may be needed. 5. You will most likely function as well when traveling as when at home.

15. Answer: 1, 4 Page: 12, 13 Feedback 1. Long flights in cramped areas increase the risk of thromboembolic events. 2. Increased air pollution is a significant problem in many countries and affects pulmonary function. 3. The medications needed may not be available in some countries in Europe. Medicare does not cover medications filled outside of the United States. 4. Some areas of the world continue to see cases of yellow fever, malaria, and other diseases. When going to a vulnerable area, proper immunizations are required. 5. Some of the physiological and psychosocial changes that can occur with aging pose special risks during travel. How a patient functions at home may not be indicative of how well he or she will function in an unfamiliar environment.

15. Wandering in older adults can occur with disorientation or cognitive impairment and is characterized as locomotion in a seemingly aimless pattern. From which of the following conditions can wandering arise? Select all that apply. 1. Pneumonia 2. Delirium 3. Mood disorder 4. Dementia 5. Vertigo

15. Answer: 2, 4 Page: 88 Feedback 1. Pneumonia is a lung infection that can make an individual very sick but does not cause wandering. It often clears up in 2 to 3 weeks. 2. Delirium is a condition that causes extreme confusion and changes in cognition, which can cause wandering. 3. Mood disorder is a mental disorder but does not cause wandering. 4. Dementia is a condition that causes individuals to experience looseness of thought and confusion, which can cause wandering. 5. Vertigo is a sensation of whirling and loss of balance, associated particularly with looking down from a great height, or caused by disease affecting the inner ear or the vestibular nerve. Vertigo does not cause wandering.

15. To perform physical activity safely and reduce the risk of injuries and other adverse events, which of the following are advisable? Select all that apply. 1. Start fast to get acclimated to the activities. 2. Use their instincts. 3. Use appropriate protective gear. 4. Follow rules and policies. 5. Have an exercise "buddy."

15. Answer: 3, 4 Page: 21 Feedback 1. Starting fast could actually increase the possibility of injury. 2. There is no guarantee that instincts are correct. 3. Using appropriate protective gear is recommended to reduce the risk of injuries. 4. Rules and policies are made to inform and protect, therefore, following them will help to reduce the risk of injuries. 5. Having an exercise "buddy" may make the activity fun but will not necessarily reduce risk of injuries, especially if the buddy does not understand the concepts of exercises and physical activity physiology.

3. Mr. Osorio is a 65-year-old male who recently retired because of a medical condition. He has good family support and is very motivated to start a wellness program to prevent worsening of his condition. These factors are an example of: 1. Barriers 2. Incentives 3. Positive self-efficacy 4. Facilitators

3. Answer: 4 Page: 20 Feedback 1. Barriers are things that prevent or hinder patients from exercising. 2. Incentive is one kind of patient facilitator, like family support. 3. Positive self-efficacy is another kind of patient facilitator, like family support. 4. Facilitators are things that help a patient desire exercise and health promotion, so these are facilitators.

3. Mrs. Rizzo is an 80-year-old Caucasian female who is presenting symptoms of chest pain, weakness, and lightheadedness. Which of the following may be a possible differential diagnosis of her symptoms? 1. Ischemic colitis 2. Spinal cord compression 3. Diabetic neuropathy 4. Costochondritis

3. Answer: 4 Page: 38, 39 Feedback 1. Ischemic colitis is a possible differential diagnosis for fecal incontinence. 2. Spinal cord compression is a possible differential diagnosis for fecal incontinence. 3. Diabetic neuropathy is a possible differential diagnosis for fecal incontinence. 4. Costochondritis is the possible differential diagnosis for her symptoms of chest pain, weakness, and lightheadedness.

3. Mr. James is 91 years old. His daughter notices that he has bruises and lacerations on his arms and reports this to the nurse practitioner, who tells her that older people bruise easily due to their fragile blood vessels. The skin lacerations happen because he has thin skin. Even so, the nurse practitioner assures the daughter that she will investigate further to ensure that he is getting proper care. She says this because she understands that: 1. These markings on the patient's skin are part of aging skin. 2. Bruises and lacerations can indicate inadequate care. 3. The daughter needs assurance that her father is okay. 4. The patient is being abused.

3. Answer: 2 Page: 97 Feedback 1. Markings on the skin may be signs of aging, a disease, or maltreatment. 2. Poorly healing wounds or chronic pressure ulcers may signal a problem not only with the patient but with the caregiver's ability to provide adequate care. Welts, lacerations, burns, and distinctive markings may indicate a need for intervention. 3. This is a result of the nurse practitioner addressing it further rather than the reason for addressing it. 4. A professional cannot assume abuse without good reason.

3. Mr. Thomas comes in for a health examination appointment with his nurse practitioner. He asks why nurse practitioners are qualified to conduct these evaluations. Which of the following is the nurse practitioner's best response? The nurse practitioner: 1. Is concerned about health and not disease. 2. Saves the physician time in the office. 3. Can assess for minor conditions and refer to the doctor for other conditions. 4. Is prepared to assess health holistically.

3. Answer: 4 Page: 6 Feedback 1. The nurse practitioner is concerned about health promotion, disease prevention, and early diagnosis. 2. The nurse practitioner may spend more time with the patient than the doctor is able to. 3. The nurse practitioner is prepared to diagnose and plan treatment for many conditions. 4. The nurse practitioner brings a holistic orientation to health and wellness development and possesses knowledge of developmental tasks and the wellness-illness continuum.

4. Mrs. Williams, 80 years old, asks her nurse practitioner to order a new alternative therapy for her. Mrs. Williams believes that this botanical supplement will provide pain relief for her fibromyalgia. The nurse practitioner explains that this therapy is likely not covered by Medicare because: 1. The botanical is expensive and not approved for use. 2. Medicare only covers treatments that are approved according to the stringent guidelines of the U.S. Preventive Services Task Force (USPSTF). 3. There is no reason to use these botanicals when opioids are proven effective. 4. This botanical is not on the Medicare list of approved medications.

4. Answer: 2 Page: 6 Feedback 1. Although this botanical may not be approved by the U.S. Food and Drug Administration (FDA), its cost will depend on the distributor. 2. Medicare will only pay for A and B level recommendations that meet the USPSTF stringent evidence guidelines, leaving other beneficial interventions without coverage. 3. Opioids are not recommended for older patients. 4. It is important to use only prescribed and approved medications/treatments.

5. Which of the following should nurse practitioners suggest to counter common patient excuses for not exercising? 1. Join a gym. 2. Find an exercise "buddy." 3. Take a 10-minute walk every day. 4. Go to the YMCA.

5. Answer: 3 Page: 20 Feedback 1. Joining a gym is not the best suggestion to counter excuses not to exercise because having to go to the gym may become another excuse. There may also be financial and transportation issues adding to the reasons not to exercise. 2. Finding an exercise "buddy" may be helpful but still may impose the excuse of not being able to find one. 3. Taking a 10-minute walk every day is the best suggestion. With this suggestion a patient cannot make the excuse of not having time or equipment. 4. Going to the YMCA is not the best suggestion to counter excuses for the same reasons as joining a gym is not the best suggestion.

5. Which of the following is the correct term for the eye condition that presents a sterile mass on the eyelid that is painless and has no reddening? 1. Blepharitis 2. Entropion 3. Hordeolum 4. Chalazion

5. Answer: 4 Page: 138 Feedback 1. Blepharitis is an inflammation of the eyelids, usually resulting in redness, swelling, and itching. 2. Entropion is the inward turning of the upper or lower eyelid so that the lid margin rests against and rubs the eyeball. 3. Hordeolum is an acute, purulent area of inflammation in the meibomian gland, commonly called a stye. It typically contains bacteria and can occur internally or externally at the lid margin. Not all styes are sterile. 4. Chalazion is an eye condition that presents a sterile mass on the eyelid that is painless and has no reddening. It is caused by an inflammation and obstruction of a meibomian gland of the upper or lower eyelid. Chalazion can interfere with vision at times and develops a rubbery consistency.

5. When a patient is seen by a nurse practitioner, the care plan is developed keeping which of the following objectives in mind? 1. Increasing the use of the emergency department for injuries from falls. 2. Maintaining the patient living in his or her home. 3. Increasing safety checks in the home. 4. Increasing use of the Welcome to Medicare visit.

5. Answer: 4 Page: 6 Feedback 1. The objective is to decrease the use of the emergency department for falls. 2. There are many factors involved in the decision for an older adult to remain in the home. 3. Although it is useful to conduct home safety checks, they are not part of the Healthy People 2020 objectives. 4. The Healthy People 2020 program has set specific objectives for prevention in older adults; use of the Welcome to Medicare visit is one of those objectives.

6. Tina is a 60-year-old female presenting symptoms of coughing, shortness of breath, and purulent sputum. She is being treated with antibiotics, use of a nebulizer, and oral steroids. Which of the following conditions could be her diagnosis? 1. Bacterial pneumonia 2. Acute bronchitis 3. Chronic bronchitis 4. GERD

6. Answer: 3 Page: 44 Feedback 1. Her symptoms are not typical of bacterial pneumonia, which is a common lung infection where the lungs' air sacks become inflamed. Signal symptoms may be fever as high as 105°F, along with profuse sweating, bluish lips and nails, and confusion. 2. Antibiotics should not be used with acute bronchitis because it is viral. 3. Chronic bronchitis is a condition that presents with symptoms of coughing, shortness of breath, and purulent sputum, and is often treated with antibiotics, use of a nebulizer, and oral steroids. 4. GERD is a gastrointestinal condition that may be associated with coughing but is treated with an antihistamine and decongestant.

7. The nurse practitioner is concerned with primary prevention strategies. How can the nurse practitioner implement primary prevention strategies for an 80-year-old male patient who smokes? 1. Review home fire safety protocols, including the proper use of smoke alarms, and discuss smoking cessation. 2. Inform him that if he does not stop smoking, the nurse practitioner cannot see him again. 3. Have a conference with his family about his smoking. 4. Plan a family meeting with the patient to discuss benefits of his smoking cessation.

7. Answer: 1 Page: 115, 116 Feedback 1. Primary prevention includes educational programs designed to educate the public on safety. For example, the individual smoking in bed would hopefully benefit from smoking cessation programs in the community, as well as instruction in safety precautions. 2. Threatening refusal of care is not ethical. 3. The patient is at risk, not the family. 4. The fact that the patient smokes is not the issue; safety is the issue.

7. A gerontological patient is being examined for a report of pain in the shoulder. The nurse practitioner completes a thorough systemic examination because: 1. Older patients with one morbidity often express difficulties in general. 2. Arthritis of the shoulder is accompanied by other neurological symptoms. 3. Older patients with arthritis often experience pain in lower extremities. 4. The patient may not report significant signs and symptoms.

7. Answer: 1 Page: 5 Feedback 1. Older patients with late-onset rheumatoid arthritis experience joint involvement more often in the larger joints, such as the shoulder, and they also experience systemic symptoms such as fever, malaise, weight loss, and depression. 2. Older patients may express symptoms in any other physical systems. 3. Patients more often experience arthritis in smaller joints. 4. A thorough examination will detect objective indicators.

8. For which of the following evaluations should the patient be referred when sleep impairment is involved with chronic rhinitis? 1. Obstructive sleep apnea 2. Pharmacological therapy 3. Acupuncturist evaluation 4. Massage therapy

8. Answer: 1 Page: 148 Feedback 1. Patients with chronic rhinitis may be referred for evaluation of obstructive sleep apnea in the presence of sleep-disordered breathing. 2. Pharmacological therapy is directed at control of the specific patient symptoms. 3. Acupuncture evaluation may be helpful to improve sleep, but may not be helpful in treating rhinitis. 4. Massage therapy may be helpful to improve sleep, but it is not helpful in treating rhinitis.

8. Which of the following is the focus of additional studies about the relationship between physical activities and cognitive changes? 1. Diet and supplements 2. Provider and patients 3. Social support and patients 4. Medical conditions and medications

8. Answer: 1 Page: 19 Feedback 1. Diet and supplements are the focus of these additional studies. 2. Although provider and patient relationship is important, these are not the focus of these additional studies. 3. Although a patient's social support is important, it is not the focus of these additional studies. 4. Medical conditions and medications should be considered in physical activity, but it is not the focus of these additional studies.

8. Understanding that the current life expectancy is 79 years, the nurse practitioner plans a patient's health promotion while considering which of the following? Select all that apply. 1. Patient's health beliefs and goals. 2. Present levels of function. 3. Benefit of treatment. 4. Patient's involvement in a religious community. 5. Primary disease or condition affecting the patient.

8. Answer: 1, 2, 3 Page: 6 Feedback 1. The nurse practitioner develops a collaborative plan that includes consideration of the patient's health beliefs and goals. 2. The nurse practitioner considers present and anticipated levels of function. 3. Risks need to be considered in light of benefits offered by interventions. 4. The nurse practitioner may consider whether the patient is connected to a religious community but cannot prescribe or recommend any specific plan. 5. Many disorders in older adults encompass multiple risk factors that involve several systems and interventions to achieve outcomes; this presents a challenge when measuring and synthesizing evidence, and reporting outcomes.

8. The nurse practitioner is conducting a safety class with community-living older adults. Which of the following should she include in her teaching of risks of burns for this population? Select all that apply. 1. Thinner skin. 2. Less vascularity. 3. Diminished nerve function. 4. A weakened immune system. 5. The burden of various comorbidities leading to enhanced wound healing and reepithelialization after burn injury.

8. Answer: 1, 2, 3, 4 Page: 98 Feedback 1. As one ages, there are significant changes in the skin, which becomes thinner, providing a less effective barrier to external stimuli. 2. With aging, there are fewer appendages and decreased vascularity. 3. Thinner skin and diminished nerve function often result in a higher incidence of deeper burns. 4. Advanced age results in a weakened immune system. 5. Along with the burden of various comorbidities, the fragility of older skin leads to delayed wound healing and reepithelialization after burn injury.

9. Mr. Edwards is 76 years old and received a burn on his leg when he dozed off and dropped his cigarette. The nurse practitioner examines his leg for the degree of burn and classifies it as second degree with some third degree in the center. Mr. Edwards asks what that means and why it hurts so much. What is the best answer? Select all that apply. 1. It means that this is a serious, deep burn in the center, and a less deep burn around the sides. 2. It hurts because the nerve endings are exposed in the second-degree area. 3. It means that the burn is advancing and getting worse. 4. It hurts because the nerves are destroyed. 5. It hurts because the nerves in the second-degree areas are exposed to the outside and are stimulated.

9. Answer: 1, 2 Page: 98 Feedback 1. Deep dermal burns extend further into the dermis; third-degree burns involve the full dermis, extending into the subcutaneous tissue. 2. In these burns there is pain from exposed nerve endings, but by the second day, pain is often described more as pressure. 3. The first step in treatment is to stop the burn. 4. Destroyed nerves do not register pain. 5. Superficial dermal burns involve the dermis and are characterized by blisters. The underlying tissue is pink, moist, and hypersensitive to touch.

9. Evaluation for head and neck disorders should be performed with a systemic and thorough examination, including inspections of the face, head, and scalp. This includes which of the following? Select all that apply. 1. Palpating bones of the head for any anatomical irregularities. 2. Inspecting skin for any inflammation. 3. Inspecting the scalp and hair for any balding patterns. 4. Inspecting patient's features for any abnormalities. 5. Assessing for thoracic outlet syndrome.

9. Answer: 1, 3, 4 Page: 127 Feedback 1. Palpating bones of the head for any anatomical irregularities should be included in the inspection and examination of the face, head, and scalp. 2. Inspecting the skin is not a part of this examination, however, cranial nerve VII, the facial nerve, should be assessed at this time, noting any facial asymmetry, weakness, drooping of the lower eyelid, and unilateral paralysis. Cranial nerve VIII is the auditory and vestibular nerve, responsible for balance and orientation in space and auditory function. 3. Areas of uniform alopecia should be noted, along with discovery of any nits or seborrhea. 4. Inspecting the patient's features includes examination of facial expressions, presence of tremor, edema, or facial drooping. 5. Thoracic outlet syndrome is a group of disorders that occur when blood vessels or nerves in the space between your collarbone and your first rib (thoracic outlet) are compressed. This can cause pain in the shoulders and neck, and numbness in the fingers.


Related study sets

EMT Chapter 26 - Soft Tissue Injuries

View Set

Business Law 1 // Ch. 19 Breach of Contract and Remedies

View Set

Cardiovascular, Hematological-oncology, Immune

View Set

TestOut Chapter 1.4: Common TCP/IP Protocols

View Set

204 PrepU Craven ch. 36: Sensory Preception

View Set

Personal Finance Student Loan Study guide

View Set